15.04.2013 Views

Geo_Book_Answers

Geo_Book_Answers

Geo_Book_Answers

SHOW MORE
SHOW LESS

You also want an ePaper? Increase the reach of your titles

YUMPU automatically turns print PDFs into web optimized ePapers that Google loves.

<strong>Answers</strong> to Exercises<br />

0<br />

CHAPTER 0 • CHAPTER CHAPTER 0 • CHAPTER<br />

LESSON 0.1<br />

1. possible answers: snowflakes and crystals;<br />

flowers and starfish<br />

2. <strong>Answers</strong> might include shapes and patterns,<br />

perspective, proportions, or optical illusions.<br />

3. <strong>Answers</strong> will vary. Bilateral symmetry.<br />

4. A, B, C, and F<br />

5. A, B, D, and E<br />

6. 4 of diamonds; none<br />

7. The line of reflection is a line along the<br />

surface of the lake.<br />

8. One line of symmetry is a vertical line through<br />

the middle of the Taj Mahal, and the other is a<br />

horizontal line between the Taj Mahal and the<br />

reflecting pool. The pool reflects the building,<br />

giving the scene more reflectional symmetry than<br />

the building itself has.<br />

9. Designs will vary, but all should have 2-fold<br />

rotational symmetry.<br />

2 lines of reflectional<br />

symmetry<br />

It is impossible to have<br />

2 lines of reflectional<br />

symmetry without<br />

rotational symmetry.<br />

10. <strong>Answers</strong> will vary.<br />

11. <strong>Answers</strong> will vary.<br />

ANSWERS TO EXERCISES 1<br />

<strong>Answers</strong> to Exercises


<strong>Answers</strong> to Exercises<br />

LESSON 0.2<br />

1. compass and straightedge<br />

2. Answer will be the Astrid or 8-Pointed Star,<br />

possibly with variations.<br />

3. <strong>Answers</strong> will be a design from among the three<br />

choices.<br />

4. The first design has 4-fold symmetry and four<br />

lines of reflectional symmetry. The triangle has<br />

3-fold rotational symmetry and three lines of<br />

reflectional symmetry. The third design has 6-fold<br />

rotational symmetry if you ignore color and 2-fold<br />

rotational symmetry if you don’t.<br />

5. possible answer:<br />

2 lines of<br />

reflectional<br />

symmetry<br />

2 ANSWERS TO EXERCISES<br />

6. possible answer:<br />

7. 3 lines of reflectional<br />

symmetry<br />

3-fold rotational<br />

symmetry: rotated<br />

120, 240, 360


LESSON 0.3<br />

1. Design with reflectional symmetry is drawn on<br />

this background.<br />

2. Design should have rotational symmetry.<br />

Possible answer:<br />

3. possible answer:<br />

4. possible answer:<br />

5. Drawing should resemble the hexagons in the<br />

given figure, except that each radius, and the side<br />

of each hexagon, should measure 1 inch.<br />

ANSWERS TO EXERCISES 3<br />

<strong>Answers</strong> to Exercises


<strong>Answers</strong> to Exercises<br />

LESSON 0.4<br />

1. Possible answer: The designs appear to go in<br />

and out of the page (they appear 3-D). The squares<br />

appear to spiral (although there are no curves in<br />

the drawing). The spiral appears to go down as if<br />

into a hole. Lines where curves meet look wavy.<br />

Bigger squares appear to bulge out.<br />

2. Possible answer: When zebras group together,<br />

their stripes make it hard for predators to see<br />

individual zebras.<br />

4 ANSWERS TO EXERCISES<br />

3. Designs will vary.<br />

4. Designs will vary.<br />

5. Possible answers: 2-D: rectangle, triangle,<br />

trapezoid; 3-D: cylinder, cone, prism. The palace<br />

facade has one line of reflectional symmetry<br />

(bilateral symmetry).


LESSON 0.5<br />

1. possible answers: Scotland, Nigeria<br />

2. possible answer:<br />

3. possible answer:<br />

4. possible answer:<br />

Cut the middle ring.<br />

5.<br />

6. <strong>Answers</strong> will be a lusona from among the three<br />

choices.<br />

7. It means to solve a problem boldly and<br />

decisively or in a creative way not considered by<br />

others.<br />

8. The square knot has reflectional symmetry<br />

across a horizontal line. The less secure granny<br />

knot has 2-fold rotational symmetry.<br />

9. The result is a regular pentagon.<br />

ANSWERS TO EXERCISES 5<br />

<strong>Answers</strong> to Exercises


<strong>Answers</strong> to Exercises<br />

LESSON 0.6<br />

1. possible answers: Morocco, Iran, Spain,<br />

Malaysia<br />

2. Alhambra<br />

3. 2.1 cm<br />

4. in terms of the original square tile: 4-fold in the<br />

center (center of orange) or corner (center of<br />

white); 2-fold in the midpoint of the edge of the<br />

tile (where two orange shapes meet)<br />

6 ANSWERS TO EXERCISES<br />

6. Designs will vary.<br />

7. Designs will vary, but should contain a regular<br />

hexagon.<br />

8. Tessellations will var y.


CHAPTER 0 REVIEW<br />

1. possible answer: Islamic, Hindu, Celtic<br />

2. Sometimes the large hexagons appear as blocks<br />

with a corner removed, and sometimes they appear<br />

as corners with small cubes nestled in them.<br />

3. Compass: A geometry tool used to construct<br />

circles.<br />

Straightedge: A geometry tool used to construct<br />

straight lines.<br />

4. possible answer:<br />

5.<br />

6. possible answers: hexagon: honeycomb,<br />

snowflake; pentagon: starfish, flower<br />

7. <strong>Answers</strong> will vary. Should be some form of an<br />

interweaving design.<br />

8. Wheel A has four lines of reflectional symmetry;<br />

WheelChasfivelinesofreflectionalsymmetry.<br />

Wheels B and D do not have reflectional symmetry.<br />

9. Wheels B and D have only rotational symmetry.<br />

Wheels A and B have 4-fold, Wheel C has 5-fold,<br />

and Wheel D has 3-fold rotational symmetry.<br />

10, 11. Drawing should contain concentric circles<br />

and symmetry in some of the rings.<br />

12. The mandala should contain all the required<br />

elements.<br />

13a. The flag of Puerto Rico is not symmetric<br />

because of the star and the colors.<br />

13b. The flag of Kenya does not have rotational<br />

symmetry because of the spearheads.<br />

13c. possible answers:<br />

Japan<br />

Nigeria<br />

ANSWERS TO EXERCISES 7<br />

<strong>Answers</strong> to Exercises


<strong>Answers</strong> to Exercises<br />

<strong>Answers</strong> to Exercises<br />

CHAPTER 1 • CHAPTER CHAPTER 1 • CHAPTER<br />

LESSON 1.1<br />

1. sample answers: point: balls, where lines cross;<br />

segment: lines of the parachute, court markings;<br />

collinear: points along a line of the court structure;<br />

coplanar: each boy’s hand, navel, and kneecap,<br />

points along two strings<br />

2. PT ,TP <br />

3. any two of the following: AR ,RA ,AT ,TA ,RT ,TR <br />

4. any two of the following: MA ,MS ,AS ,AM ,SA ,SM <br />

5. A<br />

B<br />

6. 7.<br />

y<br />

8. AC or CA 9. PQ or QP<br />

10. TR or RT, RI or IR, and TI or IT<br />

11. 12. y<br />

13. mAB 14.3 cm 14. mCD 6.7 cm<br />

15–17. Check each length to see if it is correct.<br />

Refer to text for measurements.<br />

18. R is the midpoint of PQ. X is the midpoint of<br />

WY . Y is the midpoint of XZ.No midpoints are<br />

shown in ABC.<br />

19. possible answers:<br />

20.<br />

A<br />

S<br />

L<br />

P<br />

M<br />

or<br />

8 ANSWERS TO EXERCISES<br />

K<br />

B<br />

T<br />

1<br />

A C E D<br />

B<br />

AC CE <br />

BC CD <br />

Q<br />

D<br />

–3<br />

S<br />

11<br />

R<br />

3<br />

–3<br />

E<br />

x<br />

3<br />

x<br />

21. AB ,AC 22. PM,PN <br />

23. XY ,XZ 24.<br />

25. 26.<br />

27.<br />

28. 10<br />

29–31.<br />

32. Yes, P(6, 2), Q(5, 2), R(4, 6); the slope<br />

between any two of the points is 4<br />

1 .<br />

P<br />

Q<br />

33. possible answer:<br />

Q<br />

N<br />

34.<br />

R<br />

P<br />

Y<br />

T<br />

35. <strong>Answers</strong> will vary. Possible answers:<br />

8 cm<br />

y<br />

A<br />

y<br />

B<br />

–5<br />

A<br />

C<br />

X<br />

R´<br />

G<br />

11 cm<br />

C<br />

D<br />

Q´<br />

x<br />

5<br />

O<br />

–5<br />

P´<br />

y<br />

D<br />

x<br />

R<br />

B<br />

M<br />

Y<br />

8 cm<br />

A<br />

A (4, 0)<br />

N<br />

x<br />

11 cm<br />

B<br />

M


USING YOUR ALGEBRA SKILLS 1<br />

1. (3, 4)<br />

2. (9, 1.5)<br />

3. (5.5, 5.5)<br />

4. (6, 44)<br />

5. Yes. The coordinates of the midpoint of a<br />

segment with endpoints (a, b) and (c, d) are found<br />

a c<br />

by taking the average of the x-coordinates, , 2<br />

b d<br />

and the average of the y-coordinates, .Thus 2 the<br />

a c b d<br />

midpoint is , . 2 2<br />

6. (3, 2) and (6, 4). To get the first point of<br />

trisection, sum the coordinates of points A and B<br />

to get (9, 6), then multiply those coordinates by 1<br />

<br />

3 to<br />

get (3, 2). To get the second point of trisection, sum<br />

the coordinates of points A and B to get (9, 6), then<br />

multiply those coordinates by 2<br />

3 to get (6, 4). This<br />

works because the coordinates of the first point<br />

are (0, 0).<br />

7. Find the midpoint, then find the midpoint of<br />

each half.<br />

8a. Midpoints for Figure 1 are (5.5, 6.5); for<br />

Figure 2, (16, 6.75); and for Figure 3, (29.75, 5.5).<br />

8b. For these figures the midpoints of the two<br />

diagonals are the same point.<br />

ANSWERS TO EXERCISES 9<br />

<strong>Answers</strong> to Exercises


<strong>Answers</strong> to Exercises<br />

LESSON 1.2<br />

1. TEN, NET, E; FOU, UOF, 1;<br />

ROU, UOR, 2<br />

2.<br />

N<br />

3.<br />

4.<br />

5. S, P, R, Q; none in the second figure<br />

6. possible answer:<br />

A<br />

D<br />

7. 90°<br />

8. 120°<br />

9. 45°<br />

10. 135°<br />

11. 45°<br />

12. 135°<br />

13. 30°<br />

14. 90°<br />

15. Yes; mXQA mXQY 45° 90° 135°,<br />

which equals mAQY.<br />

16. 69°<br />

17. 110°<br />

18. 40°<br />

19. 125°<br />

20. 55°<br />

21. SML has the greater measure because<br />

mSML 30° and mBIG 20°.<br />

22.<br />

23.<br />

A T<br />

G<br />

M<br />

B<br />

A 44<br />

I<br />

C<br />

B<br />

S<br />

90<br />

B<br />

10 ANSWERS TO EXERCISES<br />

L<br />

24.<br />

25.<br />

26. no<br />

C<br />

A<br />

27.<br />

28.<br />

29.<br />

30. One possibility is 4:00.<br />

31.<br />

H<br />

32.<br />

C<br />

A<br />

T<br />

R<br />

135<br />

D E<br />

22<br />

22<br />

Y<br />

90<br />

6 8<br />

33. G T<br />

I<br />

D<br />

T H<br />

A<br />

67.5 67.5<br />

A<br />

N<br />

D<br />

N<br />

O<br />

S


34.<br />

B<br />

35. MY; CK; mI 36. SEU; EUO; MO<br />

37. x 54° 38. y 102°<br />

39. z 32° 40. 288°<br />

41. 242° 42. They add to 360°.<br />

43. no<br />

44. You will miss the target because the incoming<br />

angle is too big.<br />

Laser light<br />

source<br />

W<br />

Target<br />

O<br />

I<br />

Mirror<br />

T<br />

45. 180 km. The towns can be represented by<br />

three collinear points, P, S, and G.Because S is<br />

between P and G, PS SG PG.<br />

46.<br />

12 cm<br />

47.<br />

48. MS DG means that the distance between<br />

M and S equals the distance between D and G.<br />

MS DG means that segment MS is congruent<br />

to segment DG. The first statement equates two<br />

numbers. The second statement concerns the<br />

congruence between two geometric figures.<br />

However, they convey the same information and<br />

are marked the same way on a diagram.<br />

49. <strong>Answers</strong> will vary. Possible answer.<br />

40 70<br />

A 4 cm B<br />

6 cm 6 cm<br />

4.36 cm<br />

2.18 cm 2.18 cm<br />

C<br />

40<br />

70<br />

D 6 cm<br />

E<br />

F<br />

ANSWERS TO EXERCISES 11<br />

<strong>Answers</strong> to Exercises


<strong>Answers</strong> to Exercises<br />

1–3. possible answers:<br />

1.<br />

D<br />

2.<br />

3.<br />

4.<br />

5.<br />

6.<br />

7.<br />

8.<br />

45<br />

O G<br />

R<br />

B<br />

A<br />

D<br />

T E<br />

M<br />

A<br />

C<br />

40<br />

I G<br />

A<br />

P E<br />

R<br />

B<br />

B<br />

50<br />

140<br />

C<br />

LESSON 1.3<br />

9. B is a Zoid. A Zoid is a creature that has in its<br />

interior a small triangle with a large black dot<br />

taking up most of its center.<br />

10. A good definition places an object in a class<br />

and also differentiates it from other objects in that<br />

class. A good definition has no counterexamples.<br />

12 ANSWERS TO EXERCISES<br />

E<br />

G<br />

S<br />

D<br />

D<br />

E<br />

40<br />

11. The measures of complementary angles sum<br />

to 90°, whereas the measures of supplementary<br />

angles sum to 180°.<br />

12. No, supplementary angles can be unconnected,<br />

while a linear pair must share a vertex and<br />

a common side.<br />

13a. an angle; measures less than 90°<br />

13b. angles; have measures that add to 90°<br />

13c. a point; divides a segment into two congruent<br />

segments<br />

13d. a geometry tool; is used to measure the sizes<br />

of angles in degrees<br />

14.<br />

If AB and CD intersect at point P so that P is<br />

between A and B, and P is between C and D, then<br />

APC and BPD are a pair of vertical angles.<br />

15. true<br />

16. true<br />

17. true<br />

18. false<br />

19. false<br />

A<br />

B<br />

P<br />

C<br />

D


20. false<br />

21. true<br />

22. false; Though the converse is true, a<br />

counterexample is<br />

C T<br />

23. false<br />

D<br />

24. false<br />

C T<br />

25. possible answers: (3, 0), (0, 2), or (6, 6)<br />

26. possible answers: (2, 3) or (5, 1)<br />

27. The reflected ray and the ray that passes<br />

through (called the refracted ray) are mirror<br />

images of each other. Or they form congruent<br />

angles with the mirror.<br />

B<br />

50 80<br />

A<br />

40<br />

140<br />

C<br />

A<br />

Mirror<br />

A<br />

50<br />

Reflected<br />

segment<br />

28. One possible answer is A(8, 8), B(4.5, 6.5),<br />

C(11, 1).<br />

C<br />

29. 12 cm<br />

30. 36°<br />

31. possible answer:<br />

32.<br />

33.<br />

A<br />

–12 –6<br />

B<br />

1 pt.<br />

34a. 120°<br />

<br />

360°<br />

1<br />

, 2 <br />

3 3<br />

left<br />

60°<br />

34b. 360°<br />

1<br />

, 1 <br />

6 6<br />

missing<br />

34c. 360°<br />

<br />

9<br />

40°<br />

S<br />

T<br />

3 pt.<br />

6<br />

y<br />

R<br />

1 pt.<br />

3 pt.<br />

4 pt.<br />

2 pt.<br />

5 pt. 6 pt.<br />

x<br />

Infinitely<br />

many points<br />

0 pt.<br />

2 pt.<br />

ANSWERS TO EXERCISES 13<br />

<strong>Answers</strong> to Exercises


<strong>Answers</strong> to Exercises<br />

1.<br />

2. possible answers:<br />

3.<br />

LESSON 1.4<br />

4. octagon<br />

5. hexagon<br />

6. heptagon<br />

7. pentagon<br />

8–10. One possible answer for each is shown.<br />

8. pentagon FIVER<br />

9. quadrilateral FOUR<br />

10. equilateral quadrilateral BLOC<br />

11a. a polygon; has eight sides<br />

11b. a polygon; has at least one diagonal outside<br />

of the polygon<br />

11c. a polygon; has 20 sides<br />

11d. a polygon; has all sides of equal length<br />

12. One possibility is C and Y are consecutive<br />

angles; CY and YN are consecutive sides.<br />

13. 9; possible answer:<br />

14. AC, AD , BD, BE, CE<br />

15. TIN<br />

16. WEN<br />

17a. a 44, b 58, c 34<br />

17b. mT 87° and mI 165°<br />

14 ANSWERS TO EXERCISES<br />

18. PA FI and IVE ANC<br />

19. possible answer:<br />

20. possible answer:<br />

21. 84 in.<br />

22. 5.25 cm<br />

23. AB 14 m, CD 25 m<br />

24. complementary angles: AOS and SOC;<br />

vertical angles: OCT and ECR or TCE and<br />

RCO<br />

25.<br />

26. possible answer:<br />

5<br />

130 130<br />

5 cm<br />

20<br />

130<br />

E<br />

A B<br />

C D<br />

F<br />

27. All are possible except two points.<br />

0 pts.<br />

3 pts.<br />

5 pts.<br />

1 pt.<br />

4 pts.<br />

6 pts.


LESSON 1.5<br />

1. D 2. A<br />

3. C 4. B<br />

5. C<br />

6. C<br />

7.<br />

8. possible answer:<br />

2a<br />

9. possible answer:<br />

A<br />

10. possible answer:<br />

A<br />

11. possible answer:<br />

A<br />

12. possible answers: (1, 1), (1, 0), or (4, 3)<br />

13. possible answers: (3, 3), (3, 4), (11, 4),<br />

(11, 3), (0.5, 0.5), or (7.5, 0.5)<br />

14. possible answers: (3, 1), (1, 9), (2, 2),<br />

(4, 8), (0, 1), or (1, 6)<br />

15. possible answers:<br />

A<br />

4 cm<br />

A T<br />

A<br />

R<br />

b<br />

12 cm<br />

2a<br />

C<br />

4 cm<br />

120<br />

P<br />

45<br />

9 cm<br />

3a<br />

6 cm 6 cm<br />

B A<br />

40 40<br />

C<br />

80<br />

4 cm<br />

C<br />

3a<br />

b – 2a<br />

4 cm<br />

B<br />

B<br />

C<br />

Z<br />

C<br />

A<br />

7 cm<br />

45<br />

C<br />

9 cm<br />

S M L<br />

B<br />

B<br />

16. possible answers:<br />

A<br />

A<br />

17. true<br />

18. true<br />

19. False, a diagonal connects nonconsecutive<br />

vertices.<br />

20. False, an angle bisector divides an angle into<br />

two congruent angles.<br />

21. true<br />

(–3, 5)<br />

(–4, 1)<br />

40<br />

40<br />

22. (4,1) → (3,2) (1,1) → (2, 2) (2, 4) →<br />

(3, 1) (3, 5) → (2, 2) Yes, the quadrilaterals are<br />

congruent.<br />

23. Find the midpoint of each rod. All the<br />

midpoints lie on the same line; place the edge<br />

of a ruler under this line.<br />

24–26. Sample answers for 24–26<br />

24. P<br />

25.<br />

P<br />

E<br />

14 cm<br />

10 cm<br />

y<br />

F<br />

(2, 4)<br />

(1, 1)<br />

N T<br />

26. Q U<br />

C<br />

10 cm<br />

A<br />

10 cm<br />

D A<br />

x<br />

B<br />

E<br />

E<br />

N T<br />

ANSWERS TO EXERCISES 15<br />

A<br />

<strong>Answers</strong> to Exercises


<strong>Answers</strong> to Exercises<br />

LESSON 1.6<br />

1. first figure: quadrilateral ABCD with two<br />

congruent sides and one right angle; second<br />

figure kite EFGH; third figure: trapezoid IJKL<br />

with two right angles; fourth figure parallelogram<br />

MNPQ<br />

2. B 3. D<br />

4. F<br />

6. A, D, F<br />

5. C<br />

7. D I<br />

8.<br />

9.<br />

10.<br />

11.<br />

Z O<br />

E<br />

T<br />

R<br />

B<br />

N<br />

L<br />

A<br />

E U<br />

Q<br />

12. square<br />

13. possible answers: (2, 6), (2, 4); (6, 2),<br />

(2, 4); (3, 1), (1, 3)<br />

14. 90 cm<br />

15. (5, 6)<br />

16. S(9, 0), I(4, 2)<br />

17. S(3, 0), I(1, 5)<br />

18. A(7, 6), N(5, 9) or A(5, 2), N(7, 1)<br />

19a. Possible answer: Flip one triangle and align it<br />

so that a congruent pair of sides forms a diagonal<br />

of a kite.<br />

16 ANSWERS TO EXERCISES<br />

I<br />

F<br />

H<br />

G<br />

19b. Possible answer: Rotate one triangle so that<br />

a congruent pair of sides forms a diagonal of a<br />

parallelogram.<br />

20. There are three possibilities: a rhombus, a<br />

concave kite, or a parallelogram.<br />

21a. right triangles<br />

21b. isosceles right triangles<br />

22.<br />

23.<br />

24.<br />

25. no<br />

y<br />

5 cm<br />

72 72<br />

72 72<br />

72<br />

C (0, 5)<br />

B (4, 4)<br />

x<br />

A (5, 0)<br />

5 cm


LESSON 1.7<br />

1. <strong>Answers</strong> will vary. Sample answers:The green<br />

areaintheirrigationphotoisacircle,thewaterisa<br />

radius,and a path on the far side of the circle appears<br />

to be tangent to the circle.The wood bridge is an arc of<br />

a circle,and the railings are arcs of concentric circles.<br />

The horizontal support beam under the bridge is<br />

achord.<br />

2. three of the following: AB, BD, EC, EF<br />

3. EC<br />

4. AP, EP, FP,BP, CP<br />

5. five of the following: EF ,AE ,AB ,BC ,CD ,DF ,<br />

EB ,ED ,FC ,AC ,DB ,AF ,AD ,BF <br />

6. EDC<br />

or EFC<br />

,EBC<br />

or EAC<br />

<br />

7. two of the following: ECD<br />

,EDF<br />

,FEC<br />

,DEC<br />

,...<br />

8. FG , HB <br />

9. either F or B<br />

10. possible answers: cars, trains, motorcycles;<br />

washing machines, dishwashers, vacuum cleaners;<br />

compact disc players, record players, car racing,<br />

Ferris wheel<br />

11. mPQ 110°; mPRQ<br />

250°<br />

12.<br />

13. possible answers: concentric rings on cross<br />

sections of trees (annual rings),bull’s-eye or target,<br />

ripples from a rock falling into a pond.<br />

14.<br />

15.<br />

65<br />

215<br />

P Q<br />

16. Equilateral quadrilateral. (The figure is<br />

actually a rhombus, but students have not yet<br />

learned the properties needed to conclude that the<br />

sides are parallel.)<br />

17. equilateral; 3 to 1<br />

18.<br />

19. yes; yes<br />

20. yes; no<br />

21. no; no<br />

8<br />

–5<br />

–5<br />

y<br />

P<br />

s<br />

5<br />

–5<br />

5<br />

–5<br />

B<br />

y<br />

y<br />

A<br />

Q<br />

5<br />

5<br />

x<br />

x<br />

8 16<br />

x<br />

ANSWERS TO EXERCISES 17<br />

<strong>Answers</strong> to Exercises


<strong>Answers</strong> to Exercises<br />

22. 80°. The slices of pizza do not overlap, so<br />

60° x 140°, where x is the measure in<br />

degrees of the angle of the second slice.<br />

23.<br />

24. not possible<br />

25.<br />

26.<br />

27. about 0.986°<br />

28. 15°<br />

29.<br />

30.<br />

C A<br />

C B<br />

A 100<br />

F<br />

A<br />

P A<br />

T R<br />

18 ANSWERS TO EXERCISES<br />

B<br />

T<br />

31.<br />

32. not possible<br />

33. R<br />

34.<br />

35.<br />

36.<br />

37.<br />

T<br />

E<br />

K<br />

60 60<br />

120 120 60 120<br />

4a + 2b<br />

50 70 50 70<br />

55<br />

2p<br />

I 2p<br />

U<br />

2p 2p<br />

E<br />

120<br />

I<br />

Q<br />

G<br />

T<br />

120<br />

60


LESSON 1.8<br />

1. 2.<br />

3. 4.<br />

5. 6.<br />

7. 8.<br />

9. 60 boxes 10.<br />

5<br />

3<br />

4<br />

11. 12.<br />

13. B, D 14. B<br />

15. C 16. D<br />

17. A<br />

18. 19.<br />

20. true 21. true<br />

2<br />

3<br />

4<br />

22. False. The two lines are not necessarily in the<br />

same plane, so they might be skew.<br />

23. true 24. true<br />

25. true<br />

26. False. They divide space into seven or eight<br />

parts.<br />

27. true 28. (3, 1)<br />

29. perimeter 20.5 cm; m(largest angle) 100°<br />

30.<br />

13 cm<br />

120<br />

8 cm<br />

ANSWERS TO EXERCISES 19<br />

<strong>Answers</strong> to Exercises


<strong>Answers</strong> to Exercises<br />

LESSON 1.9<br />

1. Sample answer: Furniture movers might<br />

visualize how to rotate a couch to get it up a<br />

narrow staircase.<br />

2. yes<br />

3. W ; O M E N Nadine is ahead.<br />

4. 28 posts<br />

5. 28 days<br />

6. 0 ft (The poles must be touching!)<br />

7.<br />

8.<br />

9.<br />

10.<br />

A B<br />

A<br />

3 cm<br />

A<br />

A<br />

Quadrilaterals<br />

Trapezoids<br />

11. no<br />

12. C(2, 3), A(0, 0), B(0, 5), D(2, 1)<br />

13. Y(4, 1), C(3, 1), N(0, 3)<br />

14. (x, y) → (x 3, y 2)<br />

15. AB CP , EF GH ; i k, j k<br />

16. perimeter 34 cm<br />

17. Left photo: Three points determine a plane.<br />

Middle photo: Two intersecting lines determine a<br />

plane. Right photo: A line and a point not on the<br />

line determine a plane.<br />

20 ANSWERS TO EXERCISES<br />

B<br />

B<br />

A<br />

Polygons<br />

3 cm<br />

3 cm<br />

Triangles<br />

B<br />

Obtuse Isosceles<br />

18. pyramid with hexagonal base<br />

19. prism with hexagonal base<br />

20. pyramid with square base<br />

21. x 15, y 27<br />

22. x 12, y 4<br />

23.<br />

24.<br />

25. point, line, plane<br />

26. AB 27. AB <br />

28. vertex 29. AB <br />

30. AB CD 31. protractor<br />

32. ABC<br />

34. congruent to<br />

33. AB CD <br />

35. The distance is two times the radius.<br />

P<br />

36. They bisect each other and are perpendicular.<br />

P<br />

r<br />

PQ = 2r<br />

A<br />

B<br />

r<br />

Q<br />

Q


CHAPTER 1 REVIEW<br />

1. true<br />

2. False; it is written as QP .<br />

3. true<br />

4. False; the vertex is point D.<br />

5. true<br />

6. true<br />

7. False; its measure is less than 90°.<br />

8. false; two possible counterexamples:<br />

C<br />

9. true<br />

10. true<br />

11. False; they are supplementary.<br />

12. true<br />

13. true<br />

14. true<br />

15. False; it has five diagonals.<br />

16. true<br />

17. F<br />

18. G<br />

19. L<br />

20. J<br />

21. C<br />

22. I<br />

23. no match<br />

24. A<br />

25. no match<br />

26. T<br />

27.<br />

28.<br />

A<br />

T<br />

N<br />

P<br />

E Y<br />

K<br />

D<br />

B<br />

APD and APC<br />

are a linear pair.<br />

A<br />

G<br />

5<br />

7<br />

R<br />

I E<br />

C<br />

A<br />

O<br />

D<br />

S<br />

P<br />

APD and APC<br />

are the same angle.<br />

3<br />

P<br />

29.<br />

30.<br />

31.<br />

32.<br />

33.<br />

34.<br />

35.<br />

36.<br />

37.<br />

Y<br />

C<br />

2 in.<br />

R T<br />

A P<br />

40<br />

P<br />

5 in.<br />

A<br />

B<br />

125<br />

A<br />

T<br />

D<br />

E<br />

3 in.<br />

ANSWERS TO EXERCISES 21<br />

<strong>Answers</strong> to Exercises


<strong>Answers</strong> to Exercises<br />

38. 114°<br />

39. x 2, y 1<br />

40. x 12, y 4<br />

41. x 4, y 2.5<br />

42. x 10, y 8<br />

43. AB 16 cm<br />

44. 96°<br />

45. 105°<br />

46. 30°<br />

47.<br />

48. 66 inches<br />

49. 3 feet<br />

50. The path taken by the midpoint of the ladder is<br />

anarcofacircleoraquarter-circleiftheladder<br />

slides all the way from the vertical to the horizontal.<br />

Shed<br />

Quadrilateral<br />

Rectangle Square Rhombus<br />

Path of flashlight<br />

Trapezoid<br />

22 ANSWERS TO EXERCISES<br />

51. He will get home at 5:46 (assuming he goes<br />

inside before he gets blown back again).<br />

52. (2, 3)<br />

53.<br />

54.<br />

55.<br />

56.<br />

57.


<strong>Answers</strong> to Exercises<br />

CHAPTER 2 • CHAPTER 2<br />

CHAPTER 2 • CHAPTER<br />

LESSON 2.1<br />

1. “All rocks sink.” Stony needs to find one rock<br />

that will not sink.<br />

2. If two angles are formed by drawing a ray from a<br />

line, then their measures add up to 180°.<br />

3. 10,000, 100,000, ....Each term is 10 times the<br />

previous term.<br />

4. 5 ,1,....(Written 6 with the common<br />

denominator 6, the pattern becomes<br />

1 , 2 , 3 , 4 , 5 , 6<br />

6 6 6 6 6 6 ,....)<br />

5. 17, 21 6. 28, 36<br />

7. 21, 34 8. 49, 64<br />

9. 10, 24 10. 64, 128<br />

11. 12.<br />

13. 14.<br />

15. 16.<br />

17. 1, 4, 7, 10, 13 18. 15, 21, 28, 36, 45<br />

19. <strong>Answers</strong> will vary.<br />

20. sample answers:3,6,12,24,48,...and 4,8,12,<br />

16, 20, . . .<br />

21. <strong>Answers</strong> will vary.<br />

22. 7th term: 56; 10th term: 110; 25th term: 650<br />

23. Conjecture is false; 14 2 196 but 41 2 1681.<br />

24. 11,111 11,111 123,454,321. For the sixth<br />

line, if 111,111 is multiplied by itself, then the<br />

product will be 12,345,654,321. But 1,111,111,111 <br />

1,111,111,111 1,234,567,900,987,654,321.<br />

25. 26.<br />

27. 28.<br />

29. possible answers:<br />

30. sample answer:<br />

31. collinear 32. isosceles<br />

33. dodecagon 34. parallel<br />

35. protractor 36. radius<br />

37. diagonal 38. regular<br />

39. 90° 40. perpendicular<br />

41. sample answer: 42. sample answer:<br />

A G<br />

N T<br />

I<br />

43. sample answer: 44. possible answer:<br />

45. Methods will vary. It is not possible to draw a<br />

second triangle with the same angle measures and<br />

side length that is not congruent to the first.<br />

C<br />

40 60<br />

A 9 cm B<br />

A C Clearly AC <br />

does not<br />

bisect A<br />

or C.<br />

ANSWERS TO EXERCISES 23<br />

<strong>Answers</strong> to Exercises


<strong>Answers</strong> to Exercises<br />

1. See table below.<br />

2. See table below.<br />

3. See table below.<br />

1. (Lesson 2.2)<br />

LESSON 2.2<br />

n 1 2 3 4 5 6 … n … 20<br />

f (n) 3 9 15 21 27 33 … 6n 3 … 117<br />

2. (Lesson 2.2)<br />

n 1 2 3 4 5 6 … n … 20<br />

f (n) 1 2 5 8 11 14 … … 56<br />

3. (Lesson 2.2)<br />

n 1 2 3 4 5 6 … n … 20<br />

f (n) 4 4 12 20 28 36 … … 148<br />

4. (Lesson 2.2)<br />

Number of sides 3 4 5 6 … n … 35<br />

Number of triangles formed 1 2 3 4 … n 2 … 33<br />

5. (Lesson 2.2)<br />

Figure number 1 2 3 4 5 6 … n … 200<br />

Number of tiles 8 16 24 32 40 48 … 8n … 1600<br />

6. (Lesson 2.2)<br />

Figure number 1 2 3 4 5 6 … n … 200<br />

Number of tiles 1 5 9 13 17 21 … 4n 3 … 797<br />

7. (Lesson 2.2)<br />

Figure number 1 2 3 4 5 6 … n … 200<br />

Number of matchsticks 5 9 13 17 21 25 … 4n 1 … 801<br />

Number of matchsticks<br />

in perimeter of figure<br />

5 8 11 14 17 20 … 3n 2 … 602<br />

24 ANSWERS TO EXERCISES<br />

4. See table below.<br />

5. See table below.<br />

6. See table below.<br />

7. See table below.<br />

3n 4<br />

8n 12


8. 8n is steeper; the coefficient of n.<br />

f (n)<br />

50<br />

40<br />

30<br />

20<br />

10<br />

2<br />

9. C n H 2n2<br />

H C C C C C C C C<br />

10. y 3<br />

x<br />

2<br />

3<br />

11.<br />

L<br />

12. O<br />

H H H H H H H H<br />

H H H H H H H H<br />

E<br />

4<br />

Octane (C 8 H 18 )<br />

T<br />

4n 1<br />

4n 3<br />

3n 2<br />

6 8<br />

L<br />

8n<br />

M<br />

n<br />

Q<br />

Y<br />

H<br />

13.<br />

14.<br />

15. Márisol should respond by noticing that all<br />

the triangles José drew were isosceles, but it is<br />

possible to draw a triangle with no two sides<br />

congruent. In other words, she should show him a<br />

counterexample.<br />

16. Methods will vary. It is not possible to draw<br />

another triangle with the given side lengths and<br />

angle measure that is not congruent to the first.<br />

9 cm<br />

R E<br />

T C<br />

2<br />

2<br />

C<br />

45<br />

A 8 cm B<br />

2<br />

1 1<br />

1 1<br />

3 3<br />

1 1<br />

1 1 2<br />

17. She could try eating one food at a time;<br />

inductive.<br />

18. 2600<br />

ANSWERS TO EXERCISES 25<br />

<strong>Answers</strong> to Exercises


<strong>Answers</strong> to Exercises<br />

1.<br />

See table below.<br />

2. n 1, 36<br />

LESSON 2.3<br />

3. You can draw a diagonal from one vertex to all<br />

the other vertices except three: the two adjacent<br />

vertices and the vertex itself.<br />

4. n(n 3)<br />

,<br />

2<br />

560<br />

5. n(n 1)<br />

,595<br />

2<br />

6. n(n 1)<br />

,595<br />

2<br />

7. <strong>Answers</strong> will include relationships between<br />

points in Exercises 5 and 6 and vertices of a polygon.<br />

The total number of segments connecting n random<br />

points is the number of diagonals of the n-sided<br />

polygon, n(n 3)<br />

, 2 plus the number of sides, n.<br />

Thus, the total number of segments connecting n<br />

random points is n(n 3) 2n<br />

n(n 3) 2n<br />

<br />

2 2<br />

n2 3n<br />

2n n<br />

<br />

2<br />

2 n<br />

<br />

2 n(n 1)<br />

. 2<br />

1. (Lesson 2.3)<br />

26 ANSWERS TO EXERCISES<br />

<br />

2 <br />

8. Use n(n 1)<br />

2<br />

to get 780 direct lines. Use a central<br />

hub with a line to each house to get 40 lines. The<br />

art shows the direct-line solution and the practical<br />

solution for six houses.<br />

9. Use points for the 10 teams and segments<br />

connecting them to represent four games played<br />

between them. So, use 4 n(n 1)<br />

<br />

2 to get 180 games<br />

played.<br />

10. If n(n<br />

1)<br />

<br />

2 66, then n(n 1) 132. What<br />

two consecutive numbers multiply to equal 132?<br />

12 times 11. Thus, there were 12 people at the<br />

party.<br />

11. true<br />

12. true<br />

13. False; an isosceles right triangle has two sides<br />

congruent.<br />

14. false<br />

A<br />

E<br />

B C<br />

D<br />

AED and BED are<br />

not a linear pair.<br />

15. False; they are parallel.<br />

16. true<br />

17. False; a rectangle is a parallelogram with all of<br />

its angles congruent.<br />

18. False; a diagonal is a segment in a polygon<br />

connecting any two nonconsecutive vertices.<br />

19. true<br />

20. 5049<br />

Lines 1 2 3 4 5 … n … 35<br />

Regions 2 4 6 8 10 … 2n … 70


LESSON 2.4<br />

1. inductive; deductive<br />

2. mB 65°; deductive<br />

3. inductive<br />

4. DG 258 cm; deductive<br />

5. 180°; 180°; The sum of the five angles is 180°;<br />

inductive.<br />

64<br />

27<br />

20<br />

37<br />

25<br />

32<br />

36<br />

6. LNDA is a parallelogram; deductive.<br />

7. Possible answer: AB CD, so AB BC <br />

CD BC.Because AB BC AC and CD <br />

BC BD, AC BD, so AC BD.<br />

8. 3; 10; 7<br />

9. Just over 45°; if m 90°, then 1 m 2 45°.<br />

10. 48°; 17°; 62°; mCPB<br />

11. Possible answers: Because mAPC <br />

mBPD, add the same measure to both sides to get<br />

mAPC mCPD mBPD mCPD.By<br />

angle addition, mAPC mCPD mAPD<br />

and mBPD mCPD mCPB.Therefore,<br />

mAPD mCPB.<br />

12. <strong>Answers</strong> will vary.<br />

13. The pattern cannot be generalized because<br />

once the river is straight, it cannot get any shorter.<br />

14. 900, 1080<br />

15. 75, 91<br />

16. 4<br />

5 ,12<br />

17.<br />

18.<br />

19.<br />

20.<br />

21. Sample answer: There were three sunny days<br />

in a row, so I assumed it would be sunny the fourth<br />

day, but it rained.<br />

22. L 23. M<br />

24. A 25. B<br />

26. E 27. C<br />

28. G 29. D<br />

30. H 31. I<br />

32.<br />

W<br />

33.<br />

34.<br />

35.<br />

I<br />

B<br />

D<br />

L<br />

O<br />

K<br />

140<br />

G<br />

C<br />

D<br />

N<br />

D<br />

T<br />

E<br />

O<br />

ANSWERS TO EXERCISES 27<br />

<strong>Answers</strong> to Exercises


<strong>Answers</strong> to Exercises<br />

LESSON 2.5<br />

1. a 60°, b 120°, c 120°<br />

2. a 90°, b 90°, c 50°<br />

3. a 77°, b 52°, c 77°, d 51°<br />

4. a 60°, b c 120°, d f 115°, e 65°,<br />

g i 125°, h 55°<br />

5. a 90°, b 163°, c 17°, d 110°, e 70°<br />

6. The measures of the linear pair of angles add up<br />

to 170°, not 180°.<br />

7. The angles at which he should cut measure 45°.<br />

8. Greatest: 120°. Smallest: 60°. One possible<br />

explanation: The tree is perpendicular to the<br />

horizontal. The angle of the hill measures 30°.<br />

The smaller angle and the angle between the hill<br />

and the horizontal form a pair of complementary<br />

angles, so the smaller angle equals 90° 30° 60°.<br />

The smaller angle and larger angle form a linear<br />

pair, so the larger angle equals 180° 60° 120°.<br />

9. The converse is not true. ; sample counterexample:<br />

A<br />

55<br />

125<br />

B<br />

10. each must be a right angle<br />

11. Let the measures of the congruent angles be x.<br />

They are supplementary, so x x 180°, 2x 180°,<br />

x 90°. Thus each angle is a right angle.<br />

12. The ratio is 1. The ratio does not change as<br />

long as the lines don’t coincide. Because the<br />

demonstration does not explain why, it is not<br />

a proof.<br />

13. P<br />

14.<br />

3<br />

A<br />

5<br />

28 ANSWERS TO EXERCISES<br />

T<br />

22. (Lesson 2.5)<br />

15.<br />

16.<br />

17.<br />

6<br />

18. Possible answer: All the cards look exactly as<br />

they did, so it must be the 4 of diamonds, because<br />

it has rotational symmetry while the others do not.<br />

19. 22.5°<br />

20.<br />

21. C n H 2n<br />

H H<br />

H C C<br />

H H<br />

22. See table below.<br />

23. handshake problem: n(n 1)<br />

2<br />

pieces of string<br />

24. 3160 intersections<br />

25. n(n 2)<br />

<br />

2 yields 760 handshakes.<br />

26. 21; 252<br />

27. n(n 3)<br />

2<br />

3 4<br />

H<br />

C<br />

H<br />

H<br />

C<br />

H<br />

H<br />

C<br />

H<br />

80( 79)<br />

; <br />

2<br />

560; there are 35 vertices.<br />

28. M is the midpoint of AY;deductive.<br />

Rectangle 1 2 3 4 5 6 … n … 200<br />

Perimeter of rectangle 10 14 18 22 26 30 … 4n 6 … 806<br />

O<br />

A<br />

T<br />

H<br />

C<br />

H<br />

C<br />

H<br />

H<br />

C<br />

H<br />

3160<br />

Number of squares 6 12 20 30 42 56 … … 40,602<br />

(n 1)(n 2)


LESSON 2.6<br />

1. 63° 2. 90°<br />

3. no 4. 57°<br />

5. yes 6. 113°<br />

7. a d 64°, b c 116°, e g i j <br />

k 108°, f h s 72°, m 105°, n 79°,<br />

p 90°, q 116°, t 119°; Possible explanation:<br />

Using the Vertical Angles Conjecture, n 79°.<br />

Using the Linear Pair Conjecture, p 90°. Using the<br />

Corresponding Angles Conjecture and b 116°,<br />

q 116°.<br />

8. Possible answer: In the<br />

diagram, lines and m are parallel<br />

and intersected by transversal k.<br />

Using the Corresponding Angles<br />

Conjecture,1 2. Using the<br />

Vertical Angles Conjecture,<br />

2 3. Because 1and3arebothcongruent<br />

to 2,theymustbecongruenttoeachother.So<br />

1 3. Therefore,if two parallel lines are cut by a<br />

transversal, then alternate exterior angles are<br />

congruent.<br />

9. 56° 114° 170° 180°. Thus, the lines<br />

marked as parallel cannot really be parallel.<br />

10. Alternate interior angles measure 55°, but<br />

55° 45° 180°.<br />

11. The incoming and<br />

outgoing angles measure<br />

45°. Possible explanation:<br />

Yes, the alternate interior<br />

angles are congruent, and<br />

thus, by the Converse of the<br />

Parallel Lines Conjecture,<br />

the mirrors are parallel.<br />

12. Explanations will vary.<br />

Sample explanation:“I used the<br />

protractor to make corresponding<br />

angles congruent when I<br />

drew line PQ.”<br />

26. (Lesson 2.6)<br />

A<br />

45<br />

90<br />

45<br />

90<br />

45<br />

P<br />

45<br />

1<br />

k<br />

2<br />

3<br />

Q<br />

B<br />

<br />

m<br />

13. No, tomorrow could be a holiday. Converse:“If<br />

tomorrow is a school day, then yesterday was part<br />

of the weekend;” false.<br />

14. 42° 15. 20°<br />

16. No, the lines are not parallel.<br />

17. isosceles triangle<br />

18. a parallelogram that is not also a rectangle or a<br />

rhombus<br />

19. 18 cm 20. 39°<br />

21. 3486<br />

22. 30 squares (one 4-by-4, four 3-by-3, nine<br />

2-by-2, and sixteen 1-by-1)<br />

23. The triangle moved to the left 1 unit.Yes,<br />

congruent to original.<br />

24. The quadrilateral was reflected across both<br />

axes or rotated 180° about the origin.Yes, congruent<br />

to original.<br />

25. The pentagon was reflected across the line<br />

x y. Yes, congruent to original.<br />

L<br />

M'<br />

E<br />

E'<br />

y<br />

4<br />

5<br />

N<br />

M<br />

y<br />

O'<br />

L'<br />

O<br />

6<br />

N'<br />

26. See table below.<br />

Figure number 1 2 3 4 5 6 … n … 35<br />

Number of yellow squares 2 3 4 5 6 7 … n 1 … 36<br />

Number of blue squares 3 5 7 9 11 13 … 2n 1 … 71<br />

Total number of squares 5 8 11 14 17 20 … 3n 2 … 107<br />

y<br />

5<br />

5<br />

4<br />

x<br />

x<br />

x<br />

ANSWERS TO EXERCISES 29<br />

<strong>Answers</strong> to Exercises


<strong>Answers</strong> to Exercises<br />

USING YOUR ALGEBRA SKILLS 2<br />

1. 2<br />

2. 12<br />

<br />

13<br />

97<br />

3. 46<br />

2.1<br />

4. y 23<br />

5. x 4<br />

6. Any point of the form (3p, 4p).Possible answer:<br />

(6,8),(9,12),and (12,16).To find another<br />

point go right 3 and down 4.<br />

30 ANSWERS TO EXERCISES<br />

7. 66.7 mi/h<br />

8. At 6 m/s, Skater 1 is 4 m/s faster than Skater 2 at<br />

2m/s.<br />

9. A 100% grade has a slope of 1. It has an<br />

inclination of 45°, so you probably could not<br />

drive up it. You might be able to walk up it. Grades<br />

higher than 100% are possible, but the angle of<br />

inclination would be greater than 45°.<br />

10. The slope of the adobe house flat roof is<br />

approximately 0. The Connecticut roof is steeper,<br />

with a slope of about 2, and will shed the snow.


CHAPTER 2 REVIEW<br />

1. poor inductive reasoning, but Diana was<br />

probably just being funny<br />

2. <strong>Answers</strong> will vary.<br />

3. <strong>Answers</strong> will vary.<br />

4. 19, 30<br />

5. S, 36<br />

6. 2, 5, 10, 17, 26, 37<br />

7. 1, 2, 4, 8, 16, 32<br />

8. 9.<br />

10. 900<br />

11. 930<br />

12. See table below.<br />

13. See table below.<br />

14. n2 ,302 900<br />

15. n(n<br />

1) 100( 101)<br />

, 2 2<br />

5050<br />

16. n(n 1)<br />

<br />

2<br />

741; therefore, n 39<br />

17. n(n 1)<br />

<br />

2<br />

2926; therefore, n 77<br />

18. n 2, 54 2 52<br />

19a. possible answers: EFC and AFG, AFE<br />

and CFG, FGD and BGH, or BGF and<br />

HGD<br />

19b. possible answers: AFE and EFC, AFG<br />

and CFG, BGF and FGD, or BGH and<br />

HGD (there are four other pairs)<br />

19c. possible answers: EFC and FGD, AFE<br />

and BGF, CFG and DGH, or AFG and<br />

BGH<br />

12. (Chapter 2 Review)<br />

19d. possible answers: AFG and FGD or<br />

CFG and BGF<br />

20. possible answers: GFC by the Vertical<br />

Angles Conjecture, BGF by the Corresponding<br />

Angles Conjecture, and DGH, using the<br />

Alternate Exterior Angles Conjecture<br />

21. True. Converse: “If two polygons have the<br />

same number of sides, then the two polygons are<br />

congruent”; false. Counterexamples may vary;<br />

students might draw a concave quadrilateral and a<br />

convex quadrilateral.<br />

22.<br />

4.5 cm<br />

56<br />

56<br />

124<br />

7 cm<br />

7 cm<br />

56<br />

124 56<br />

4.5 cm<br />

23. PV RX and SU VX ; explanations will vary.<br />

24. The bisected angle measures 50° because of<br />

AIA. So each half measures 25°. The bisector is a<br />

transversal, so the measure of the other acute angle<br />

in the triangle is also 25° by AIA. However, this<br />

angle forms a linearpairwiththeanglemeasuring<br />

165°,and 25° 165° 180°, which contradicts the<br />

Linear Pair Conjecture.<br />

25. a 38°, b 38°, c 142°, d 38°, e 50°,<br />

f 65°, g 106°, h 74°; The angle with measure e<br />

forms a linear pair with an angle with measure 130°<br />

because of the CorrespondingAngles Conjecture.So<br />

e measures 50° because of the Linear Pair Conjecture.<br />

The angle with measure f is half of the angle with<br />

measure 130°, so f65°.The angle with measure g is<br />

congruent to the angle with measure 106° by the<br />

CorrespondingAngles Conjecture,so g106°.<br />

n 1 2 3 4 5 6 … n … 20<br />

f(n) 2 1 4 7 10 13 … … 55<br />

13. (Chapter 2 Review)<br />

3n 5<br />

n 1 2 3 4 5 6 … n … 20<br />

f(n) 1 3 6 10 15 21 … … 210<br />

n(n 1)<br />

<br />

2<br />

ANSWERS TO EXERCISES 31<br />

<strong>Answers</strong> to Exercises


<strong>Answers</strong> to Exercises<br />

<strong>Answers</strong> to Exercises<br />

CHAPTER 3 • CHAPTER 3<br />

CHAPTER 3 • CHAPTER<br />

1.<br />

2.<br />

3.<br />

4.<br />

5. possible answer:<br />

LESSON 3.1<br />

6. m3 m1 m2; possible answer:<br />

7. possible answer:<br />

A B<br />

8.<br />

A<br />

C<br />

E<br />

L<br />

G<br />

1 2 3<br />

C<br />

AB<br />

AB<br />

AB CD<br />

AB 2EF CD<br />

copy<br />

AC BC<br />

AB<br />

32 ANSWERS TO EXERCISES<br />

F<br />

D<br />

E<br />

B<br />

EF<br />

CD<br />

CD<br />

EF<br />

2<br />

1<br />

9. For Exercise 7, trace the triangle. For Exercise 8,<br />

trace the segment onto three separate pieces of<br />

patty paper. Lay them on top of each other, and<br />

slide them around until the segments join at the<br />

endpoints and form a triangle.<br />

10. One method: Draw DU . Copy Q and<br />

construct COY QUD.Duplicate DUA<br />

at point O.Construct OYP UDA.<br />

Q<br />

U<br />

11. One construction method is to create congruent<br />

circles that pass through each other’s center. One<br />

side of the triangle is between the centers of the<br />

circles; the other sides meet where the circles<br />

intersect.<br />

12. a 50°, b 130°, c 50°, d 130°, e 50°,<br />

f 50°, g 130°, h 130°, k 155°, l 90°,<br />

m 115°, n 65°<br />

13. west<br />

14. An isosceles triangle is a triangle that has at<br />

least one line of reflectional symmetry.Yes, all<br />

equilateral triangles are isosceles.<br />

15.<br />

16. new coordinates: A(0, 0), Y(4, 0), D(0, 2)<br />

17. Methods will vary. It isn’t possible to draw a<br />

second triangle with the same side lengths that is<br />

not congruent to the first.<br />

11 cm 8 cm<br />

10 cm<br />

6<br />

Y<br />

y<br />

Y'<br />

–6 D A' A 6<br />

–6<br />

A<br />

D<br />

D'<br />

C<br />

O<br />

x<br />

P<br />

Y


1.<br />

2.<br />

3.<br />

4.<br />

5.<br />

A B<br />

Q D<br />

C D<br />

1<br />

CD<br />

2<br />

Edge of the paper<br />

Original segment<br />

1<br />

CD<br />

2<br />

2AB –<br />

1<br />

CD<br />

2<br />

M N<br />

AB CD<br />

LESSON 3.2<br />

AB AB<br />

1<br />

CD<br />

2<br />

MN =<br />

1<br />

(AB + CD)<br />

2<br />

6. Exercises 1–5 with patty paper:<br />

Exercise 1 This is the same as Investigation 1.<br />

Exercise 2<br />

Step 1 Draw a segment on patty paper. Label it QD .<br />

Step 2 Fold your patty paper so that endpoints Q<br />

and D coincide. Crease along the fold.<br />

Step 3 Unfold and draw a line in the crease.<br />

Step 4 Label the point of intersection A.<br />

Step 5 Fold your patty paper so that endpoints Q<br />

and A coincide. Crease along the fold.<br />

Step 6 Unfold and draw a line in the crease.<br />

Step 7 Label the point of intersection B.<br />

Step 8 Fold your patty paper so that endpoints A<br />

and D coincide. Crease along the fold.<br />

Step 9 Unfold and draw a line in the crease.<br />

Step 10 Label the point of intersection C.<br />

Exercise 3 This is the same as Investigation 1.<br />

Exercise 4<br />

Step 1 Do Investigation 1 to get 1<br />

2 CD.<br />

Step 2 On a second piece of patty paper, trace AB <br />

two times so that the two segments form a segment<br />

of length 2AB.<br />

Step 3 Lay the first piece of patty paper on top of<br />

the second so that the endpoints coincide and the<br />

shorter segment is on top of the longer segment.<br />

Step 4 Trace the rest of the longer segment with a<br />

different colored writing utensil. That will be the<br />

answer.<br />

Exercise 5<br />

Step 1 Trace segments AB and CD so that the two<br />

segments form a segment of length AB CD.<br />

Step 2 Fold your patty paper so that points A and<br />

D coincide. Crease along the fold.<br />

Step 3 Unfold and draw a line in the crease.<br />

7. The perpendicular bisectors all intersect in one<br />

point.<br />

8. The medians all intersect in one point.<br />

C<br />

A<br />

M<br />

I<br />

9. GH appears to be parallel to EF, and its length<br />

is half the length of EF.<br />

G<br />

B<br />

N<br />

D H<br />

E<br />

L<br />

F<br />

L<br />

A<br />

ANSWERS TO EXERCISES 33<br />

<strong>Answers</strong> to Exercises


<strong>Answers</strong> to Exercises<br />

10. The quadrilateral appears to be a rhombus.<br />

11.<br />

Any point on the perpendicular bisector of the<br />

segment connecting the two offices would be<br />

equidistant from the two post offices. Therefore,<br />

any point on one side of the perpendicular bisector<br />

would be closer to the post office on that side.<br />

12. It is a parallelogram.<br />

T<br />

E<br />

R<br />

D<br />

I<br />

V<br />

Ness Station<br />

F L<br />

A<br />

O<br />

C<br />

S<br />

Umsar<br />

Station<br />

13. The triangles are not necessarily congruent,<br />

but their areas are equal. A cardboard triangle<br />

would balance on its median.<br />

34 ANSWERS TO EXERCISES<br />

14. One way to balance it is along the median. The<br />

two halves weigh the same.<br />

sample figure:<br />

C<br />

D<br />

A<br />

Area CDB = 158 in 2<br />

Area DAB = 158 in 2<br />

15. F 16. E<br />

17. B 18. A<br />

19. D 20. C<br />

21. B, C, D, E, H, I, O, X (K in some fonts, though<br />

not this one)<br />

22. Methods will vary.<br />

10 cm<br />

40 70<br />

A B<br />

C<br />

B<br />

Ruler<br />

It is not possible to draw a second triangle with the<br />

same angle and side measures that is not congruent<br />

to the first.


1.<br />

LESSON 3.3<br />

The answer depends on the angle drawn and where<br />

P is placed.<br />

2. C<br />

3.<br />

Two altitudes fall outside the triangle, and one falls<br />

inside.<br />

4. From the point, swing arcs on the line to<br />

construct a segment whose midpoint is that point.<br />

Then construct the perpendicular bisector of the<br />

segment.<br />

5. Construct perpendiculars from point Q and<br />

point R.Mark offQS and RE congruent to QR .<br />

Connect points S and E.<br />

Q<br />

S<br />

A<br />

B<br />

U<br />

B<br />

D<br />

I<br />

O T<br />

R<br />

E<br />

P<br />

B<br />

G<br />

6. The two folds are parallel.<br />

7. Fold the patty paper through the point so that<br />

two perpendiculars coincide to see the side closest<br />

to the point. Fold again using the perpendicular<br />

of the side closest to the point and the third<br />

perpendicular; compare those sides.<br />

8. Draw a line. Mark two points on it, and label<br />

them A and C. Construct a perpendicular at C.<br />

Mark off CB congruent to CA. The altitude CD is<br />

also the angle bisector, median, and perpendicular<br />

bisector.<br />

A<br />

C<br />

9.<br />

10.<br />

11.<br />

12.<br />

Q<br />

T<br />

P<br />

D<br />

B<br />

O B<br />

U<br />

E L<br />

A B<br />

Complement<br />

of A<br />

A<br />

ANSWERS TO EXERCISES 35<br />

<strong>Answers</strong> to Exercises


<strong>Answers</strong> to Exercises<br />

13. See table below.<br />

14.<br />

15.<br />

16.<br />

17.<br />

F<br />

A<br />

C D<br />

B<br />

Y<br />

I T<br />

E<br />

F<br />

13. (Lesson 3.3)<br />

36 ANSWERS TO EXERCISES<br />

18. not congruent<br />

19. possible answer:<br />

20. possible answer:<br />

5 cm<br />

6 cm<br />

40<br />

5 cm<br />

8 cm<br />

9 cm<br />

9 cm<br />

Rectangular pattern with triangles<br />

Rectangle 1 2 3 4 5 6 … n … 35<br />

Number of shaded<br />

triangles<br />

2 9 20 35 54 77 … … 2484<br />

40<br />

8 cm<br />

6 cm<br />

(2n 1)(n 1)


1. D<br />

2. F<br />

3. A<br />

4. C<br />

5. E<br />

6.<br />

7.<br />

8.<br />

9a, b.<br />

9c.<br />

10.<br />

R<br />

M<br />

M<br />

z<br />

M<br />

Altitude<br />

M O S<br />

135<br />

z<br />

z<br />

E<br />

45<br />

90 45<br />

45<br />

E<br />

S<br />

LESSON 3.4<br />

A<br />

P<br />

R A<br />

R P<br />

A<br />

U<br />

B<br />

M<br />

Angle<br />

bisector<br />

Median<br />

P<br />

11.<br />

12. The angle bisectors are perpendicular. The<br />

sum of the measures of the linear pair is 180°. The<br />

sum of half of each must be 90°.<br />

13. If a point is equally distant from the sides of<br />

an angle, then it is on the bisector of an angle. This<br />

is true for points in the same plane as the angle.<br />

Mosaic answers: Square pattern constructions:<br />

perpendiculars, equal segments, and midpoints;<br />

The triangles are not identical, as the downward<br />

ones have longer bases.<br />

14. y 110°<br />

15. mR 46°<br />

16. Angle A is the largest; mA 66°,<br />

mB 64°, mC 50°.<br />

17. STOP<br />

18.<br />

A<br />

N<br />

L<br />

G<br />

T<br />

O<br />

I<br />

S<br />

ANSWERS TO EXERCISES 37<br />

<strong>Answers</strong> to Exercises


<strong>Answers</strong> to Exercises<br />

19.<br />

20.<br />

B<br />

5.6 cm<br />

A<br />

A<br />

130<br />

3.5 cm C<br />

B<br />

6.5 cm<br />

C<br />

21. No, the triangles don’t look the same.<br />

40 60 40<br />

8 cm<br />

60<br />

8 cm<br />

38 ANSWERS TO EXERCISES<br />

22a. A web of lines fills most of the plane, except<br />

a U-shaped region and a V-shaped region. (The<br />

U-shaped region is actually bounded by a section<br />

of a parabola and straight lines. If AB were<br />

extended to AB , the U would be a complete<br />

parabola.)<br />

B<br />

C<br />

D<br />

Parabola<br />

A<br />

22b. a line segment parallel to AB and half the<br />

length (The segment is actually the midsegment<br />

of ABD.)


LESSON 3.5<br />

1. 2.<br />

3. Construct a segment with length z. Bisect the<br />

z<br />

segment to get length . 2 Bisect again to get a<br />

z . Construct a square with<br />

segment with length <br />

4<br />

z<br />

each side of length . 4<br />

4.<br />

5. sample construction:<br />

T<br />

6. Draw a line and construct ML perpendicular to<br />

it. Swing an arc from point M to point G so that<br />

MG RA. From point G, swing an arc to construct<br />

RG. Finish the parallelogram by swinging an arc of<br />

length RA from R and swinging an arc of length GR<br />

from M. There is only one possible parallelogram.<br />

G<br />

A<br />

1 z<br />

4<br />

P<br />

A<br />

1 z<br />

2<br />

L<br />

M A<br />

7. 1 S, 2 U<br />

x<br />

x<br />

A<br />

R<br />

R<br />

z<br />

x<br />

x<br />

x<br />

8. 1 S and 2 U by of the Alternate<br />

Interior Angles Conjecture<br />

9. The ratios appear to be the same.<br />

10. 1 3 and 2 4 by the<br />

Corresponding Angles Conjecture<br />

11. A parallelogram<br />

12. Using the Converse of the Parallel Lines<br />

Conjecture, the angle bisectors are parallel:<br />

DAB ABC, so AD BC .<br />

13. Construct the perpendicular bisector of each<br />

of the three segments connecting the fire stations.<br />

Eliminate the rays beyond where the bisectors<br />

intersect. A point within any region will be closest<br />

to the fire station in that region.<br />

14. Z O<br />

15. B<br />

M<br />

16.<br />

D<br />

R<br />

D<br />

B<br />

A C<br />

T<br />

R E<br />

W<br />

K C<br />

RC = KE = 8 cm<br />

I<br />

60<br />

R O<br />

17. a 72°, b 108°, c 108°, d 108°, e 72°,<br />

f 108°, g 108°, h 72°, j 90°, k 18°, l 90°,<br />

m 54°, n 62°, p 62°, q 59°, r 118°;<br />

Explanations will vary. Sample explanation:<br />

c is 108° because of the Corresponding Angles<br />

Conjecture. Using the Vertical Angles Conjecture,<br />

2m 108°, so m 54°. p n because of the<br />

Corresponding Angles Conjecture. Using the<br />

Linear Pair Conjecture, n 62°, so p 62°.<br />

Using the Linear Pair Conjecture, r p 180°.<br />

Because p 62°, r 118°.<br />

ANSWERS TO EXERCISES 39<br />

<strong>Answers</strong> to Exercises


<strong>Answers</strong> to Exercises<br />

USING YOUR ALGEBRA SKILLS 3<br />

1. perpendicular<br />

2. neither<br />

3. perpendicular<br />

4. parallel<br />

5. possible answer: (2, 5) and (7, 11)<br />

6. possible answer: (1, 5) and (2, 12)<br />

7. Ordinary; no two slopes are the same, so no<br />

sides are parallel, although TE EM because their<br />

slopes are opposite reciprocals.<br />

8. Ordinary, for the same reason as in Exercise 7—<br />

none of the sides are quite parallel.<br />

9. trapezoid: KC RO<br />

10a. Slope HA slope ND 1<br />

6 ;<br />

slope HD slope NA 6. Quadrilateral HAND<br />

is a rectangle because opposite sides are parallel<br />

and adjacent sides are perpendicular.<br />

40 ANSWERS TO EXERCISES<br />

10b. Midpoint HN midpoint AD 1<br />

diagonals of a rectangle bisect each other.<br />

11a. Yes, the diagonals are perpendicular.<br />

Slope OE 1; slope VR 1.<br />

11b. Midpoint VR midpoint OE (2, 4).<br />

The diagonals of OVER bisect each other.<br />

11c. OVER appears to be a rhombus. Slope<br />

2 ,3.The<br />

OV slope RE 1<br />

5 and slope OR slope<br />

VE 5, so opposite sides are parallel. Also, all of<br />

the sides appear to have the same length.<br />

12a. Both slopes equal 1<br />

2 .<br />

12b. The segments are not parallel because they<br />

are coincident.<br />

12c. distinct<br />

13. (3, 6)


LESSON 3.6<br />

1. Sample description: Construct one of the<br />

segments, and mark arcs of the correct length from<br />

the endpoints. Draw sides to where those arcs meet.<br />

M<br />

A<br />

M<br />

2.<br />

O<br />

Sample description: Construct O. Mark off<br />

distances OD and OT on the sides of the angle.<br />

Connect D and T.<br />

3.<br />

I<br />

I<br />

D<br />

O<br />

M A<br />

O D<br />

O<br />

Y<br />

I Y<br />

A<br />

Y<br />

S<br />

T<br />

T<br />

Sample description: Construct IY. Construct I at<br />

I and Y at Y. Label the intersection of the rays<br />

point G.<br />

4. Yes, students’ constructions must be either<br />

larger or smaller than the triangle in the book.<br />

Sample description: Draw one side with a different<br />

length than the lengths in the book. Duplicate an<br />

S<br />

S<br />

G<br />

angle at each end of that segment congruent to one<br />

of the angles in the book. Where they meet is the<br />

third vertex of the triangle.<br />

5.<br />

A<br />

A<br />

C<br />

Sample description: Construct A and mark off<br />

the distance AB. From B swing an arc of length BC<br />

to intersect the other side of A at two points.<br />

Each gives a different triangle.<br />

6.<br />

C<br />

y x<br />

____<br />

2<br />

A<br />

y<br />

x<br />

C<br />

T<br />

B<br />

B<br />

y x<br />

____<br />

2<br />

x<br />

Sample description: Mark the distance y, mark<br />

back the distance x, and bisect the remaining<br />

length of y x. Using an arc of that length, mark<br />

arcs on the ends of segment x. The point where<br />

they intersect is the vertex angle of the triangle.<br />

7.<br />

Sample description: Draw an angle. Mark off equal<br />

segments on the sides of the angle. Use a different<br />

compass setting to draw intersecting arcs from the<br />

ends of those segments.<br />

8. Sample description: Draw an angle and mark<br />

off unequal distances on the sides. At the endpoint<br />

of the longer segment (not the angle vertex), swing<br />

an arc with the length of the shorter segment. From<br />

the endpoint of the shorter segment, swing an arc<br />

the length of the longer segment. Connect the<br />

endpoints of the segments to the intersection<br />

points of the arcs to form a quadrilateral.<br />

ANSWERS TO EXERCISES 41<br />

<strong>Answers</strong> to Exercises


<strong>Answers</strong> to Exercises<br />

9.<br />

Sample description: Draw a segment and draw an<br />

angle at one end of the segment. Mark off a<br />

distance equal to that segment on the other side of<br />

the angle. Draw an angle at that point and mark off<br />

the same distance. Connect that point to the other<br />

end of the original segment.<br />

10.<br />

Sample description: Draw an angle and mark off<br />

equal lengths on the two sides. Use that length to<br />

determine another point that distance from the<br />

points on the sides. Connect that point with the<br />

two points on the side of the angle.<br />

11. <strong>Answers</strong> will vary. The angle bisector lies<br />

between the median and the altitude. The order of<br />

the points is either M, R, S or S, R, M. One possible<br />

conjecture: In a scalene obtuse triangle the angle<br />

bisector is always between the median and the<br />

altitude.<br />

mABC = 111<br />

B<br />

R Median<br />

A S M<br />

Altitude Angle<br />

bisector<br />

42 ANSWERS TO EXERCISES<br />

C<br />

12. new coordinates: E(4, 6), A(7, 0), T(1, 2)<br />

13.<br />

–5<br />

14. half a cylinder<br />

15. 503<br />

16.<br />

R<br />

–5<br />

–5<br />

y<br />

T'<br />

T<br />

E<br />

E'<br />

110<br />

E<br />

3.2 cm<br />

110 110<br />

A 5.5 cm C<br />

A'<br />

A<br />

Reflectional Rotational<br />

Figure symmetries symmetries<br />

Trapezoid 0 0<br />

Kite 1 0<br />

Parallelogram 0 2<br />

Rhombus 2 2<br />

Rectangle 2 2<br />

x


1. incenter<br />

LESSON 3.7<br />

Because the station needs to be equidistant from<br />

the paths, it will need to be on each of the angle<br />

bisectors.<br />

2. circumcenter<br />

3. incenter<br />

The center of the circular sink must be equidistant<br />

from the three counter edges, that is, the incenter of<br />

the triangle.<br />

4. circumcenter<br />

To find the point equidistant from three points,<br />

find the circumcenter of the triangle with those<br />

points as vertices.<br />

5. Circumcenter. Find the perpendicular bisectors<br />

of two of the sides of the triangle formed by the<br />

classes. Locate the pie table where these two lines<br />

intersect.<br />

6.<br />

7.<br />

Stove<br />

Fridge Sink<br />

8. Yes, any circle with a larger radius would not<br />

fit within the triangle. To get a circle with a larger<br />

radius tangent to two of the sides would force the<br />

circle to pass through the third side twice.<br />

9. No, on an obtuse triangle the circle with the<br />

largest side of the triangle as the diameter of the<br />

circle creates the smallest circular region that<br />

contains the triangle. The circumscribed circle of<br />

an acute triangle does create the smallest circular<br />

region that contains the triangle.<br />

10. For an acute triangle, the circumcenter is<br />

inside the triangle; for an obtuse triangle, the<br />

circumcenter is outside the triangle. The<br />

circumcenter of a right triangle lies on the<br />

midpoint of the hypotenuse.<br />

11. For an acute triangle, the orthocenter is inside<br />

the triangle; for an obtuse triangle, the orthocenter<br />

is outside the triangle. The orthocenter of a right<br />

triangle lies on the vertex of the right angle.<br />

12. The midsegment appears parallel to side MA <br />

and half the length.<br />

13. The base angles of the isosceles trapezoid<br />

appear congruent.<br />

T<br />

A O<br />

M<br />

A<br />

M H T<br />

S<br />

ANSWERS TO EXERCISES 43<br />

<strong>Answers</strong> to Exercises


<strong>Answers</strong> to Exercises<br />

14. The measure of A is 90°. The angle inscribed<br />

in a semicircle appears to be a right angle.<br />

15. The two diagonals appear to be perpendicular<br />

bisectors of each other.<br />

T<br />

M<br />

16.<br />

17.<br />

A<br />

9<br />

y<br />

Construct the incenter by bisecting the two angles<br />

shown. Any other point on the angle bisector of the<br />

third angle must be equidistant from the two<br />

unfinished sides. From the incenter, make congruent<br />

arcs that intersect the unfinished sides. The<br />

intersection points are equidistant from the incenter.<br />

Use two congruent arcs to find another point that<br />

is equidistant from the two points you just<br />

constructed. The line that connects this point and<br />

the incenter is the angle bisector of the third angle.<br />

18. <strong>Answers</strong> should describe the process of<br />

discovering that the midpoints of the altitudes are<br />

collinear for an isosceles right triangle.<br />

19. a triangle<br />

20.<br />

M<br />

6.0 cm 6.0 cm<br />

60 6.0 cm 60<br />

R<br />

O<br />

60 60<br />

6.0 cm<br />

A<br />

x + y = 9<br />

H<br />

9<br />

T<br />

6.0 cm<br />

44 ANSWERS TO EXERCISES<br />

x<br />

21.<br />

K<br />

4.8 cm<br />

Y<br />

40 40<br />

E<br />

6.4 cm<br />

22. construction of an angle bisector<br />

23. construction of a perpendicular line through a<br />

point on a line<br />

24. construction of a line parallel to a given line<br />

through a point not on the line<br />

25. construction of an equilateral triangle<br />

26. construction of a perpendicular bisector<br />

T


LESSON 3.8<br />

1. The center of gravity is the centroid. She needs<br />

to locate the incenter to create the largest circle<br />

within the triangle.<br />

2. AM 20; SM 7; TM 14; UM 8<br />

3. BG 24; IG 12 4. RH 42; TE 45<br />

5. The points of concurrency are the same point<br />

for equilateral triangles because the segments are<br />

the same.<br />

6.<br />

7. ortho-/in-/centroid/circum-; the order changes<br />

when the angle becomes larger than 60°. The<br />

points become one when the triangle is equilateral.<br />

8. Start by constructing a quadrilateral, then make<br />

a copy of it. Draw a diagonal in one, and draw a<br />

different diagonal in the second. Find the centroid<br />

of each of the four triangles. Construct a segment<br />

connecting the two centroids in each quadrilateral.<br />

Place the two quadrilaterals on top of each other<br />

matching the congruent segments and angles.Where<br />

the two segments connecting centroids intersect is<br />

the centroid of the quadrilateral.<br />

A<br />

D<br />

Circumcenter<br />

M 1<br />

Centroid<br />

Incenter<br />

Orthocenter<br />

Orthocenter<br />

Incenter<br />

Circumcenter<br />

M 2<br />

C<br />

Centroid<br />

B<br />

D<br />

M4 A<br />

M 3<br />

C<br />

B<br />

9. circumcenter<br />

10. The shortest chord through P is a segment<br />

perpendicular to the diameter through P, which is<br />

the longest chord through P.<br />

11.<br />

12. rule: 2n 2, possible answer:<br />

13.<br />

O<br />

P<br />

H H H<br />

H C C C C<br />

H H H<br />

A<br />

C<br />

14. a 128°, b 52°, c 128°, d 128°,<br />

e 52°, f 128°, g 52°, h 38°,<br />

k 52°, m 38°, n 71°, p 38°<br />

15. Construct altitudes from the two accessible<br />

vertices to construct the orthocenter. Through the<br />

orthocenter, construct a line perpendicular to the<br />

southern boundary of the property. This method<br />

will divide the property equally only if the southern<br />

boundary is the base of an isosceles triangle.<br />

Altitude to<br />

missing vertex<br />

16. 1580 greetings<br />

C<br />

H H H<br />

C<br />

C<br />

B'<br />

B<br />

C<br />

H H H<br />

H<br />

ANSWERS TO EXERCISES 45<br />

<strong>Answers</strong> to Exercises


<strong>Answers</strong> to Exercises<br />

CHAPTER 3 REVIEW<br />

1. False; a geometric construction uses a straightedge<br />

and a compass.<br />

2. False; a diagonal connects two non-consecutive<br />

vertices.<br />

3. true 4. true<br />

5. false<br />

6. False; the lines can’t be a given distance from a<br />

segment because the segment has finite length and<br />

the lines are infinite.<br />

7. false<br />

D<br />

A B<br />

8. true 9. true<br />

10. False; the orthocenter does not always lie<br />

inside the triangle.<br />

11. A 12. B or K 13. I 14. H<br />

15. G 16. D 17. J 18. C<br />

19. 20.<br />

21. 22.<br />

23. Construct a 90° angle and bisect it twice.<br />

24.<br />

25. incenter<br />

26. Dakota Davis should locate the circumcenter<br />

of the triangular region formed by the three stones,<br />

which is the location equidistant from the stones.<br />

27.<br />

B<br />

A<br />

B<br />

A C<br />

C<br />

z<br />

Copy<br />

46 ANSWERS TO EXERCISES<br />

C<br />

28.<br />

29.<br />

30. mA mD.You must first find B.<br />

mB 180° 2(mA).<br />

B<br />

2y<br />

31.<br />

32.<br />

A<br />

A B<br />

I<br />

y<br />

R<br />

1_<br />

2 z<br />

y<br />

1_<br />

2 z<br />

y y x<br />

Segment<br />

5x<br />

P R<br />

3x 4x<br />

x<br />

2y<br />

Q<br />

B A<br />

D F<br />

T<br />

D<br />

33. rotational symmetry<br />

34. neither 35. both<br />

36. reflectional symmetry<br />

37. D 38. A 39. C 40. B<br />

41. False; an isosceles triangle has two congruent<br />

sides.<br />

D<br />

4x<br />

y


42. true<br />

43. False; any non-acute triangle is a<br />

counterexample.<br />

44. False; possible explanation: The orthocenter is<br />

the point of intersection of the three altitudes.<br />

45. true<br />

46. False; any linear pair of angles is a<br />

counterexample.<br />

47. False; each side is adjacent to one congruent<br />

side and one noncongruent side, so two consecutive<br />

sides may not be congruent.<br />

48. false;<br />

49. False; the measure of an arc is equal to the<br />

measure of its central angle.<br />

50. false; TD 2DR<br />

51. False; a radius is not a chord.<br />

52. true<br />

53. False; inductive reasoning is the process of<br />

observing data, recognizing patterns, and making<br />

generalizations about those patterns.<br />

54. paradox<br />

55a. 2 and 6 or 3 and 5<br />

55b. 1 and 5<br />

55c. 138°<br />

56. 55<br />

57. possible answer:<br />

58a. yes<br />

58b. If the month has 31 days, then the month is<br />

October.<br />

60. (Chapter 3 Review)<br />

58c. no<br />

59.<br />

60. See table below.<br />

61. See table below.<br />

62. a 38°, b 38°, c 142°, d 38°, e 50°,<br />

f 65°, g 106°, h 74°.<br />

Possible explanation: The angle with measure c is<br />

congruent to an angle with measure 142° because<br />

of the Corresponding Angles Conjecture, so<br />

c 142°. The angle with measure 130° is<br />

congruent to the bisected angle by the<br />

Corresponding Angles Conjecture. The angle with<br />

measure f has half the measure of the bisected<br />

angle, so f 65°.<br />

63. Triangles will vary. Check that the triangle is<br />

scalene and that at least two angle bisectors have<br />

been constructed.<br />

64. mFAD 30° so mADC 30°, but<br />

its vertical angle has measure 26°. This is a<br />

contradiction.<br />

65. minimum: 101 regions by 100 parallel lines;<br />

maximum: 5051 regions by 100 intersecting,<br />

noncurrent lines<br />

n 1 2 3 4 5 6 … n … 20<br />

f(n) 1 2 5 8 11 14 … … 56<br />

61. (Chapter 3 Review)<br />

f(n) 3n 4<br />

n 1 2 3 4 5 6 … n … 20<br />

f(n) 0 3 8 15 24 35 … … 399<br />

f(n) n 2 1<br />

Q<br />

Q'<br />

ANSWERS TO EXERCISES 47<br />

<strong>Answers</strong> to Exercises


<strong>Answers</strong> to Exercises<br />

<strong>Answers</strong> to Exercises<br />

CHAPTER 4 • CHAPTER CHAPTER 4 • CHAPTER<br />

LESSON 4.1<br />

1. The angle measures change, but the sum<br />

remains 180°.<br />

2. 73°<br />

3. 60°<br />

4. 110°<br />

5. 24°<br />

6. 3 360° 180° 900°<br />

7. 3 180° 180° 360°<br />

8. 69°; 47°; 116°; 93°; 86°<br />

9. 30°; 50°; 82°; 28°; 32°; 78°; 118°; 50°<br />

10.<br />

11.<br />

12. First construct E, using the method used in<br />

Exercise 10.<br />

A<br />

13.<br />

E<br />

M<br />

R<br />

A<br />

R<br />

M A<br />

G<br />

L<br />

14. From the Triangle Sum Conjecture<br />

mA mS mM 180°. Because M is a<br />

right angle, mM 90°. By substitution,<br />

mA mS 90° 180°. By subtraction,<br />

mA mS 90°. So two wrongs make a right!<br />

15. <strong>Answers</strong> will vary. See the proof on page 202.<br />

To prove the Triangle Sum Conjecture, the Linear<br />

Pair Conjecture and the Alternate Interior Angles<br />

Conjecture must be accepted as true.<br />

16. It is easier to draw PDQ if the Triangle Sum<br />

Conjecture is used to find that the measure of<br />

D is 85°. Then PD can be drawn to be 7 cm, and<br />

48 ANSWERS TO EXERCISES<br />

R<br />

Fold<br />

4<br />

G<br />

L<br />

A<br />

E<br />

R<br />

angles P and D can be drawn at each endpoint<br />

using the protractor.<br />

40<br />

85<br />

P 7 cm D<br />

17. The third angles of the triangles also have the<br />

same measures; are equal in measure<br />

18. You know from the Triangle Sum Conjecture<br />

that mA mB mC 180°, and mD <br />

mE mF 180°. By the transitive property,<br />

mA mB mC mD mE mF.<br />

You also know that mA mD, and mB <br />

mE. You can substitute for mD and mE in the<br />

longer equation to get mA mB mC <br />

mA mB mF. Subtracting equal terms<br />

from both sides, you are left with mC mF.<br />

19. For any triangle, the sum of the angle measures<br />

is 180°, by the Triangle Sum Conjecture. Since the<br />

triangle is equiangular, each angle has the same<br />

measure, say x. So x x x 180°, and x 60°.<br />

20. false<br />

21. false<br />

22. false<br />

23. false<br />

55<br />

24. true<br />

25. eight; 100<br />

Q


LESSON 4.2<br />

1. 79°<br />

2. 54°<br />

3. 107.5°<br />

4. 44°; 35 cm<br />

5. 76°; 3.5 cm<br />

6. 72°; 36°; 8.6 cm<br />

7. 78°; 93 cm<br />

8. 75 m; 81°<br />

9. 160 in.; 126°<br />

10. a 124°, b 56°, c 56°, d 38°, e 38°,<br />

f 76°, g 66°, h 104°, k 76°, n 86°,<br />

p 38°; Possible explanation: The angles with<br />

measures 66° and d form a triangle with the angle<br />

with measure e and its adjacent angle. Because d,<br />

e, and the adjacent angle are all congruent,<br />

3d 66° 180°. Solve to get d 38°. This is<br />

also the measure of one of the base angles of the<br />

isosceles triangle with vertex angle measure h.<br />

Using the Isosceles Triangle Conjecture, the other<br />

base angle measures d, so 2d h 180°, or<br />

76° h 180°. Therefore, h 104°.<br />

11. a 36°, b 36°, c 72°, d 108°, e 36°;<br />

none<br />

12a. Yes. Two sides are radii of a circle. Radii must<br />

be congruent; therefore, each triangle must be<br />

isosceles.<br />

12b. 60°<br />

13. NCA<br />

14. IEC<br />

15.<br />

16.<br />

17. possible answer:<br />

Fold 2<br />

D<br />

A<br />

M N<br />

G<br />

Fold 1<br />

K<br />

105 60<br />

45<br />

C<br />

Fold 3<br />

Fold 4<br />

18. perpendicular<br />

19. parallel<br />

20. parallel<br />

21. neither<br />

22. parallelogram<br />

23. 40<br />

0 8 16 24 32 40<br />

P<br />

F<br />

B<br />

H<br />

E<br />

24. New: (6, 3), (2, 5), (3, 0). Triangles are<br />

congruent.<br />

25. New: (3, 3), (3, 1), (1, 5). Triangles<br />

are congruent.<br />

ANSWERS TO EXERCISES 49<br />

<strong>Answers</strong> to Exercises


<strong>Answers</strong> to Exercises<br />

USING YOUR ALGEBRA SKILLS 4<br />

1. false 2. true<br />

3. not a solution 4. solution<br />

5. not a solution 6. x 7<br />

7. y 4 8. x 8<br />

9. x 4.2 10. n 1<br />

2 <br />

11. x 2 12. t 18<br />

13. n 2<br />

5 14a. x 9<br />

<br />

4<br />

9<br />

14b. x ; 4 The two methods produce identical<br />

results. Multiplying by the lowest common<br />

denominator (which is comprised of the factors of<br />

both denominators) and then reducing common<br />

factors (which clears the denominators on either<br />

side) is the same as simply multiplying each numerator<br />

by the opposite denominator (or cross multiplying).<br />

Algebraically you could show that the two methods<br />

are equivalent as follows:<br />

a c<br />

b<br />

d<br />

bda b bd c d<br />

abd<br />

<br />

b<br />

bcd<br />

<br />

d<br />

ad bc<br />

The method of “clearing fractions” results in the<br />

method of “cross multiplying.”<br />

50 ANSWERS TO EXERCISES<br />

15. You get an equation that is always false, so<br />

there is no solution to the equation.<br />

16. Camella is not correct. Because the equation<br />

0 0 is always true, the truth of the equation does<br />

not depend on the value of x. Therefore, x can be<br />

any real number. Camella’s answer, x 0, is only<br />

one of infinitely many solutions.<br />

17.<br />

x<br />

2x<br />

2x<br />

If x equals the measure of the vertex angle, then<br />

the base angles each measure 2x. Applying the<br />

Triangle Sum Conjecture results in the following<br />

equation:<br />

x 2x 2x 180°<br />

5x 180°<br />

x 36°<br />

The measure of the vertex angle is 36° and the<br />

measure of each base angle is 72°.


1. yes<br />

2. no<br />

3. no<br />

4 5<br />

9<br />

5 6<br />

12<br />

LESSON 4.3<br />

4. yes<br />

5. a, b, c<br />

6. c, b, a<br />

7. b, a, c<br />

8. a, c, b<br />

9. a, b, c<br />

10. v, z, y, w, x<br />

11. 6 length 102<br />

12. By the Triangle Inequality Conjecture, the<br />

sum of 11 cm and 25 cm should be greater than<br />

48 cm.<br />

13. b 55°, but 55° 130° 180°, which is<br />

impossible by the Triangle Sum Conjecture.<br />

14. 135°<br />

15. 72°<br />

16. 72°<br />

17. a b c 180° and x c 180°. Subtract c<br />

from both sides of both equations to get x 180 c<br />

and a b 180 c. Substitute a b for 180 c<br />

in the first equation to get x a b.<br />

18. 45°<br />

19. a 52°, b 38°, c 110°, d 35°<br />

20. a 90°, b 68°, c 112°, d 112°, e 68°,<br />

f 56°, g 124°, h 124°<br />

21. By the Triangle Sum Conjecture, the third<br />

angle must measure 36° in the small triangle, but it<br />

measures 32° in the large triangle. These are the<br />

same angle, so they can’t have different measures.<br />

22. ABE<br />

23. FNK<br />

24. cannot be determined<br />

ANSWERS TO EXERCISES 51<br />

<strong>Answers</strong> to Exercises


<strong>Answers</strong> to Exercises<br />

LESSON 4.4<br />

1. <strong>Answers</strong> will vary. Possible answer: If three<br />

sides of one triangle are congruent to three sides of<br />

another triangle, then the triangles are congruent<br />

(all corresponding angles are also congruent).<br />

2. <strong>Answers</strong> will vary. Possible answer: The picture<br />

statement means that if two sides of one triangle<br />

are congruent to two sides of another triangle, and<br />

the angles between those sides are also congruent,<br />

then the two triangles are congruent.<br />

If you know this: then you also know this:<br />

3. <strong>Answers</strong> will vary. Possible answer:<br />

4. SAS<br />

5. SSS<br />

6. cannot be determined<br />

7. SSS<br />

8. SAS<br />

9. SSS (and the Converse of the Isosceles Triangle<br />

Conjecture)<br />

10. yes, ABC ADE by SAS<br />

11. Possible answer: Boards nailed diagonally in<br />

the corners of the gate form triangles in those<br />

corners. Triangles are rigid, so the triangles in the<br />

gate’s corners will increase the stability of those<br />

corners and keep them from changing shape.<br />

12. FLE by SSS<br />

52 ANSWERS TO EXERCISES<br />

25. (Lesson 4.4)<br />

Side length 1 2 3 4 5 … n … 20<br />

Elbows 4 4 4 4 4 4 4<br />

T’s 0 4 8 12 16 76<br />

13. Cannot be determined. SSA is not a congruence<br />

conjecture.<br />

14. AIN by SSS or SAS<br />

15. Cannot be determined. Parts do not correspond.<br />

16. SAO by SAS<br />

17. Cannot be determined. Parts do not correspond.<br />

18. RAY by SAS<br />

19. The midpoint of SD and PR is (0, 0).<br />

Therefore, DRO SPO by SAS.<br />

20. Because the LEV is marking out two triangles<br />

that are congruent by SAS, measuring the length<br />

of the segment leading to the finish will also<br />

approximate the distance across the crater.<br />

21. 22.<br />

Crosses 0 1 4 9 16 361<br />

23. a 37°, b 143°, c 37°, d 58°,<br />

e 37°, f 53°, g 48°, h 84°, k 96°,<br />

m 26°, p 69°, r 111°, s 69°; Possible<br />

explanation: The angle with measure h is the vertex<br />

angle of an isosceles triangle with base angles<br />

measuring 48°, so h 2(48) 180°, and h 84°.<br />

The angle with measure s and the angle with<br />

measure p are corresponding angles formed by<br />

parallel lines, so s p 69°.<br />

24. 3 cm third side 19 cm<br />

25. See table below.<br />

26a. y 6 b. y 13<br />

c. y <br />

3<br />

3<br />

x<br />

4<br />

2<br />

27. (5, 3)<br />

4n 4 (n 1) 2


LESSON 4.5<br />

1. If two angles and the included side of one triangle<br />

are congruent to the corresponding side and angles<br />

of another triangle, then the triangles are congruent.<br />

2. If two angles and a non-included side of one<br />

triangle are congruent to the corresponding side<br />

and angles of another triangle, then the triangles<br />

are congruent.<br />

If you know this: then you also know this:<br />

3. <strong>Answers</strong> will vary. Possible answer:<br />

4. ASA<br />

5. cannot be determined<br />

6. SAA<br />

7. cannot be determined<br />

8. ASA<br />

9. cannot be determined<br />

10. FED by SSS<br />

11. WTA by ASA or SAA<br />

12. SAT by SAS<br />

13. PRN by ASA or SAS; SRE by ASA<br />

14. Cannot be determined. Parts do not<br />

correspond.<br />

15. MRA by SAS<br />

16. Cannot be determined.AAA does not guarantee<br />

congruence.<br />

17. WKL by ASA<br />

18. Yes, ABC ADE by SAA or ASA.<br />

19. Slope AB slope CD 3 and slope BC <br />

slope DA 1<br />

3 ,soAB BC,CD DA , and<br />

BC DA . ABC CDA by SAA.<br />

20.<br />

21. The construction is the same as the<br />

construction using ASA once you find the third<br />

angle, which is used here. (Finding the third angle<br />

is not shown.)<br />

22. Construction will show a similar but larger<br />

(or smaller) triangle constructed from a drawn<br />

triangle by duplicating two angles on either end of a<br />

new side that is not congruent to the corresponding<br />

side.<br />

23. Draw a line segment. Construct a perpendicular.<br />

Bisect the right angle. Construct a triangle with<br />

two congruent sides and with a vertex that<br />

measures 135°.<br />

24. 125<br />

25. False. One possible counterexample is a kite.<br />

26. None. One triangle is determined by SAS.<br />

27.<br />

28a. about 100 km southeast of San Francisco<br />

28b. Yes. No, two towns would narrow it down<br />

to two locations. The third circle narrows it down<br />

to one.<br />

Eureka<br />

L<br />

K<br />

Reno<br />

Sacramento<br />

San<br />

Francisco<br />

CALIFORNIA<br />

Los Angeles<br />

Miles<br />

0 50 100 200<br />

0 100<br />

Kilometers<br />

400<br />

O R EGO N<br />

Elko<br />

N E VA D A<br />

Las<br />

Vegas<br />

M<br />

Boise<br />

I DAHO<br />

U TAH<br />

ARIZO NA<br />

ANSWERS TO EXERCISES 53<br />

<strong>Answers</strong> to Exercises


<strong>Answers</strong> to Exercises<br />

LESSON 4.6<br />

1. Yes. BD BD (same segment), A C<br />

(given), and ABD CBD (given), so DBA <br />

DBC by SAA. AB CB by CPCTC.<br />

2. Yes. CN WN and C W (given), and<br />

RNC ONW (vertical angles), CNR <br />

WON by ASA. RN ON by CPCTC.<br />

3. Cannot be determined. The congruent parts<br />

lead to the ambiguous case SSA.<br />

4. Yes. S I, G A (given), and TS IT<br />

(definition of midpoint), so TIA TSG by<br />

SAA. SG IA by CPCTC.<br />

5. Yes. FO FR and UO UR (given), and UF<br />

UF (same segment), so FOU FRU by SSS.<br />

O R by CPCTC.<br />

6. Yes. MN MA and ME MR (given), and<br />

M M (same angle), so EMA RMN by<br />

SAS. E R by CPCTC.<br />

7. Yes. BT EU and BU ET (given), and<br />

UT UT (same segment), so TUB UTE by<br />

SSS. B E by CPCTC.<br />

8. Cannot be determined. HLF LHA by<br />

ASA, but HA and HF are not corresponding sides.<br />

9. Cannot be determined. AAA does not guarantee<br />

congruence.<br />

10. Yes. The triangles are congruent by SAS.<br />

11. Yes. The triangles are congruent by SAS, and<br />

the angles are congruent by CPCTC.<br />

12. Draw AC and DF to form ABC and DEF.<br />

AB CB DE FE because all were drawn with<br />

the same radius. AC DF for the same reason.<br />

ABC DEF by SSS. Therefore, B E by<br />

CPCTC.<br />

21. (Lesson 4.6)<br />

54 ANSWERS TO EXERCISES<br />

13. cannot be determined<br />

14. KEI by ASA<br />

15. UTE by SAS<br />

16.<br />

17.<br />

18. a 112°, b 68°, c 44°, d 44°, e 136°,<br />

f 68°, g 68°, h 56°, k 68°, l 56°, m 124°;<br />

Possible explanation: f and g are measures of base<br />

angles of an isosceles triangle, so f g.The vertex<br />

angle measure is 44°, so subtract 44° from 180° and<br />

divide by 2 to get f 68°. The angle with measure<br />

m is the exterior angle of a triangle. Add the remote<br />

interior angle measures 56° and 68° to get<br />

m 124°.<br />

19. ASA. The “long segment in the sand” is a<br />

shared side of both triangles<br />

20. (4, 1)<br />

21. See table below.<br />

22. Value C is always decreasing.<br />

23. x 3, y 10<br />

Number of sides 3 4 5 6 7 … 12 … n<br />

Number of struts needed<br />

to make polygon rigid<br />

0 1 2 3 4 … 9 … n 3


1. See flowchart below.<br />

2. See flowchart below.<br />

1. (Lesson 4.7)<br />

LESSON 4.7<br />

3. See flowchart below.<br />

4. See flowchart below.<br />

<br />

<br />

<br />

<br />

<br />

1<br />

SE SU<br />

? Given<br />

2<br />

E U<br />

4<br />

ESM USO<br />

? ?<br />

5<br />

MS SO<br />

? Given ASA Congruence<br />

Conjecture<br />

? CPCTC<br />

3<br />

1 2<br />

1<br />

2<br />

?<br />

<br />

Vertical Angles Conjecture<br />

2. (Lesson 4.7)<br />

I is midpoint of CM<br />

Given<br />

I is midpoint of BL<br />

?<br />

<br />

1 NS is a median<br />

3<br />

4<br />

CI IM<br />

Definition<br />

of midpoint<br />

IL IB<br />

?<br />

<br />

5 1 2<br />

?<br />

<br />

3 NS NS<br />

Given Same segment<br />

6<br />

? ?<br />

<br />

<br />

<br />

2<br />

S is a midpoint<br />

4<br />

WS SE<br />

6<br />

ESN<br />

WSN ?<br />

7<br />

Definition<br />

of median<br />

1 NS is an<br />

angle bisector<br />

Given<br />

Given<br />

3. (Lesson 4.7)<br />

4. (Lesson 4.7)<br />

2<br />

5<br />

Definition<br />

of midpoint<br />

WN NE<br />

?<br />

<br />

Definition of ?<br />

<br />

angle bisector<br />

3 W E<br />

4<br />

1 ?<br />

<br />

?<br />

<br />

NS NS<br />

?<br />

<br />

Definition of<br />

midpoint<br />

?<br />

<br />

Vertical Angles Conjecture<br />

Given<br />

2<br />

?<br />

<br />

5<br />

? ?<br />

<br />

<br />

<br />

CIL MIB<br />

by SAS<br />

7<br />

?<br />

<br />

CPCTC<br />

W ?<br />

<br />

SSS ? CPCTC<br />

Given ? SAA<br />

? CPCTC<br />

Same segment<br />

WNS ENS<br />

6<br />

WN NE<br />

<br />

CL MB <br />

<br />

7 NEW is<br />

isosceles<br />

?<br />

<br />

E<br />

Definition of<br />

isosceles triangle<br />

ANSWERS TO EXERCISES 55<br />

<strong>Answers</strong> to Exercises


<strong>Answers</strong> to Exercises<br />

5. See flowchart below.<br />

6. Given: ABC with BA BC, CD AD <br />

Show: BD is the angle bisector of ABC.<br />

B<br />

1<br />

2<br />

A<br />

C<br />

BA BC CD AD BD BD<br />

Given Given<br />

Same segment<br />

ABD CBD<br />

SSS<br />

1 2<br />

CPCTC<br />

→<br />

BD bisects ABC<br />

Definition of angle<br />

bisector<br />

7. The angle bisector does not go to the midpoint<br />

of the opposite side in every triangle, only in an<br />

isosceles triangle.<br />

8. NE, because it is across from the smallest angle<br />

in NAE. It is shorter than AE, which is across<br />

from the smallest angle in LAE.<br />

9. The triangles are congruent by SSS, so the two<br />

central angles cannot have different measures.<br />

10. PRN by ASA; SRE by ASA<br />

11. Cannot be determined. Parts do not<br />

correspond.<br />

56 ANSWERS TO EXERCISES<br />

D<br />

5. (Lesson 4.7)<br />

1<br />

2<br />

SA NE<br />

?<br />

?<br />

Given<br />

SE NA<br />

Given<br />

3<br />

4<br />

5<br />

3 4<br />

AIA Conjecture<br />

SN SN<br />

12. ACK by SSS<br />

13. a 72°, b 36°, c 144°, d 36°, e 144°,<br />

f 18°, g 162°, h 144°, j 36°, k 54°,<br />

m 126°<br />

14. The circumcenter is equidistant from all three<br />

vertices because it is on the perpendicular bisector<br />

of each side. Every point on the perpendicular<br />

bisector of a segment is equidistant from the<br />

endpoints. Similarly, the incenter is equidistant<br />

from all three sides because it is on the angle<br />

bisector of each angle, and every point on an angle<br />

bisector is equidistant from the sides of the angle.<br />

15. ASA. The fishing pole forms the side.<br />

“Perpendicular to the ground” forms one angle.<br />

“Same angle on her line of sight” forms the other<br />

angle.<br />

16. 2<br />

7 <br />

17.<br />

18. y<br />

4<br />

Y<br />

X<br />

B O<br />

B' Y'4<br />

?<br />

? 6<br />

<br />

? <br />

? 7<br />

?<br />

?<br />

?<br />

1 2<br />

ESN ANS<br />

SA NE <br />

AIA Conjecture ASA<br />

CPCTC<br />

Same segment<br />

O'<br />

This proof shows that in a parallelogram,<br />

opposite sides are congruent.<br />

X'<br />

x


1. 6<br />

2. 90°; 18°<br />

3. 45°<br />

4. See flowchart below.<br />

5. See flowchart below.<br />

6. 1<br />

Isosceles ABC<br />

with AC BC<br />

and CD bisects<br />

C<br />

Given<br />

3<br />

AD BD<br />

CPCTC<br />

4. (Lesson 4.8)<br />

LESSON 4.8<br />

2<br />

ADC BDC<br />

Conjecture A<br />

(Exercise 4)<br />

4<br />

CD is a median<br />

Def. of median<br />

<br />

1<br />

CD is the bisector of C<br />

2 ? 1 2<br />

Given Definition of<br />

angle bisector<br />

5. (Lesson 4.8)<br />

1 ABC is isosceles with<br />

AC BC, and CD is<br />

the bisector of C<br />

Given<br />

3<br />

4<br />

ABC is isosceles<br />

with AC BC<br />

Given<br />

CD CD<br />

Same segment<br />

2 ADC BDC<br />

Conjecture A<br />

3<br />

1 and 2 form<br />

a linear pair<br />

Definition of<br />

linear pair<br />

7.<br />

1<br />

AC BC<br />

2<br />

CD CD<br />

3<br />

D is<br />

Given Same segment<br />

midpoint<br />

of AB<br />

Given<br />

5<br />

ADC BDC<br />

4<br />

AD BD<br />

SSS<br />

Def. of midpoint<br />

6 ACD DCB<br />

CPCTC<br />

8<br />

CD is altitude<br />

of ABC<br />

Conjecture B<br />

(Exercise 5)<br />

7 CD is angle<br />

bisector of ACB<br />

Def. of angle bisector<br />

9<br />

CD AB<br />

Def. of altitude<br />

8. Yes. First show that the three exterior triangles<br />

are congruent by SAS.<br />

4 1 2<br />

?<br />

<br />

CPCTC<br />

5 ADC BDC<br />

?<br />

<br />

5 1 and 2 are<br />

supplementary<br />

Linear Pair<br />

Conjecture<br />

SAS<br />

7<br />

Definition of<br />

perpendicular<br />

CD is an altitude<br />

6 1 and 2 are<br />

right angles<br />

8<br />

Congruent<br />

supplementary<br />

angles are 90<br />

CD AB <br />

?<br />

<br />

?<br />

<br />

Definition of<br />

altitude<br />

ANSWERS TO EXERCISES 57<br />

<strong>Answers</strong> to Exercises


<strong>Answers</strong> to Exercises<br />

9.<br />

C<br />

A D B<br />

Drawing the vertex angle bisector as an auxiliary<br />

segment, we have two triangles. We can show them<br />

to be congruent by SAS, as we did in Exercise 4.<br />

Then, A B, by CPCTC. Therefore, base angles<br />

of an isosceles triangle are congruent.<br />

10. The proof is similar to the one on page 245,<br />

but in reverse, and using the Converse of the<br />

Isosceles Triangle Conjecture.<br />

11.<br />

30<br />

12. a 128°, b 128°, c 52°, d 76°, e 104°,<br />

f 104°, g 76°, h 52°, j 70°, k 70°, l 40°,<br />

m 110°, n 58°<br />

13. between 16 and 17 minutes<br />

58 ANSWERS TO EXERCISES<br />

14. y 5<br />

x<br />

3<br />

16<br />

15. 120<br />

16. (4, 6) or (4, 0) or any point at which the<br />

x-coordinate is either 1 or 7 and the y-coordinate<br />

does not equal 3<br />

17. Hugo and Duane can locate the site of the<br />

fireworks by creating a diagram using SSS.<br />

340 m/s 5 s<br />

= 1.7 km<br />

18. C n H 2n<br />

H<br />

Duane<br />

Fireworks<br />

H<br />

H H<br />

H<br />

C<br />

C C<br />

1.5 km<br />

H<br />

H<br />

C C<br />

H<br />

H<br />

C C<br />

H<br />

H<br />

H H<br />

H<br />

340 m/s 3 s<br />

= 1.02 km<br />

Hugo


CHAPTER 4 REVIEW<br />

1. Their rigidity gives strength.<br />

2. The Triangle Sum Conjecture states that the<br />

sum of the measures of the angles in every triangle<br />

is 180°. Possible answers: It applies to all triangles;<br />

many other conjectures rely on it.<br />

3. The angle bisector of the vertex angle is also the<br />

median and the altitude.<br />

4. The distance between A and B is along the<br />

segment connecting them. The distance from A<br />

to C to B can’t be shorter than the distance from A<br />

to B. Therefore, AC CB AB. Points A, B, and C<br />

form a triangle. Therefore, the sum of the lengths<br />

of any two sides is greater than the length of the<br />

third side.<br />

5. SSS, SAS, ASA, or SAA<br />

6. In some cases, two different triangles can<br />

be constructed using the same two sides and<br />

non-included angle.<br />

7. cannot be determined<br />

8. ZAP by SAA<br />

9. OSU by SSS<br />

10. cannot be determined<br />

11. APR by SAS<br />

12. NGI by SAS<br />

13. cannot be determined<br />

14. DCE by SAA or ASA<br />

15. RBO or OBR by SAS<br />

16. AMD UMT by SAS, AD UT by<br />

CPCTC<br />

17. cannot be determined<br />

18. cannot be determined<br />

19. TRI ALS by SAA, TR AL by<br />

CPCTC<br />

20. SVE NIK by SSS, EL KV by<br />

overlapping segments property<br />

21. cannot be determined<br />

22. cannot be determined<br />

23. LAZ IAR by ASA, LRI IZL by<br />

ASA, and LRD IZD by ASA<br />

24. yes. PTS TPO by ASA or SAA<br />

25. ANG is isosceles, so A G. However,<br />

the sum of mA mN mG 188°. The<br />

measures of the three angles of a triangle must sum<br />

to 180°.<br />

26. ROW NOG by ASA, implying that<br />

OW OG . However, the two segments shown are<br />

not equal in measure.<br />

27. a g e d b f c. Thus, c is the<br />

longest segment, and a and g are the shortest.<br />

28. x 20°<br />

29. Yes. TRE SAE by SAA, so sides are<br />

congruent by CPCTC.<br />

30. Yes. FRM RFA by SAA. RFM <br />

FRA by CPCTC. Because base angles are<br />

congruent, FRD is isosceles.<br />

31. x 48°<br />

32. The legs form two triangles that are congruent<br />

by SAS. Because alternate interior angles are<br />

congruent by CPCTC, the seat must be parallel to<br />

the floor.<br />

33. Construct P and A to be adjacent. The<br />

angle that forms a linear pair with the conjunction<br />

of P and A is L. Construct A. Mark off the<br />

length AL on one ray. Construct L. Extend the<br />

unconnected sides of the angles until they meet.<br />

Label the point of intersection P.<br />

P<br />

A y L<br />

34. Construct P. Mark off the length PB on one<br />

ray. From point B, mark off the two segments that<br />

intersect the other ray of P at distance x.<br />

S 1<br />

A<br />

L P<br />

P z<br />

B<br />

x<br />

S 2<br />

x<br />

ANSWERS TO EXERCISES 59<br />

<strong>Answers</strong> to Exercises


<strong>Answers</strong> to Exercises<br />

35. See flowchart below.<br />

36. Given three sides, only one triangle is possible;<br />

therefore, the shelves on the right hold their shape.<br />

The shelves on the left have no triangles and move<br />

freely as a parallelogram.<br />

35. (Chapter 4 Review)<br />

60 ANSWERS TO EXERCISES<br />

TMI RME<br />

2<br />

? ?<br />

<br />

<br />

<br />

37. Possible method: Construct an equilateral<br />

triangle and bisect one angle to obtain 30°.<br />

Adjacent to that angle, construct a right angle<br />

and bisect it to obtain 45°.<br />

38. d, a b, c, e, f<br />

<br />

<br />

<br />

<br />

<br />

<br />

<br />

<br />

<br />

<br />

<br />

<br />

<br />

<br />

<br />

<br />

<br />

Vertical angles ?<br />

TM ME TMI EMR<br />

T E<br />

or<br />

R I<br />

TI RE<br />

M is midpoint<br />

of TE and IR<br />

1 ?<br />

3 ? ?<br />

? Definition<br />

of midpoint<br />

5<br />

? ?<br />

? SAS<br />

6<br />

? ?<br />

? CPCTC<br />

7 ? ?<br />

? Converse of<br />

AIA Conjecture<br />

? Given<br />

4 ? ?<br />

IM MR <br />

? Definition of midpoint


<strong>Answers</strong> to Exercises<br />

5<br />

CHAPTER 5 • CHAPTER CHAPTER 5 • CHAPTER<br />

LESSON 5.1<br />

1. See table below.<br />

2. See table below.<br />

3. 122°<br />

4. 136°<br />

5. 108°; 36°<br />

6. 108°; 106°<br />

7. 105°; 82°<br />

8. 120°; 38°<br />

9. The sum of the interior angle measures of the<br />

quadrilateral is 358°. It should be 360°.<br />

10. The measures of the interior angles shown sum<br />

to 554°. However, the figure is a pentagon, so the<br />

measures of its interior angles should sum to 540°.<br />

11. 18<br />

12. a 116°, b 64°, c 90°, d 82°, e 99°,<br />

f 88°, g 150°, h 56°, j 106°, k 74°,<br />

m 136°, n 118°, p 99°; Possible explanation:<br />

The sum of the angles of a quadrilateral is 360°, so<br />

a b 98° d 360°. Substituting 116° for a<br />

and 64° for b gives d 82°. Using the larger<br />

quadrilateral, e p 64° 98° 360°.<br />

Substituting e for p, the equation simplifies to<br />

2e 198°, so e 99°. The sum of the angles of a<br />

pentagon is 540°, so e p f 138° 116° <br />

540°. Substituting 99° for e and p gives f 88°.<br />

1. (Lesson 5.1)<br />

13. 17<br />

14. 15<br />

15. the twelfth century<br />

16. The angles of the trapezoid measure 67.5° and<br />

112.5°; 67.5° is half the value of each angle of a regular<br />

octagon, and 112.5° is half the value of 360° 135°.<br />

135<br />

67.5<br />

17. <strong>Answers</strong> will vary; see the answer for<br />

Developing Proof on page 259. Using the Triangle<br />

Sum Conjecture, a b j c d k e f <br />

l g h i 4(180°), or 720°. The four angles in<br />

the center sum to 360°, so j k l i 360°.<br />

Subtract to get a b c d e f g <br />

h 360°.<br />

18. x 120°<br />

19. The segments joining the opposite midpoints<br />

of a quadrilateral always bisect each other.<br />

20. D<br />

21. Counterexample: The base angles of an<br />

isosceles right triangle measure 45°; thus they are<br />

complementary.<br />

Number of sides of polygon 7 8 9 10 11 20 55 100<br />

Sum of measures of angles 900° 1080° 1260° 1440° 1620° 3240° 9540° 17640°<br />

2. (Lesson 5.1)<br />

Number of sides of 5 6 7 8 9 10 12 16 100<br />

equiangular polygon<br />

Measure of each angle 108° 120° 128 4<br />

°<br />

7 135° 140° 144° 150° 1571 ° 1762<br />

2 5 °<br />

of equiangular polygon<br />

ANSWERS TO EXERCISES 61<br />

<strong>Answers</strong> to Exercises


<strong>Answers</strong> to Exercises<br />

LESSON 5.2<br />

1. 360°<br />

2. 72°; 60°<br />

3. 15<br />

4. 43<br />

5. a 108°<br />

6. b 45 1<br />

3 °<br />

7. c 51 3<br />

°,<br />

7<br />

d 115 5<br />

7 °<br />

8. e 72°, f 45°, g 117°, h 126°<br />

9. a 30°, b 30°, c 106°, d 136°<br />

10. a 162°, b 83°, c 102°, d 39°, e 129°,<br />

f 51°, g 55°, h 97°, k 83°<br />

11. See flowchart below.<br />

12. Yes. The maximum is three. The minimum is<br />

zero. A polygon might have no acute interior<br />

angles.<br />

11. (Lesson 5.2)<br />

?<br />

Linear Pair Conjecture<br />

?<br />

Linear Pair Conjecture<br />

?<br />

Linear Pair Conjecture<br />

62 ANSWERS TO EXERCISES<br />

1 a b 180<br />

2 c d 180<br />

3 e f 180<br />

13. <strong>Answers</strong> will vary. Possible proof using<br />

the diagram on the left: a b i 180°, c d <br />

h 180°, and e f g 180° by the Triangle<br />

Sum Conjecture. a b c d e f g <br />

h i 540° by the addition property of equality.<br />

Therefore, the sum of the measures of the angles of<br />

a pentagon is 540°. To use the other diagram,<br />

students must remember to subtract 360° to<br />

account for angle measures k through o.<br />

14. regular polygons: equilateral triangle and<br />

regular dodecagon; angle measures: 60°, 150°,<br />

and 150°<br />

15. regular polygons: square, regular hexagon, and<br />

regular dodecagon; angle measures: 90°, 120°,<br />

and 150°<br />

16. Yes. RAC DCA by SAS. AD CR by<br />

CPCTC.<br />

17. Yes. DAT RAT by SSS. D R by<br />

CPCTC.<br />

540°<br />

4<br />

a + b + c + d + e + f = ? <br />

Addition property<br />

of equality<br />

?<br />

? 180°<br />

5<br />

a + c + e = <br />

Triangle Sum Conjecture<br />

6<br />

b + d + f = ?<br />

360°<br />

Subtraction property<br />

of equality


LESSON 5.3<br />

1. 64 cm 2. 21°; 146° 3. 52°; 128°<br />

4. 15 cm 5. 72°; 61° 6. 99°; 38 cm<br />

7. w 120°, x 45°, y 30°<br />

8. w 1.6 cm, x 48°, y 42°<br />

9. See flowchart below.<br />

10. <strong>Answers</strong> may vary. This proof uses the Kite<br />

Angle Bisector Conjecture.<br />

Y<br />

2<br />

B N<br />

1 X<br />

E<br />

Given: Kite BENY with vertex angles B and N<br />

Show: Diagonal BN is the perpendicular bisector<br />

of diagonal YE.<br />

From the definition of kite, BE BY.From the<br />

Kite Angle Bisector Conjecture, 1 2. BX <br />

BX because they are the same segment. By SAS,<br />

BXY BXE. So by CPCTC, XY XE.<br />

Because YXB and EXB form a linear pair, they<br />

are supplementary, so mYXB mEXB <br />

180°. By CPCTC, YXB EXB, or mYXB <br />

mEXB, so by substitution, 2mYXB 180°, or<br />

mYXB 90°. So mYXB mEXB 90°.<br />

Because XY XE and YXB and EXB are right<br />

angles, BN is the perpendicular bisector of YE.<br />

11. possible answer: E I<br />

I<br />

12. possible answer:<br />

Z I<br />

Q U<br />

The other base is ZI . Q and U are a pair of base<br />

angles. Z and I are a pair of base angles.<br />

9. (Lesson 5.3)<br />

1 BE BY<br />

Given<br />

2 EN YN<br />

3<br />

Given<br />

BN BN<br />

<br />

Same segment<br />

? <br />

?<br />

T<br />

E<br />

K<br />

13. possible answer:<br />

OW is the other base. S and H are a pair of<br />

base angles. O and W are a pair of base angles.<br />

SW HO .<br />

14. Only one kite is possible<br />

F<br />

because three sides determine<br />

a triangle.<br />

N<br />

B<br />

15.<br />

E<br />

16. infinitely many, possible construction:<br />

E<br />

W O<br />

S H<br />

I<br />

B O<br />

S H<br />

N<br />

W<br />

17. 80°, 80°, 100°, 100°<br />

18. Because ABCD is an isosceles trapezoid, A<br />

B. AGF BHE by SAA. Thus, AG BH <br />

by CPCTC.<br />

19. a 80°, b 20°, c 160°, d 20°, e 80°,<br />

f 80°, g 110°, h 70°, m 110°, n 100°;<br />

Possible explanation: Because d forms a linear pair<br />

with e and its congruent adjacent angle,d 2e <br />

180°.Substituting d 20° gives 2e 160°,so e 80°.<br />

Using theVerticalAngles Conjecture and d 20°, the<br />

unlabeled angle in the small right triangle measures<br />

20°, which means h 70°. Because g and h are a<br />

linear pair, they are supplementary, so g 110°.<br />

? <br />

5 1 and<br />

3 <br />

6<br />

and <br />

?<br />

? 4<br />

Congruence<br />

shortcut<br />

<br />

Definition of<br />

angle bisector<br />

?<br />

BEN ?<br />

?<br />

?<br />

BYN 2 BN bisects B, BN bisects N<br />

SSS<br />

4<br />

CPCTC<br />

ANSWERS TO EXERCISES 63<br />

<strong>Answers</strong> to Exercises


<strong>Answers</strong> to Exercises<br />

LESSON 5.4<br />

1. three; one 2. 28<br />

3. 60°; 140° 4. 65°<br />

5. 23 6. 129°; 73°; 42 cm<br />

7. 35 8. See flowchart below.<br />

9. Parallelogram. Draw a diagonal of the original<br />

quadrilateral. The diagonal forms two tri-angles.<br />

Each of the two midsegments is parallel to the<br />

diagonal, and thus the midsegments are parallel to<br />

each other. Now draw the other diagonal of the<br />

original quadrilateral. By the same reasoning, the<br />

second pair of midsegments is parallel. Therefore,<br />

the quadrilateral formed by joining the midpoints is<br />

a parallelogram.<br />

10. The length of the edge of the top base<br />

measures 30 m. We know this by the Trapezoid<br />

Midsegment Conjecture.<br />

11. Ladie drives a stake into the ground to create a<br />

triangle for which the trees are the other two<br />

vertices. She finds the midpoint from the stake to<br />

each tree. The distance between these midpoints is<br />

half the distance between the trees.<br />

12. Explanations will vary.<br />

80<br />

Cabin<br />

40 60<br />

60 cm<br />

8. (Lesson 5.4)<br />

64 ANSWERS TO EXERCISES<br />

1<br />

2<br />

FOA with<br />

midsegment LN<br />

Given<br />

IOA with<br />

midsegment RD<br />

Given<br />

3<br />

4<br />

LN OA<br />

?<br />

<br />

Triangle Midsegment Conjecture<br />

OA RD <br />

?<br />

<br />

13. If a quadrilateral is a kite, then exactly one<br />

diagonal bisects a pair of opposite angles. Both the<br />

original and converse statements are true.<br />

14. a 54°, b 72°, c 108°, d 72°, e 162°,<br />

f 18°, g 81°, h 49.5°, i 130.5°, k 49.5°,<br />

m 162°, n 99°; Possible explanation: The third<br />

angle of the triangle containing f and g measures<br />

81°, so using the Vertical Angles Conjecture, the<br />

vertex angle of the triangle containing h also<br />

measures 81°. Subtract 81° from 180° and divide by<br />

2 to get h 49.5°. The other base angle must also<br />

measure 49.5°. By the Corresponding Angles<br />

Conjecture, k 49.5°.<br />

15. (3, 8)<br />

16. (0, 8)<br />

17. coordinates: E(2, 3.5), Z(6, 5); the slope of<br />

EZ 3<br />

8 ,and the slope ofYT 3<br />

8 <br />

18.<br />

F<br />

R<br />

K<br />

N<br />

There is only one kite, but more than one way to<br />

construct it.<br />

Triangle Midsegment<br />

Conjecture<br />

5<br />

LN RD <br />

?<br />

<br />

Two lines parallel to the<br />

same line are parallel


1. 34 cm; 27 cm<br />

2. 132°; 48°<br />

3. 16 in.; 14 in.<br />

4. 63 m<br />

5. 80<br />

6. 63°; 78°<br />

7.<br />

8.<br />

L<br />

D<br />

P<br />

P<br />

9. <br />

V h<br />

13. (Lesson 5.5)<br />

R<br />

LESSON 5.5<br />

T S<br />

A<br />

O<br />

R<br />

<br />

V w<br />

10.<br />

11. (b a, c)<br />

12. possible answer:<br />

a<br />

b<br />

c<br />

b d<br />

c a<br />

1<br />

LEAN is a<br />

3<br />

AEN LNE<br />

parallelogram<br />

<br />

AIA Conjecture<br />

7<br />

2<br />

EA LN<br />

?<br />

?<br />

4<br />

<br />

AIA Conjecture<br />

?<br />

6<br />

<br />

ASA<br />

?<br />

8<br />

5<br />

?<br />

Given<br />

EAL NLA AET LNT<br />

Definition of<br />

parallelogram AE LN<br />

Opposite sides<br />

congruent<br />

<br />

V c<br />

d<br />

<br />

V b<br />

13. See flowchart below.<br />

14. The parallelogram linkage is used for the<br />

sewing box so that the drawers remain parallel to<br />

each other (and to the ground) so that the contents<br />

cannot fall out.<br />

15. a 135°, b 90°<br />

16. a 120°, b 108°, c 90°, d 42°, e 69°<br />

ET NT<br />

CPCTC<br />

AT LT<br />

?<br />

CPCTC<br />

9<br />

EN and LA bisect<br />

each other<br />

?<br />

Definition of<br />

segment bisector<br />

ANSWERS TO EXERCISES 65<br />

<strong>Answers</strong> to Exercises


<strong>Answers</strong> to Exercises<br />

17. x 104°, y 98°. The quadrilaterals on the<br />

left and right sides are kites. Nonvertex angles are<br />

congruent. The quadrilateral at the bottom is an<br />

isosceles trapezoid. Base angles are congruent, and<br />

consecutive angles between the bases are<br />

supplementary.<br />

18. a 84°, b 96°<br />

19. No. The congruent angles and side do not<br />

correspond.<br />

20.<br />

21. Parallelogram. Because the triangles are<br />

congruent by SAS, 1 2. So, the segments are<br />

66 ANSWERS TO EXERCISES<br />

parallel. Use a similar argument to show that the<br />

other pair of opposite sides is parallel.<br />

1<br />

2<br />

22. Kite or dart. Radii of the same circle are<br />

congruent. If the circles have equal radii, a rhombus<br />

is formed.


1.<br />

2.<br />

3.<br />

(0, 4)<br />

y<br />

y<br />

y<br />

(0, 6)<br />

USING YOUR ALGEBRA SKILLS 5<br />

(0, 1)<br />

(3, 8)<br />

(1, –1)<br />

(2, 9)<br />

x<br />

x<br />

x<br />

4. y x 2<br />

6<br />

5. y 13<br />

x 74<br />

<br />

13<br />

6. y x 1<br />

7. y 3x 5<br />

8. y 2<br />

x<br />

5<br />

8<br />

5 <br />

9. y 80 4x<br />

10. y 3x 26<br />

11. y 1<br />

x<br />

4<br />

3<br />

12. y 6<br />

5 x<br />

13. y x 1<br />

14. y 2 3<br />

x<br />

9<br />

4 <br />

9<br />

ANSWERS TO EXERCISES 67<br />

<strong>Answers</strong> to Exercises


<strong>Answers</strong> to Exercises<br />

LESSON 5.6<br />

1. Sometimes true; it is true only if the<br />

parallelogram is a rectangle.<br />

2. Always true; by the definition of rectangle, all<br />

the angles are congruent. By the Quadrilateral Sum<br />

Conjecture and division, they all measure 90°, so<br />

any two angles in a rectangle, including consecutive<br />

angles, are supplementary.<br />

3. Always true by the Rectangle Diagonals<br />

Conjecture.<br />

4. Sometimes true; it is true only if the rectangle is<br />

a square.<br />

true<br />

true<br />

5. Always true by the Square Diagonals Conjecture.<br />

6. Sometimes true; it is true only if the rhombus is<br />

equiangular.<br />

true<br />

false<br />

false<br />

false<br />

7. Always true; all squares fit the definition of<br />

rectangle.<br />

8. Always true; all sides of a square are congruent<br />

and form right angles, so the sides become the legs<br />

of the isosceles right triangle and the diagonal is<br />

the hypotenuse.<br />

9. Always true by the Parallelogram Opposite<br />

Angles Conjecture.<br />

10. Sometimes true; it is true only if the<br />

parallelogram is a rectangle. Consecutive angles of<br />

a parallelogram are always supplementary, but are<br />

congruent only if they are right angles.<br />

11. 20<br />

12. 37°<br />

13. 45°, 90°<br />

14. DIAM is not a rhombus because it is not<br />

equilateral and opposite sides are not parallel.<br />

15. BOXY is a rectangle because its adjacent sides<br />

are perpendicular.<br />

16. Yes. TILE is a rhombus, and a rhombus is a<br />

parallelogram.<br />

68 ANSWERS TO EXERCISES<br />

17.<br />

18. Constructions will vary.<br />

B<br />

A<br />

19. one possible construction:<br />

I<br />

E V<br />

L O<br />

E<br />

K<br />

P S<br />

E<br />

20. Converse: If the diagonals of a quadrilateral<br />

are congruent and bisect each other, then the<br />

quadrilateral is a rectangle.<br />

Given: Quadrilateral ABCD with diagonals<br />

AC BD. AC and BD bisect each other<br />

Show: ABCD is a rectangle<br />

A<br />

8 E<br />

B<br />

D<br />

C<br />

1 2 3<br />

7<br />

6 5<br />

4<br />

B<br />

A<br />

E<br />

Because the diagonals are congruent and bisect<br />

each other, AE BE DE EC.Using the<br />

Vertical Angles Conjecture, AEB CED and<br />

BEC DEA.So AEB CED and AED<br />

CEB by SAS. Using the Isosceles Triangle<br />

Conjecture and CPCTC, 1 2 5 6,<br />

and 3 4 7 8. Each angle of the<br />

quadrilateral is the sum of two angles, one from<br />

each set, so for example, mDAB m1 m8.<br />

By the addition property of equality, m1 <br />

m8 m2 m3 m5 m4 m6 <br />

m7. So mDAB mABC mBCD <br />

mCDA. So the quadrilateral is equiangular. Using<br />

1 5 and the Converse of AIA, AB CD.<br />

Using 3 7 and the Converse of AIA,<br />

BC AD . Therefore ABCD is an equiangular<br />

parallelogram, so it is a rectangle.<br />

21. If the diagonals are congruent and bisect each<br />

other, then the room is rectangular (converse of the<br />

Rectangle Diagonals Conjecture).<br />

K


22. The platform stays parallel to the floor<br />

because opposite sides of a rectangle are parallel<br />

(a rectangle is a parallelogram).<br />

23. The crosswalks form a parallelogram: The<br />

streets are of different widths, so the crosswalks are<br />

of different lengths. The streets would have to meet<br />

at right angles for the crosswalks to form a rectangle.<br />

The corners would have to be right angles and the<br />

streets would also have to be of the same width for<br />

the crosswalk to form a square.<br />

24. Place one side of the ruler along one side of<br />

the angle. Draw a line with the other side of the<br />

ruler. Repeat with the other side of the angle. Draw<br />

a line from the vertex of the angle to the point<br />

where the two lines meet.<br />

25. Rotate your ruler so that each endpoint of the<br />

segment barely shows on each side of the ruler.<br />

Draw the parallel lines on each side of your ruler.<br />

Now rotate your ruler the other way and repeat the<br />

process to get a rhombus. The original segment is<br />

one diagonal of the rhombus. The other diagonal<br />

will be the perpendicular bisector of the original<br />

segment.<br />

26. See flowchart below.<br />

27. Yes, it is true for rectangles.<br />

Given: 1 2 3 4<br />

Show: ABCD is a rectangle<br />

By the Quadrilateral Sum Conjecture, m1 <br />

m2 m3 m4 360°. It is given that all<br />

four angles are congruent, so each angle measures<br />

26. (Lesson 5.6)<br />

1<br />

2<br />

3<br />

QU AD<br />

Given<br />

QD AU<br />

?<br />

?<br />

Given SSS<br />

DU DU<br />

Same segment<br />

4<br />

QUD ADU<br />

8<br />

Given<br />

5<br />

1 2<br />

3 4<br />

? CPCTC<br />

QU UA AD DQ<br />

90°. Because 4 and 5 form a linear pair,<br />

m4 m5 180°. Substitute 90° for m4 and<br />

solve to get m5 90°. By definition of congruent<br />

angles, 5 3, and 5 and 3 are alternate<br />

interior angles, so AD BC by the Converse of the<br />

Parallel Lines Conjecture. Similarly, 1 and 5 are<br />

congruent corresponding angles, so AB CD by the<br />

Converse of the Parallel Lines Conjecture. Thus,<br />

ABCD is a parallelogram by the definition of<br />

parallelogram. Because it is an equiangular<br />

parallelogram, ABCD is a rectangle.<br />

28. a 54°, b 36°, c 72°, d 108°, e 36°,<br />

f 144°, g 18°, h 48°, j 48°, k 84°<br />

29. possible answers: (1, 0); (0, 1); (1, 2); (2, 3)<br />

30. y 8 6<br />

x<br />

9<br />

8 <br />

9<br />

or 8x 9y 86<br />

7<br />

31. y x 10<br />

12<br />

<br />

5<br />

or 7x 10y 24<br />

32. velocity 1.8 mi/h; angle of path 106.1°<br />

clockwise from the north<br />

60<br />

1.5 mi/h<br />

2 mi/h<br />

6<br />

QU AD<br />

7 QUAD is a<br />

QD AU<br />

parallelogram<br />

Converse of the<br />

Parallel Lines<br />

Conjecture<br />

9<br />

Definition of<br />

rhombus<br />

Definition of<br />

parallelogram<br />

QUAD is a<br />

rhombus<br />

?<br />

ANSWERS TO EXERCISES 69<br />

<strong>Answers</strong> to Exercises


<strong>Answers</strong> to Exercises<br />

LESSON 5.7<br />

1. See flowchart below. 2. See flowchart below.<br />

3. See flowchart below.<br />

4.<br />

1 SP OA<br />

1<br />

1<br />

Given<br />

1. (Lesson 5.7)<br />

Given<br />

2 SP OA<br />

Given<br />

3 1 2<br />

Same segment<br />

70 ANSWERS TO EXERCISES<br />

4<br />

SOAK is a<br />

parallelogram<br />

2. (Lesson 5.7)<br />

Given<br />

? Given<br />

AIA Conjecture<br />

PO PO<br />

Parallelogram BATH<br />

with diagonal BT<br />

4 Parallelogram BATH<br />

with diagonal HA<br />

3. (Lesson 5.7)<br />

?<br />

? <br />

?<br />

Same segment<br />

2<br />

3<br />

5<br />

SOP APO<br />

SAS<br />

6<br />

3 4<br />

CPCTC<br />

7 PA SO<br />

Converse of<br />

AIA Conjecture<br />

8 SOAP is a parallelogram<br />

SO KA<br />

Definition of parallelogram<br />

Definition of<br />

parallelogram<br />

SK<br />

OA ?<br />

<br />

4<br />

<br />

? <br />

?<br />

?<br />

? ?<br />

? 1<br />

<br />

?<br />

? WA RT<br />

Given<br />

WR AT WRT TAW<br />

2<br />

<br />

?<br />

Given<br />

WT WT <br />

SSS<br />

3<br />

?<br />

2<br />

Definition of<br />

parallelogram<br />

4<br />

5<br />

6<br />

BAT THB<br />

5. parallelogram<br />

6. sample flowchart proof:<br />

1 2<br />

IY GO<br />

YOG OYI<br />

Opposite sides of<br />

rectangle are congruent<br />

3 4<br />

AIA<br />

1 2<br />

?<br />

SA <br />

<br />

? SA<br />

?<br />

Conjecture proved in CPCTC<br />

Exercise 1<br />

5<br />

BAH = <br />

6<br />

HBA <br />

?<br />

?<br />

?<br />

?<br />

THA<br />

ATH<br />

Conjecture proved<br />

in Exercise 1<br />

CPCTC<br />

5<br />

7<br />

AIA<br />

Same segment<br />

7<br />

3 BAT THB<br />

Definition of rectangle<br />

4<br />

YOG OYI<br />

5<br />

SAS<br />

YG IO<br />

CPCTC<br />

SOA <br />

?<br />

?<br />

ASA<br />

AKS<br />

3<br />

YO OY<br />

Same segment<br />

1 <br />

? 6<br />

<br />

9 WATR is a<br />

parallelogram<br />

4 <br />

? 8<br />

? <br />

?<br />

? <br />

<br />

?<br />

?<br />

?<br />

?<br />

?<br />

?<br />

2 RT WA by Converse of the Parallel<br />

Lines Conjecture<br />

CPCTC<br />

3<br />

RW TA<br />

CPCTC<br />

Definition of<br />

parallelogram<br />

Converse of the<br />

Parallel Lines Conjecture


7.<br />

2<br />

RE AB <br />

3<br />

BR EA <br />

4<br />

AR EB <br />

Given Parallelogram Opposite<br />

Sides Conjecture<br />

5<br />

6<br />

1 BEAR is a parallelogram<br />

Given<br />

EBR ARB RAE BEA<br />

Parallelogram Opposite<br />

Sides Conjecture<br />

8. Because AR is parallel to ZT, corresponding<br />

3 and 2 are congruent. Opposite sides of<br />

parallelogram ZART are congruent so AR TZ.<br />

Because the trapezoid is isosceles, AR PT, and<br />

substituting gives ZT PT making PTZ<br />

isosceles and 1 and 2 congruent. By<br />

substitution, 1 and 3 are congruent.<br />

SSS<br />

EBR ARB RAE BEA<br />

CPCTC<br />

7 BEAR is a rectangle<br />

Definition of rectangle<br />

9. (Lesson 5.7)<br />

1<br />

Isosceles trapezoid<br />

GTHR<br />

Given<br />

2<br />

GR TH <br />

Given<br />

9. See sample flowchart below.<br />

10. Ifthefabricispulledalongthewarportheweft,<br />

nothing happens. However, if the fabric is pulled<br />

along the bias, it can be stretched because the<br />

rectangles are pulled into parallelograms.<br />

11. 30° angles in 4-pointed star, 30° angles in<br />

6-pointed star; yes<br />

12. Heshouldmeasurethealternateinterior<br />

angles to see whether they’re congruent.If they are,<br />

theedgesareparallel.<br />

13. ES : y 2x 3; QI : y 1 x 2 2<br />

14. (12, 7)<br />

15. 1<br />

3 <br />

16.<br />

9<br />

6 inches<br />

12 3<br />

3<br />

GT GT<br />

<br />

5<br />

RGT HTG<br />

6<br />

GH TR <br />

Same segment<br />

SAS CPCTC<br />

4<br />

RGT HTG<br />

Isosceles Trapezoid<br />

Conjecture<br />

6<br />

ANSWERS TO EXERCISES 71<br />

<strong>Answers</strong> to Exercises


<strong>Answers</strong> to Exercises<br />

CHAPTER 5 REVIEW<br />

1. 360° divided by the number of sides<br />

2. Sample answers: Using an interior angle, set the<br />

interior angle measure formula equal to the angle<br />

and solve for n. Using an exterior angle, divide into<br />

360°. Or find the interior angle measure and go<br />

from there.<br />

3. Trace both sides of the ruler as shown at right.<br />

4. Make a rhombus using the double-edged<br />

straightedge, and draw a diagonal connecting the<br />

angle vertex to the opposite vertex.<br />

5. Sample answer: Measure the diagonals with<br />

string to see if they are congruent and bisect each<br />

other.<br />

6. Sample answer: Draw a third point and connect<br />

it with each of the two points to form two sides of a<br />

triangle. Find the midpoints of the two sides and<br />

connect them to construct the midsegment. The<br />

13. (Chapter 5 Review)<br />

Opposite sides<br />

are parallel<br />

Opposite sides<br />

are congruent<br />

Opposite angles<br />

are congruent<br />

72 ANSWERS TO EXERCISES<br />

distance between the two points is twice the length<br />

of the midsegment.<br />

7. x 10°, y 40° 8. x 60 cm<br />

9. a 116°, c 64° 10. 100<br />

11. x 38 cm<br />

12. y 34 cm, z 51 cm<br />

13. See table below.<br />

14. a 72°, b 108°<br />

15. a 120°, b 60°, c 60°, d 120°, e 60°,<br />

f 30°, g 108°, m 24°, p 84°; Possible<br />

explanation: Because c 60°, the angle that forms<br />

a linear pair with e and its congruent adjacent<br />

angle measures 60°. So 60° 2e 180°, and<br />

e 60°. The triangle containing f has a 60° angle.<br />

The other angle is a right angle because it forms a<br />

linear pair with a right angle. So f 30° by the<br />

Triangle Sum Conjecture. Because g is an interior<br />

angle in an equiangular pentagon, divide 540° by 5<br />

to get g 108°.<br />

16. 15 stones<br />

17. (1, 0)<br />

18. When the swing is motionless, the seat, the bar<br />

at the top, and the chains form a rectangle. When<br />

you swing left to right, the rectangle changes to a<br />

parallelogram. The opposite sides stay equal in<br />

length,so they stay parallel.The seat and the bar<br />

at the top are also parallel to the ground.<br />

19. a = 60°, b = 120°<br />

Isosceles<br />

Kite trapezoid Parallelogram Rhombus Rectangle Square<br />

No No Yes Yes Yes Yes<br />

No No Yes Yes Yes Yes<br />

No No Yes Yes Yes Yes<br />

Diagonals bisect<br />

each other No No Yes Yes Yes Yes<br />

Diagonals are<br />

perpendicular Yes No No Yes No Yes<br />

Diagonals are<br />

congruent No Yes No No Yes Yes<br />

Exactly one line<br />

of symmetry<br />

Yes Yes No No No No<br />

Exactly two lines<br />

of symmetry No No No Yes Yes No


20.<br />

Speed: 901.4 km/h. Direction: slightly west of<br />

north. Figure is approximate.<br />

21.<br />

E<br />

22. possible answers:<br />

F<br />

23.<br />

Resultant<br />

vector<br />

S<br />

y<br />

N E<br />

x<br />

R<br />

50 km/h<br />

Q<br />

Y<br />

L<br />

1<br />

x<br />

2<br />

L z<br />

1<br />

900 km/h<br />

F<br />

R<br />

x<br />

P<br />

ABCD is a parallelogram<br />

Given<br />

R<br />

Y<br />

L<br />

1<br />

x<br />

2<br />

2<br />

AD CB<br />

Definition of<br />

parallelogram<br />

3 AB CD<br />

Definition of<br />

parallelogram<br />

4 1 3<br />

AIA<br />

24. possible answers:<br />

F<br />

5<br />

BD BD<br />

Same segment<br />

6 2 4<br />

AIA<br />

F<br />

x<br />

25. 20 sides<br />

26. 12 cm<br />

27. See flowchart below.<br />

28. possible answer:<br />

A B<br />

4<br />

3<br />

1<br />

2<br />

D C<br />

x<br />

R<br />

Given: Parallelogram ABCD<br />

Show: AB CD and AD CB <br />

See flowchart below.<br />

2<br />

DE <br />

?<br />

1<br />

DENI is<br />

a rhombus<br />

3<br />

NE <br />

?<br />

4<br />

DN <br />

?<br />

5<br />

<br />

? <br />

?<br />

6<br />

1 <br />

?<br />

3 <br />

?<br />

7<br />

DN bisects<br />

IDE and INE<br />

?<br />

?<br />

?<br />

?<br />

?<br />

?<br />

27. (Chapter 5 Review)<br />

DI <br />

Definition of rhombus<br />

NI DEN DIN 2, 4<br />

Given<br />

Definition of<br />

rhombus<br />

DN <br />

SSS<br />

CPCTC Definition of<br />

angle bisector<br />

28. (Chapter 5 Review)<br />

?<br />

Same segment<br />

ASA<br />

R<br />

Y<br />

7 ABD CDB<br />

z<br />

Y<br />

D<br />

z<br />

D<br />

8 AB CD<br />

CPCTC<br />

9<br />

AD CB<br />

CPCTC<br />

ANSWERS TO EXERCISES 73<br />

<strong>Answers</strong> to Exercises


<strong>Answers</strong> to Exercises<br />

<strong>Answers</strong> to Exercises<br />

CHAPTER 6 • CHAPTER CHAPTER 6 • CHAPTER<br />

LESSON 6.1<br />

1. 50° 2. 55°<br />

3. 30° 4. 105°<br />

5. 76 in.<br />

6. possible answer:<br />

7. Possible answer: The perpendicular to the<br />

tangent passes through the center of the circle. Use<br />

the T-square to find two diameters of the Frisbee.<br />

The intersection of these two lines is the center.<br />

8. Construct OT . Construct a line through point T<br />

perpendicular to OT .<br />

9. Construct OX ,OY , and OZ . Construct tangents<br />

through points X, Y, and Z.<br />

X<br />

O<br />

Z<br />

r<br />

10. Construct tangent circles M and N. Construct<br />

equilateral MNP.Construct circle P with radius s.<br />

s s<br />

M N<br />

P<br />

O<br />

74 ANSWERS TO EXERCISES<br />

T<br />

t Y<br />

6<br />

Target<br />

11. Construct a line and label a point T on it. On<br />

the line, mark off two points, L and M, each on<br />

the same side of T at distances r and t from T,<br />

respectively. Construct circle L with radius r.<br />

Construct circle M with radius t.<br />

r<br />

L M T<br />

t<br />

12. Start with the construction from Exercise 11.<br />

On line LM , mark off length TK so that TK s<br />

and K is on the opposite side of T from L and M.<br />

Construct circle K with radius s.<br />

r<br />

L M T K<br />

t s<br />

13. Sample answer: If the three points do not lie<br />

on the same semicircle, the tangents form a circumscribed<br />

triangle. If two of the three points are<br />

on opposite sides of a diameter, the tangent lines to<br />

those two points are parallel. If the points lie on the<br />

same semicircle, they form a triangle outside the<br />

circle, with one side touching (called an exscribed<br />

triangle).<br />

14. sample answer: internally tangent: wheels on a<br />

roller-coaster car in a loop, one bubble inside<br />

another; externally tangent: touching coins, a<br />

snowman, a computer mouse ball and its roller<br />

balls<br />

15. Constructions will vary.<br />

16. 360° 30° 90° 90° 150° and<br />

150°<br />

<br />

360°<br />

41.6%<br />

17. Angles A and B must be right angles, but this<br />

would make the sum of the angle measures in the<br />

quadrilateral shown greater than 360°.<br />

18a. rhombus 18b. rectangle<br />

18c. kite 18d. parallelogram<br />

19. 78° 20. 9.7 km<br />

21. x 55° 55° 180° and 40° y <br />

y 180°, so x y 70°<br />

22. 11<br />

<br />

21


LESSON 6.2<br />

1. 165°, definition of measure of an arc<br />

2. 84°, Chord Arcs Conj.<br />

3. 70°, Chord Central Angles Conj.<br />

4. 8 cm, Chord Distance to Center Conj.<br />

5. mAC 68°; mB 34° (Because OBC is<br />

isosceles, mB mC, mB mC 68°,<br />

and therefore mB 34°.)<br />

6. w 115°, x 115°, y 65°; Chord Arcs<br />

Conjecture<br />

7. 20 cm, Perpendicular to a Chord Conj.<br />

8. w 110°, x 48°, y 82°, z 120°; definition<br />

of arc measure<br />

9. x 96°, Chord Arcs Conjecture; y 96°,<br />

Chord Central Angles Conjecture; z 42°,<br />

Isosceles Triangle Conjecture and Triangle Sum<br />

Conjecture.<br />

10. x 66°, y 48°, z 66°; Corresponding<br />

Angles Conjecture, Isosceles Triangle Conjecture,<br />

Linear Pair Conjecture<br />

11. The length of the chord is greater than the<br />

length of the diameter.<br />

12. The perpendicular bisector of the segment<br />

does not pass through the center of the circle.<br />

13. The longer chord is closer to the center; the<br />

longest chord, which is the diameter, passes<br />

through the center.<br />

20. (Lesson 6.2)<br />

1<br />

AB CD<br />

14. The central angle of the smaller circle is larger,<br />

because the chord is closer to the center.<br />

5 cm<br />

15. M(4, 3), N(4, 3), O(4, 3)<br />

16. The center of the circle is the circumcenter of<br />

the triangle. Possible construction:<br />

17. possible construction:<br />

O<br />

18. 13.8 cm<br />

?<br />

2<br />

AO CO<br />

4<br />

<br />

? <br />

?<br />

All radii of a circle<br />

are congruent<br />

3<br />

BO DO<br />

?<br />

?<br />

Given<br />

AOB COD<br />

SSS Congruence<br />

Conjecture<br />

All radii of a circle are congruent<br />

5 cm<br />

19. They can draw two chords and locate the<br />

intersection of their perpendicular bisectors. The<br />

radius is just over 5 km.<br />

20. See flowchart below.<br />

5<br />

AOB <br />

?<br />

?<br />

COD<br />

CPCTC<br />

ANSWERS TO EXERCISES 75<br />

<strong>Answers</strong> to Exercises


<strong>Answers</strong> to Exercises<br />

21. y 1<br />

7 x; (0, 0) is a point on this line.<br />

22a. true<br />

A<br />

C<br />

D<br />

X<br />

B<br />

Possible explanation: If AB is the perpendicular<br />

bisector of CD , then every point on AB is<br />

equidistant from endpoints C and D. Therefore<br />

AC AD and BC BD. Because CD is not the<br />

perpendicular bisector of AB, Cis not equidistant<br />

from A and B. Likewise, D is not equidistant from<br />

A and B. So,AC and BC are not congruent, and<br />

AD and BD are not congruent. Thus ACBD<br />

has exactly two pairs of consecutive congruent<br />

sides, so it is a kite.<br />

22b. false, isosceles trapezoid<br />

22c. false, rectangle<br />

23. 140°, 160°, 60°; 180° minus the measure of<br />

the intercepted arc<br />

76 ANSWERS TO EXERCISES<br />

24. Using the Tangent Conjecture, 2 and 4<br />

are right angles, so m2 m4 180°.<br />

According to the Quadrilateral Sum Conjecture,<br />

m1 m2 m3 m4 360°, so by the<br />

Subtraction Property of Equality, m1 <br />

m3 180°, or m1 180° m3. The<br />

measure of a central angle equals the measure of<br />

its arc, so by substitution, m1 180° mAB .<br />

25. the circumcenter of the triangle formed by<br />

the three light switches<br />

26. 7<br />

27. Station Beta is closer.<br />

N<br />

72<br />

38<br />

Alpha<br />

7.2 mi<br />

28. D<br />

Downed aircraft<br />

8.2 mi<br />

4.6 mi<br />

48<br />

312<br />

Beta


1 OR CD<br />

?<br />

5 OC OD<br />

?<br />

All radii of<br />

a circle are<br />

congruent<br />

LESSON 6.3<br />

1. 65° 2. 30° 3. 70° 4. 50°<br />

5. 140°, 42° 6. 90°, 100° 7. 50° 8. 148°<br />

9. 44° 10. 142° 11. 120°, 60°<br />

12. 140°, 111° 13. 71°, 41° 14. 180°<br />

15. 75°<br />

16. The two inscribed angles intercept the same<br />

arc, so they should be congruent.<br />

17. BFE DFA (Vertical Angles Conjecture).<br />

BGD FHD (all right angles congruent).<br />

Therefore, B D (Third Angle Conjecture).<br />

mB 1<br />

2 mAC ,mD 1<br />

2 mEC ,AC EC <br />

18. Possible answer: Place the corner so that it is<br />

an inscribed angle. Trace the inscribed angle. Use<br />

the side of the paper to construct the hypotenuse<br />

of the right triangle (which is the diameter). Repeat<br />

the process. The place where the two diameters<br />

intersect is the center.<br />

19. possible answer:<br />

It works on acute and right triangles.<br />

20. The camera can be placed anywhere on the<br />

major arc (measuring 268°) of a circle such that the<br />

row of students is a chord intersecting the circle to<br />

form a minor arc measuring 92°. This illustrates<br />

the conjecture that inscribed angles that intercept<br />

the same arc are congruent (Inscribed Angles<br />

Intercepting Arcs Conjecture).<br />

21. two congruent externally tangent circles with<br />

half the diameter of the original circle<br />

24. (Lesson 6.3)<br />

Given<br />

<br />

6<br />

OCD is isosceles<br />

?<br />

2<br />

ORC and ORD<br />

are right angles<br />

Definition of<br />

perpendicular lines<br />

?<br />

Definition of<br />

isosceles triangle<br />

22. a 108°; b 72°; c 36°; d 108°;<br />

e 108°; f 72°; g 108°; h 90°; l 36°;<br />

m 18°; n 54°; p 36°<br />

23. (2, 1); Possible method: Plot the<br />

three points. Construct the midpoint and the<br />

perpendicular bisector of the segments connecting<br />

two different pairs of points. The center is the point<br />

of intersection of the two lines. To check, construct<br />

the circle through the three given points.<br />

24. See flowchart below.<br />

25. Start with an equilateral triangle whose vertices<br />

are the centers of the three congruent circles. Then<br />

locate the incenter/circumcenter/orthocenter/<br />

centroid (all the same point because the triangle is<br />

equilateral) to find the center of the larger circle. To<br />

find the radius, construct a segment from the<br />

incenter of the triangle through the vertex of the<br />

triangle to a point on the circle.<br />

26. mA 60°, mB 36°, mC 90°;<br />

60° 36° 90° 180°<br />

3 mORC 90<br />

mORD 90<br />

?<br />

7 C ?<br />

?<br />

Definition of<br />

right angle<br />

D<br />

7. Isosceles Triangle<br />

Conjecture<br />

4 mORC mORD<br />

(ORC ORD)<br />

?<br />

8 OCR ?<br />

?<br />

9 CR ?<br />

?<br />

10 OR bisects CD<br />

?<br />

Substitution<br />

property<br />

SAA<br />

DR<br />

CPCTC<br />

ODR<br />

Definition of bisect<br />

ANSWERS TO EXERCISES 77<br />

<strong>Answers</strong> to Exercises


<strong>Answers</strong> to Exercises<br />

LESSON 6.4<br />

1. Proof: mACD 1<br />

2 mAD mABD by the<br />

Inscribed Angle Conjecture. ACD ABD.<br />

2. Proof: By the Inscribed Angle Conjecture,<br />

mACB 1<br />

2 mAD B 1 (180°) 2 90°. ACB is a<br />

right angle.<br />

3. Proof: By the Inscribed Angle Conjecture,<br />

mC 1<br />

2 mYLI and mL 1<br />

2 mYCI 1 (360° 2 <br />

mYLI<br />

) 180° 1<br />

2 mYLI 180° mC. L<br />

and C are supplementary. (A similar proof can be<br />

used to show that I and Y are supplementary.)<br />

4. Proof: 1 2 by AIA. mBC 2m2 <br />

2m1 mAD by the Inscribed Angle Conjecture.<br />

BC AD .<br />

5. True. Opposite angles of a parallelogram are<br />

congruent. If it is inscribed in a circle, the opposite<br />

angles are also supplementary. So they are right<br />

angles, and the parallelogram is indeed<br />

equiangular, or a rectangle.<br />

6. Construct diagonals RG and DB. Diagonal RG<br />

is the perpendicular bisector of BD by the Kite<br />

Diagonal Bisector Conjecture. Because kite BRDG<br />

is inscribed in the circle, BD and RG are chords of<br />

the circle. By the Perpendicular Bisector of a Chord<br />

Conjecture, the perpendicular bisector of BD<br />

passes through the center of the circle. Because RG<br />

is a chord of the circle that passes through the<br />

center, by the definition of diameter, RG is a<br />

diameter.<br />

7. Draw in radii GR,ER,TR, and AR. Because they<br />

are radii of the same circle, GR ER and AR TR.<br />

By the Parallel Lines Intercepted Arcs Conjecture,<br />

GA ET . Their central angles must also be<br />

congruent, so GRA ERT.Thus GRA <br />

ERT by SAS, so GA ET by CPCTC. GATE is<br />

an isosceles trapezoid.<br />

8. x 65°, y 40°, z 148°; half the measure of<br />

the intercepted arc<br />

9. Because the radii of a circle are congruent,<br />

OA OB, so OAB is isosceles. By the Isosceles<br />

Triangle Conjecture, 2 3, so m2 m3.<br />

By the Triangle Sum Conjecture, m1 m2 <br />

m3 180°, so by substitution, m1 <br />

2(m2) 180°, or m1 180° 2(m2).<br />

78 ANSWERS TO EXERCISES<br />

By the Tangent Conjecture, OB BC , so<br />

mOBC 90°. By Angle Addition, m2 <br />

m4 mOBC, so m2 m4 90°,<br />

or m2 90° m4. Substitute 90° m4<br />

for m2 in the earlier equation: m1 180° <br />

2(90° m4). Simplifying gives m1 2(m4).<br />

Because mAB m1, substitution gives mAB <br />

2(m4), or m4 1<br />

2 mAB .<br />

10a. S; An equilateral triangle is equiangular, but a<br />

rhombus is not equiangular.<br />

10b. A<br />

10c. N; Only one diagonal is the perpendicular<br />

bisector of the other.<br />

10d. A<br />

10e. S; An equilateral triangle has rotational<br />

symmetry and three lines of symmetry; a<br />

parallelogram has rotational symmetry but no line<br />

of symmetry.<br />

11. L<br />

12. J<br />

13. K<br />

14. D<br />

15. E<br />

16. B<br />

17. H<br />

18. G<br />

19. N<br />

20. a b b a 180°, so a b 90°<br />

2<br />

21. 21<br />

22. It is given that PQ RS. OP OQ OR <br />

OS because all radii in a circle are congruent.<br />

Therefore, OPQ ORS by SSS and 2 1<br />

by CPCTC. Now we move on to the smaller right<br />

triangles inside OPQ and ORS.It is given that<br />

OT PQ and OV RS. Therefore, OTQ and<br />

OVS are right angles by the definition of<br />

perpendicular lines and OTQ OVS because<br />

all right angles are congruent. Thus, OTQ <br />

OVS by SAA and OT OV by CPCTC.


LESSON 6.5<br />

1. d 5 cm<br />

2. C 10 cm<br />

3. r 12<br />

<br />

<br />

m<br />

4. C 5.5 or 11<br />

<br />

2<br />

5. C 12 cm<br />

6. d 46 m<br />

7. C 15.7 cm<br />

8. C 25.1 cm<br />

9. r 7.0 m<br />

10. C 84.8 in.<br />

11. 565 ft<br />

12. C 6 cm<br />

13. 16 in.<br />

14. Trees grow more in years with more rain;<br />

244 yr<br />

18. (Lesson 6.5)<br />

1<br />

2<br />

? MA MT <br />

Given<br />

3 MAST is a kite<br />

15. 1399 tiles<br />

16. g 40°, n 30°, x 70°; y 142°; z 110°;<br />

Conjecture: The measure of the angle formed by<br />

two intersecting chords is one-half the sum of the<br />

measures of the two intercepted arcs. In the<br />

diagrams, mAEN 1<br />

2 mAN mLG .<br />

17. mAGN 1<br />

2 mAN and mLNG 1<br />

2 mLG<br />

by the Inscribed Angle Conjecture. mAEN <br />

mAGN mLNG bytheTriangleExterior<br />

Angle Conjecture. So, mAEN 1<br />

2 mAN mLG <br />

by substitution and the distributive property.<br />

18. See flowchart below.<br />

19. b 90°, c 42°, d 70°, e 48°, f 132°,<br />

g 52°<br />

20. 150°<br />

5<br />

<br />

360°<br />

or 12<br />

21. The base angles of the isosceles triangle have a<br />

measure of 39°. Because the corresponding angles<br />

are congruent, m is parallel to n.<br />

22. 10x 2y<br />

4<br />

?<br />

MS is the perpendicular bisector of AT<br />

?<br />

Given<br />

?<br />

?<br />

SA ST Definition of kite Kite Diagonal Bisector Conjecture<br />

ANSWERS TO EXERCISES 79<br />

<strong>Answers</strong> to Exercises


<strong>Answers</strong> to Exercises<br />

LESSON 6.6<br />

1. 4398 km/h<br />

2. 11 m/s<br />

3. 37,000,000 revolutions<br />

4. 637 revolutions<br />

5. Mama; C 50 in.<br />

6. 168 cm<br />

7. d 7.6 ft. The table will fit, but the chairs may<br />

be a little tight in a 12-by-14 ft room. 12 chairs <br />

192 in., 12 spaces 96 in., C 288 in., d 91.7 in.<br />

7.6 ft.<br />

8. 0.35 ft/s<br />

9. a 37.5°, s 17.5°, x 20°; y 80°; z 61°;<br />

Conjecture: The measure of an angle formed by<br />

two secants that intersect outside a circle is<br />

one-half the difference of the larger arc measure<br />

and the smaller arc measure.<br />

80 ANSWERS TO EXERCISES<br />

10. mESA 1<br />

2 mEA and mSAN 1<br />

2 mSN by<br />

the Inscribed Angle Conjecture. mSAN <br />

mESA mECA by the Triangle Exterior Angle<br />

Conjecture. So, 1<br />

2 mSN 1<br />

2 mEA mECA by<br />

substitution. With a little more algebra, mECA <br />

1<br />

2 mSN mEA .<br />

11. Both triangles are isosceles, so the base angles<br />

in each triangle are congruent. But one of each base<br />

angle is part of a vertical pair. So, a b by the<br />

Vertical Angles Conjecture and transitivity.<br />

12. C<br />

13. 38°<br />

14. 48°<br />

15. 30 cm<br />

16. 400<br />

rev<br />

<br />

1 min<br />

2 . 26<br />

ft<br />

<br />

1 rev<br />

1 min<br />

<br />

60<br />

s<br />

1089 ft/s<br />

17. 12 cm third side 60 cm. This is based on<br />

the Triangle Inequality Conjecture.


1. 1<br />

2 ,3<br />

USING YOUR ALGEBRA SKILLS 6<br />

2. (3, 3)<br />

3. 2 5 ,3<br />

4. (7, 2)<br />

5. infinitely many solutions<br />

6. no solution<br />

7. 4. The lines intersect at the solution point, (7,2).<br />

5<br />

–5<br />

–10<br />

5. The lines are the same. There are infinitely many<br />

solutions.<br />

4<br />

y<br />

y<br />

y 2x 16<br />

5<br />

( 7, 2)<br />

y <br />

1<br />

3<br />

x 2<br />

x<br />

10<br />

y <br />

1<br />

2<br />

x <br />

3<br />

2<br />

6. The lines are parallel (the slopes are the same,<br />

but the y-intercepts are different).<br />

There is no solution.<br />

y<br />

–10<br />

y = –2x + 9<br />

8. Possible solution using the equations<br />

y 1<br />

4<br />

x 2 1 and y 2x<br />

3 1 : 3<br />

y 1<br />

x<br />

2<br />

1<br />

2 4<br />

x<br />

3<br />

1 <br />

3<br />

1<br />

x<br />

2<br />

1<br />

4x 28 3x 6<br />

22 7x<br />

22<br />

<br />

7<br />

x<br />

y 1 2<br />

2 2 7<br />

1<br />

y 18<br />

<br />

7<br />

x<br />

–5<br />

5<br />

–5<br />

x<br />

y = –2x – 1<br />

The circumcenter is 22<br />

, <br />

7<br />

18<br />

. 7 Only two<br />

perpendicular bisectors are needed because<br />

the third bisector intersects at the same point.<br />

9a. Plan A: y 4x 20; Plan B: y 7x;<br />

x 6 h 40 min, y $46.67<br />

9b. y<br />

Cost ($)<br />

50<br />

40<br />

30<br />

20<br />

10<br />

(0, 20)<br />

The point of intersection shows when both plans<br />

cost the same, the answer for 9a.<br />

9c. Plan B; 7 1<br />

<br />

2 hours, with Plan A<br />

10. (3, 1), (0, 3), (3, 1)<br />

11a. <br />

11b. (6, 6)<br />

11c. (6, 6); (6, 6)<br />

11d. The diagonals intersect at their midpoints,<br />

which supports the conjecture that the diagonals of<br />

a parallelogram bisect each other.<br />

12. y 2 3<br />

x<br />

3<br />

1 <br />

3<br />

13. (4, 7)<br />

14. 69<br />

,<br />

14<br />

6<br />

7 <br />

y 3<br />

x<br />

4<br />

3<br />

2 <br />

y x 12<br />

15. 3, 3<br />

2 <br />

Plan A<br />

Plan B<br />

2 4 6<br />

Hours<br />

6 2_ , 46<br />

2_<br />

<br />

3<br />

16. The circumcenter is the midpoint of the<br />

hypotenuse. For a right triangle, the perpendicular<br />

bisectors of the legs are two of the midsegments<br />

of the triangle. As with any pair of triangle<br />

midsegments, they intersect at the midpoint of<br />

the third side.<br />

x<br />

3<br />

ANSWERS TO EXERCISES 81<br />

<strong>Answers</strong> to Exercises


<strong>Answers</strong> to Exercises<br />

LESSON 6.7<br />

1. 4<br />

<br />

3<br />

in. 2. 8 m 3. 14 cm<br />

4. 9 m 5. 6 ft 6. 4 m<br />

7. 27 in. 8. 100 cm 9. 217 m/min<br />

10. 4200 mi<br />

11. Desks are about 17 meters from the center.<br />

About four desks will fit because an arc with<br />

one-half the radius and the same central angle<br />

will be one-half as long as the outer arc.<br />

12. The measure of the central angle is 7.2°<br />

because of the Corresponding Angles Conjecture.<br />

7.2<br />

Therefore, 500 360<br />

C, so C 25,000 mi.<br />

13. 18°/s. No, the angular velocity is measured in<br />

degrees per second not in distance per second, so it<br />

is the same at every point on the carousel.<br />

14. Outer horse 2.5 m/s, inner horse 1.9 m/s.<br />

One horse has traveled farther in the same amount of<br />

time (tangential velocity), but both horses have<br />

rotated the same number of times (angular velocity).<br />

15. a 50°, b 75°, x 25°; y 45°; z 35°;<br />

Conjecture: The measure of the angle formed by<br />

an intersecting tangent and secant to a circle is<br />

one-half the difference of the larger intercepted arc<br />

measure and the smaller intercepted arc measure.<br />

16. Let z represent the measure of the exterior<br />

angle of PBA formed by AB and tangent PB .<br />

By the Tangent Chord Conjecture, z 1<br />

2 mAB .By<br />

the Inscribed Angle Conjecture, mBAP 1<br />

2 mBC .<br />

By the Triangle Exterior Angle Conjecture, z <br />

mBPA mBAP, or mBPA z mBAP.<br />

82 ANSWERS TO EXERCISES<br />

By substitution, mBPA 1<br />

2 mAB 1<br />

2 mBC .<br />

Using the distributive property, mBPA <br />

1<br />

2 (mAB mBC ).<br />

17. a 70°, b 110°, c 110°, d 70°, e 20°,<br />

f 20°, g 90°, h 70°, k 20°, m 20°,<br />

n 20°, p 140°, r 80°, s 100°, t 80°,<br />

u 120°<br />

18. possible answer:<br />

19. 170°<br />

8<br />

20. y x 15<br />

289<br />

<br />

15<br />

21. 45°<br />

22. The sum of the lengths is 8 cm for Case 1,<br />

Case 2, Case 3, and Case 10.<br />

23. 6<br />

24. Yes, as long as the three points are noncollinear;<br />

possible answer: connect the points with<br />

segments, then find the point of concurrency of<br />

the perpendicular bisectors (same as circumcenter<br />

construction).


CHAPTER 6 REVIEW<br />

1. <strong>Answers</strong> will vary.<br />

2. Draw two nonparallel chords. The intersection<br />

of their perpendicular bisectors is the center of the<br />

circle.<br />

Fold the paper so that two semicircles coincide.<br />

Repeat with two different semicircles. The center is<br />

the intersection of the two folds.<br />

Place the outside or inside corner of the L in the<br />

circle so that it is an inscribed right angle. Trace the<br />

sidesofthecorner.Drawthehypotenuseoftheright<br />

triangle (which is the diameter of the circle). Repeat.<br />

The center is the intersection of the two diameters.<br />

3. The velocity vector is always perpendicular to<br />

the radius at the point of tangency to the object’s<br />

circular path.<br />

4. Sample answer: An arc measure is between 0° and<br />

360°.An arc length is proportional to arc measure<br />

and depends on the radius of the circle.<br />

5. 55° 6. 65° 7. 128° 8. 118°<br />

9. 91° 10. 66° 11. 125.7 cm 12. 42.0 cm<br />

13. 15 cm<br />

14. 14 ft<br />

15. 2 57° 2 35° 180°<br />

16. 84° 56° 56° 158° 360°<br />

17. mEKL 1<br />

2 mEL 1<br />

<br />

2 (180° 108°) 36° <br />

mKLY. KE YL by Converse of the Parallel<br />

Lines Conjecture.<br />

18. mJI 360° 56° 152° 152° mMI<br />

.<br />

mJMI 1<br />

2 mJI 1<br />

<br />

2 mMI<br />

mMJI.By the<br />

Converse of the Isosceles Triangle Conjecture,<br />

JIM is isosceles.<br />

19. mKIM 2mKEM 140°. mKI<br />

<br />

140° 70° 70° mMI<br />

. mIKM <br />

1<br />

<br />

2 mMI<br />

1<br />

<br />

2 mKI<br />

mIMK. By the Converse of the<br />

Isosceles Triangle Conjecture, KIM is isosceles.<br />

20. Ertha can trace the incomplete circle on paper.<br />

She can lay the corner of the pad on the circle to<br />

trace an inscribed right angle. Then Ertha should<br />

mark the endpoints of the intercepted arc and use<br />

the pad to construct the hypotenuse of the right<br />

triangle, which is the diameter of the circle.<br />

21. Sample answer: Construct<br />

perpendicular bisectors of two<br />

sides of the triangle. The point<br />

at which they intersect (the<br />

circumcenter) is the center of<br />

the circle. The distance from<br />

the circumcenter to each vertex<br />

is the radius.<br />

22. Sample answer: Construct the incenter (from<br />

the angle bisectors) of the triangle. From the incenter,<br />

which is the center of the circle, construct a perpendicular<br />

to a side. The distance from the incenter to<br />

the foot of the perpendicular is the radius.<br />

23. Sample answer: Construct a right angle and<br />

label the vertex R.Mark offRE and RT with any<br />

lengths. From point E, swing an arc with radius RT.<br />

From point T, swing an arc with radius RE. Label<br />

the intersection of the arcs as C. Construct the<br />

diagonals ET and RC. Their intersection is the<br />

center of the circumscribed circle. The circle’s<br />

radius is the distance from the center to a vertex. It<br />

is not possible to construct an inscribed circle in a<br />

rectangle unless it is a square.<br />

24. Sample answer: Construct acute angle R.Mark<br />

off equal lengths RM and RH.FrompointsM and<br />

H, swing arcs of lengths equal to RM. Label the<br />

intersection of the arcs as O. Construct RHOM. The<br />

intersection of the diagonals is the center of the inscribed<br />

circle. Construct a perpendicular to a side to<br />

find the radius. It is not possible to construct a circumscribed<br />

circle unless the rhombus is a square.<br />

R<br />

E C<br />

R<br />

H<br />

25. 4x 3y 32, or y 4 2<br />

x<br />

3<br />

3 <br />

3<br />

26. (3, 2) 27. d 0.318 m<br />

28. Melanie: 151 m/min or 9 km/h; Melody:<br />

94 m/min or 6 km/h.<br />

2(6357)<br />

29. 1.849 1.852 1.855 <br />

360 60<br />

30. Possible<br />

location<br />

5500 ft<br />

Possible<br />

location<br />

M<br />

5280 ft<br />

D T<br />

T<br />

O<br />

7700 ft<br />

2(6378)<br />

360 60<br />

ANSWERS TO EXERCISES 83<br />

<strong>Answers</strong> to Exercises


<strong>Answers</strong> to Exercises<br />

31. 200<br />

<br />

<br />

ft 63.7 ft 32. 8 m 25.1 m<br />

48<br />

33. The circumference is 360<br />

2(45) 12;the<br />

diameter is 12 cm.<br />

34. False. 20° 20° 140° 180°. An angle with<br />

measure 140° is obtuse.<br />

35. true<br />

36. false<br />

D<br />

37. true 38. true 39. true 40. true<br />

41. False. (7 2) 180° 900°. It could have seven<br />

sides.<br />

42. False. The sum of the measures of any triangle<br />

is 180°.<br />

43. False. The sum of the measures of one set of<br />

exterior angles for any polygon is 360°. The sum of<br />

the measures of the interior angles of a triangle is<br />

180° and of a quadrilateral is 360°. Neither is greater<br />

than 360°, so these are two counterexamples.<br />

44. False. The consecutive angles between the<br />

bases are supplementary.<br />

45. False. 48° 48° 132° 180°<br />

46. False. Inscribed angles that intercept the same<br />

arc are congruent.<br />

47. False. The measure of an inscribed angle is<br />

half the measure of the arc.<br />

48. true<br />

49. False. AC and BD bisect each other, but AC is<br />

not perpendicular to BD.<br />

D<br />

A B<br />

50. False. It could be isosceles.<br />

51. False. 100° 100° 100° 60° 360°<br />

52. false; AB CD <br />

A<br />

A<br />

90<br />

84 ANSWERS TO EXERCISES<br />

B<br />

C<br />

B<br />

C<br />

90<br />

65. (Chapter 6 Review)<br />

D<br />

C<br />

53. False. The ratio of the circumference to the<br />

diameter is .<br />

54. false; 24 24 <br />

48 48 96<br />

24 cm 24 cm<br />

55. true 56. This is a paradox.<br />

57. a 58°,b 61°,c 58°,d 122°,e 58°,<br />

f 64°,g 116°,h 52°,i 64°,k 64°,<br />

l 105°,m 105°,n 105°,p 75°,q 116°,<br />

r 90°,s 58°,t 122°,u 105°,v 75°,<br />

w 61°,x 29°,y 151°<br />

58. TAR YRA by SAS, TAE YRE<br />

by SAA<br />

59. FTO YTO by SAA, SAS, or SSS;<br />

FLO YLO by SAA, SAS, or SSS;<br />

FTL YTL by SSS, SAS, or ASA<br />

60. PTR ART by SSS or SAS;<br />

TPA RAP by SSS, SAS, SAA, or ASA;<br />

TLP RLA by SAA or ASA<br />

61. ASA<br />

62. mAC 84°, length of AC 11.2 35.2 in.<br />

63. x 63°, y 27°, w 126°<br />

64. sample answer:<br />

65. See table below.<br />

66a. The circle with its contents has 3-fold rotational<br />

symmetry, the entire tile does not.<br />

66b. No, it does not have reflectional symmetry.<br />

67. 68. 9.375 cm<br />

69. 70.<br />

n 1 2 3 4 5 6 . . . n ... 20<br />

f(n) 5 1 3 7 11 15 ... 9 4n ... 71<br />

48 cm 48 cm<br />

150<br />

30


<strong>Answers</strong> to Exercises<br />

CHAPTER 7 • CHAPTER CHAPTER 7 • CHAPTER<br />

LESSON 7.1<br />

1. Rigid; reflected, but the size and shape do not<br />

change.<br />

2. Nonrigid; the shape changes.<br />

3. Nonrigid; the size changes.<br />

4. 5.<br />

6.<br />

<br />

7<br />

7. possible answer: a boat moving across the water<br />

8. possible answer: a Ferris wheel<br />

9a. Sample answer: Fold the paper so that the<br />

images coincide, and crease.<br />

9b. Construct a segment that connects<br />

two corresponding points. Construct the<br />

perpendicular bisector of that segment.<br />

10a. Extend the three horizontal segments onto<br />

the other side of the reflection line. Use your<br />

compass to measure lengths of segments and<br />

distances from the reflection line.<br />

10b.<br />

P<br />

16. (Lesson 7.1)<br />

11.<br />

12. reflectional symmetry<br />

13. 4-fold rotational and reflectional symmetry<br />

14. reflectional symmetry<br />

15. 7-fold symmetry: possible answers are F or J.<br />

9-fold symmetry: possible answers are E or H.<br />

Basket K has 3-fold rotational symmetry but not<br />

reflectional symmetry.<br />

16. See table below; n, n<br />

17.<br />

18.<br />

, or<br />

19. P(a, b), Q(a, b), R(a, b)<br />

20. possible construction:<br />

P<br />

21. 50th figure: 154 (50 shaded, 104 unshaded);<br />

nth figure: 3n 4 (n shaded, 2(n 2) unshaded)<br />

22. 46<br />

23. It is given that 1 2, and 2 3<br />

because of the Vertical Angles Conjecture, so<br />

1 3. Segment DC is congruent to itself.<br />

DCE and DCB are both right angles, so they<br />

are congruent. Therefore, DCB DCE by<br />

ASA, and BC CE by CPCTC.<br />

Number of sides of<br />

regular polygon<br />

3 4 5 6 7 8 . . . n<br />

Number of reflectional<br />

symmetries<br />

3 4 5 6 7 8 ... n<br />

Number of rotational<br />

symmetries ( 360°)<br />

3 4 5 6 7 8 ... n<br />

P<br />

ANSWERS TO EXERCISES 85<br />

<strong>Answers</strong> to Exercises


<strong>Answers</strong> to Exercises<br />

1.<br />

2. reflection<br />

3.<br />

4.<br />

5.<br />

–5<br />

–4<br />

5<br />

5<br />

y<br />

8<br />

–8<br />

y<br />

rotation<br />

y<br />

translation<br />

5<br />

–5<br />

y<br />

x<br />

reflection<br />

5<br />

LESSON 7.2<br />

86 ANSWERS TO EXERCISES<br />

y<br />

5<br />

reflection<br />

5<br />

5<br />

7<br />

x<br />

x<br />

x<br />

x<br />

6. Rules that involve x or y changing signs,<br />

or switching places, produce reflections.<br />

If both x and y change signs, the rule produces a<br />

rotation. Rules that produce translations involve a<br />

constant being added to the x and/or y terms.<br />

5, 0 is the translation vector for Exercise 1.<br />

7. (x, y) → (x, y)<br />

8. (x, y) → (x, y)<br />

9.<br />

10. There are two possible points, one on the N<br />

wall and one on the W wall.<br />

11.<br />

12. by the Minimal Path Conjecture<br />

13.<br />

14.<br />

W<br />

T<br />

Proposed freeway<br />

Perry<br />

,<br />

W<br />

N<br />

W E<br />

S<br />

T<br />

N<br />

8 ball<br />

S<br />

N<br />

S<br />

H H'<br />

Mason<br />

Cue ball<br />

H''<br />

H<br />

E<br />

E


15. possible answer: HIKED<br />

16. one, unless it is equilateral, in which case it<br />

has three<br />

17. two, unless it is a square, in which case it has<br />

four<br />

18.<br />

19. sample construction:<br />

20. sample construction:<br />

21. false; possible counterexample: trapezoid<br />

with two right angles<br />

22. false; possible counterexample: isosceles<br />

trapezoid<br />

ANSWERS TO EXERCISES 87<br />

<strong>Answers</strong> to Exercises


<strong>Answers</strong> to Exercises<br />

LESSON 7.3<br />

1. 10, 10<br />

2. A 180° rotation. If the centers of rotation differ,<br />

rotate 180° and add a translation.<br />

3a. 20 cm<br />

3b. 20 cm, but in the opposite direction<br />

4a. 80° counterclockwise<br />

4b. 80° clockwise<br />

5. 180°<br />

6. 3 cm<br />

7. possible answer:<br />

8. possible answer:<br />

N<br />

9.<br />

O A<br />

H<br />

10. Two reflections across intersecting lines yield<br />

a rotation. The measure of the angle of rotation is<br />

twice the measure of the angle between the lines of<br />

reflection, or twice 90°, or 180°.<br />

88 ANSWERS TO EXERCISES<br />

O′ A′<br />

H′<br />

N′<br />

H′′<br />

O′′ A′′<br />

N′′<br />

Center of rotation<br />

11. <strong>Answers</strong> may vary. Possible answer: reflection<br />

across the figure’s horizontal axis and 60°<br />

clockwise rotation.<br />

12.<br />

13.<br />

,<br />

14. Sample answer: Draw a figure on an overhead<br />

transparency and then project the image onto a<br />

screen.<br />

15. possible answers: rotational: playing card,<br />

ceiling fan, propeller blade; reflectional: human<br />

body, backpack<br />

16. one: yes; two: no; three: yes<br />

17. possible answer:<br />

A O B<br />

18a. <br />

14 11<br />

0 <br />

?<br />

?<br />

5 11<br />

18b.<br />

a b<br />

d e<br />

12<br />

? ?<br />

?<br />

c a 2b 3c<br />

f<br />

20<br />

d<br />

9 0<br />

?<br />

?<br />

12 7<br />

d–e<br />

0<br />

13<br />

2a 3b 4c<br />

?<br />

?<br />

?<br />

0 d f


LESSON 7.4<br />

1. <strong>Answers</strong> will vary. 2. <strong>Answers</strong> will vary.<br />

3. 3 3 .4 2 4. 3 4 .6<br />

5. 3 2 .4.3.4 6. 3.4.6.43.4 2 .6<br />

7. 3 3 .4 2 3 2 .4.3.4 8. 3 6 3 2 .4.12<br />

9a. The dual of a square tessellation is a square<br />

tessellation.<br />

9b. The dual of a hexagon tessellation is a triangle<br />

tessellation.<br />

9c. If tessellation A is the dual of tessellation B,<br />

then tessellation B is the dual of tessellation A.<br />

10. The dual is a 3 4 3 8 tessellation of isosceles<br />

right triangles.<br />

11.<br />

12.<br />

13. A ring of ten pentagons fits around a decagon,<br />

and another decagon can fit into any two of the<br />

pentagons. But another ring of pentagons around<br />

the second decagon doesn’t leave room for a third<br />

decagon.<br />

14.<br />

15. <strong>Answers</strong> will vary.<br />

16. y 1<br />

x<br />

2<br />

4<br />

–3<br />

17. possible answer: TOT<br />

18.<br />

W<br />

4<br />

–6<br />

y<br />

8-ball<br />

5<br />

x<br />

N<br />

Cue ball<br />

S<br />

E<br />

ANSWERS TO EXERCISES 89<br />

<strong>Answers</strong> to Exercises


<strong>Answers</strong> to Exercises<br />

LESSON 7.5<br />

1. <strong>Answers</strong> will vary.<br />

2. The dual is a 5 3 5 4 tessellation.<br />

3.<br />

4. Yes. The four angles of the quadrilateral<br />

will be around each point of intersection in the<br />

tessellation.<br />

5. c a c a<br />

b<br />

b<br />

b<br />

a<br />

c<br />

c b a<br />

a c<br />

b<br />

c b a<br />

a c<br />

b<br />

c<br />

a<br />

b a c b a c b<br />

90 ANSWERS TO EXERCISES<br />

By the Triangle Sum Conjecture, a b c 180°.<br />

Around each point, we have 2(a b c) <br />

2 180° 360°. Therefore, a triangle will fill the<br />

plane edge to edge without gaps or overlaps. Thus, a<br />

triangle can be used to create a monohedral tiling.<br />

6. three ways<br />

7.<br />

8. y 2x 3<br />

8<br />

–2<br />

y<br />

5<br />

x


LESSON 7.6<br />

1. <strong>Answers</strong> will vary.<br />

2. <strong>Answers</strong> will vary.<br />

3. <strong>Answers</strong> will vary.<br />

4. regular hexagons<br />

5. squares or parallelograms<br />

6. squares or parallelograms<br />

7.<br />

8.<br />

9. <strong>Answers</strong> will vary.<br />

10. <strong>Answers</strong> will vary.<br />

11.<br />

A<br />

S<br />

B<br />

E<br />

12. y 2<br />

x<br />

3<br />

3; the slope is the opposite sign.<br />

–10<br />

5<br />

y<br />

13. 3.4.6.44.6.12<br />

14. 440<br />

rev<br />

<br />

1 min<br />

1<br />

2 28 ft min<br />

1290 ft/s<br />

1 rev 60<br />

s<br />

15. Possible explanations:<br />

15a. true; The kite diagonal between vertex angles<br />

is the perpendicular bisector of the other diagonal;<br />

in a square, diagonals would bisect each other<br />

15b. False; it could be an isosceles trapezoid.<br />

15c. False; it could be a rectangle.<br />

15d. true; Parallel lines cut off congruent arcs of a<br />

circle, so inscribed angles (the base angles of the<br />

trapezoid) are congruent.<br />

10<br />

x<br />

ANSWERS TO EXERCISES 91<br />

<strong>Answers</strong> to Exercises


<strong>Answers</strong> to Exercises<br />

1. equilateral triangles.<br />

2. regular hexagons.<br />

3.<br />

4.<br />

5. <strong>Answers</strong> will vary.<br />

6. <strong>Answers</strong> will vary.<br />

7. sample design:<br />

LESSON 7.7<br />

8. False; they must bisect each other in a<br />

parallelogram.<br />

92 ANSWERS TO EXERCISES<br />

9. true<br />

10. true<br />

11. False; it could be a kite or an isosceles<br />

trapezoid.<br />

12. The path would be 1<br />

<br />

4 of Earth’s circumference,<br />

approximately 6280 miles, which will take<br />

126 hours, or around 5 1<br />

<br />

4 days.<br />

13a. Using the Reflection Line Conjecture, the<br />

line of reflection is the perpendicular bisector of<br />

AAand BB. Because these segments are both<br />

perpendicular to the reflection line, they are<br />

parallel to each other. Note that if AB is parallel to<br />

the reflection line, quadrilateral AABB will be a<br />

rectangle instead of a trapezoid.<br />

13b. Yes; it has reflectional symmetry, so legs and<br />

base angles are congruent.<br />

13c. greatest: near each of the acute vertices;<br />

least: at the intersection of the diagonals (where A,<br />

C, and B become collinear and A,C,and B<br />

become collinear)<br />

14a. <br />

?<br />

?<br />

3 5 6<br />

6 0<br />

1<br />

0<br />

14b. 13 30<br />

<br />

?<br />

2<br />

8 3<br />

12<br />

4<br />

8<br />

9<br />

0<br />

7<br />

2<br />

?<br />

? 5<br />

1 10<br />

108 9<br />

?<br />

?<br />

28 15<br />

?<br />

29<br />

50


1. parallelograms<br />

2. parallelograms<br />

3.<br />

4.<br />

LESSON 7.8<br />

5. <strong>Answers</strong> will vary.<br />

6. <strong>Answers</strong> will vary.<br />

7. Circumcenter is (3, 4); orthocenter is (10, 8).<br />

8.<br />

9.<br />

10.<br />

ANSWERS TO EXERCISES 93<br />

<strong>Answers</strong> to Exercises


<strong>Answers</strong> to Exercises<br />

USING YOUR ALGEBRA SKILLS 7<br />

1. y 1<br />

6 x<br />

2. y 2x 2<br />

3. Centroid is 2, 2<br />

3 ;orthocenter is (0,5).<br />

94 ANSWERS TO EXERCISES<br />

4. Centroid is (4, 0); orthocenter is (3, 0).<br />

5. 1, 4<br />

3 <br />

6. (1, 1)<br />

7. (5, 8)


CHAPTER 7 REVIEW<br />

1. true 2. true<br />

3. true 4. true<br />

5. true 6. true<br />

7. False; a regular pentagon does not create a<br />

monohedral tessellation and a regular hexagon<br />

does.<br />

8. true 9. true<br />

10. False; two counterexamples are given in<br />

Lesson 7.5.<br />

11. False; any hexagon with one pair of opposite<br />

sides parallel and congruent will create a<br />

monohedral tessellation.<br />

12. This statement can be both true and false.<br />

13. 6-fold rotational symmetry<br />

14. translational symmetry<br />

15. Reflectional; color arrangements will vary, but<br />

the white candle must be in the middle.<br />

16. The two towers are not the reflection (or<br />

even the translation) of each other. Each tower<br />

individually has bilateral symmetry. The center<br />

portion has bilateral symmetry.<br />

17. <strong>Answers</strong> will vary.<br />

18. <strong>Answers</strong> will vary.<br />

19. 3 6 3 2 .4.3.4; 2-uniform<br />

20. 4.8 2 ; semiregular<br />

21. y 1<br />

2 x<br />

y<br />

x<br />

22.<br />

T H<br />

23. Use a grid of squares. Tessellate by translation.<br />

24. Use a grid of equilateral triangles. Tessellate by<br />

rotation.<br />

25. Use a grid of parallelograms. Tessellate by<br />

glide reflection.<br />

26. Yes. It is a glide reflection for one pair of sides<br />

and midpoint rotation for the other two sides.<br />

27. No.Because the shape is suitable for glide<br />

reflection,the rows of parallelograms should<br />

alternatethedirectioninwhichtheylean(row1<br />

leans right,row 2 leans left,row 3 leans right,and<br />

so on).<br />

28.<br />

ANSWERS TO EXERCISES 95<br />

<strong>Answers</strong> to Exercises


<strong>Answers</strong> to Exercises<br />

<strong>Answers</strong> to Exercises<br />

CHAPTER 8 • CHAPTER CHAPTER 8 • CHAPTER<br />

1. 228 m 2<br />

2. 41.85 cm 2<br />

3. 8 yd<br />

4. 21 cm<br />

5. 91 ft 2<br />

6. 182 m 2<br />

7. 96 in 2<br />

8. 210 cm<br />

9. A 42 ft 2<br />

10. sample answer:<br />

4 cm<br />

11. 3 square units<br />

12. 10 square units<br />

13. 7 1<br />

<br />

2 square units<br />

14. sample answers:<br />

15. possible answer:<br />

16 cm<br />

16 cm<br />

12 cm<br />

48 cm 2<br />

16 cm<br />

64 cm2 4 cm<br />

4 cm<br />

LESSON 8.1<br />

8 cm<br />

16 cm<br />

8<br />

16 cm<br />

64 cm 2<br />

16. 23.1 m2 17. 2(4)(3) 2(5.5)(3) 57 m2 18. For a constant perimeter, area is maximized by<br />

a square. 100 m 4 25 m per side; A 625 m2 .<br />

1<br />

19. <br />

530<br />

<br />

20. 112<br />

21. 96 square units<br />

22. 32 square units<br />

96 ANSWERS TO EXERCISES<br />

8 cm<br />

8 cm<br />

6 cm<br />

48 cm 2<br />

23. 500 cm2 24a. smallest: 191.88 cm2 ; largest: 194.68 cm2 24b. <strong>Answers</strong> will vary. Sample answer: about<br />

193 cm2 .<br />

24c. <strong>Answers</strong> will vary. The smallest and largest<br />

area values differ at the ones place, so the digits<br />

after the decimal point are insignificant compared<br />

to the effect of the limit of precision in the<br />

measurements.<br />

25a. In one Ohio Star block, the sum of the red<br />

patches is 36 in2 , the sum of the blue patches is<br />

72 in2 ,and the yellow patch is 36in2 .<br />

25b. 42<br />

25c. About 1814 in2 of red fabric, about 3629 in2 of blue fabric, and about 1814 in2 of yellow<br />

fabric. The border requires 5580 in2 (if it does<br />

not need the extra 20%).<br />

26. 100; 36 64. The area of the square on the<br />

longer side is the same as the sum of the areas on<br />

the other two legs.<br />

27. a 76°,b 52°,c 104°,d 52°,e 76°,<br />

f 47°, g 90°, h 43°, k 104°, m 86°.<br />

Explanations will vary.<br />

28. sample construction:<br />

A<br />

29a. 29b.<br />

29c.<br />

30°<br />

30°<br />

M


1. 20 cm 2<br />

2. 49.5 m 2<br />

3. 300 square units<br />

4. 60 cm 2<br />

5. 6 cm<br />

6. 9 ft<br />

7. 30 ft<br />

8. 5 cm<br />

9. 16 m<br />

10. 168 cm<br />

11. 12 cm<br />

12. 3.6 ft; 10.8 ft<br />

13. sample answer:<br />

12 cm<br />

9 cm 12 cm<br />

14. sample answer:<br />

4 cm<br />

15. sample answer:<br />

46 cm 35 cm<br />

12 cm<br />

45 cm 56 cm<br />

LESSON 8.2<br />

16. The length of the base of the triangle equals<br />

the sum of the lengths of both bases of the<br />

trapezoid.<br />

2 cm<br />

8 cm<br />

10 cm 9 cm<br />

12 cm<br />

9 cm<br />

7 cm<br />

7 cm<br />

3 cm 3 cm<br />

7 cm 9 cm<br />

17. 1 (To 2 see why, draw altitude PQ.)<br />

18. more than half, because the top card<br />

completely covers one corner of the bottom card<br />

19a. 86 in. of balsa wood and 960 in2 of Mylar<br />

19b. 56 in. (or less, if he tilts the kite)<br />

20. 3600 shingles (to cover an area of 900 ft2 )<br />

21. The isosceles triangle is a right triangle<br />

because the angles on either side of the right<br />

angle are complementary. If you use the trapezoid<br />

area formula, the area of the trapezoid is<br />

(a b)(a b). If you add the areas of the<br />

1<br />

2<br />

three triangles, the area of the trapezoid is<br />

1<br />

2 c2 ab.<br />

22.<br />

D<br />

A<br />

h<br />

b 1<br />

b 2<br />

Given: trapezoid ABCD with height h. area<br />

of ABD 1<br />

2 hb1 ;area ofBCD 1<br />

2 hb2 ;area<br />

of trapezoid sum of areas of two triangles<br />

1<br />

2 hb 1 b 2<br />

B<br />

C<br />

23. 11 1<br />

<br />

4 square units<br />

24. 7 square units<br />

25. 70 m<br />

26. 144 cm2 27. 828 ft2 ; 144 ft<br />

28. 1440 cm2 ; 220 cm<br />

29a. incenter<br />

29b. orthocenter<br />

29c. centroid<br />

30. a 34°, b 68°, c 68°, d 56°, e 56°,<br />

f 90°, g 34°, h 56°, m 56°, n 90°,<br />

p 34°. Possible explanation: Let O be the center<br />

of the circle. mBC 112° by the Inscribed Angle<br />

Conjecture, and d e mBC by the Central<br />

Angle Conjecture. OBA is congruent to OCA<br />

by SSS, so d e 56°. DEC<br />

is a semicircle, so<br />

mDE 68°. By the Inscribed Angle Conjecture,<br />

p 34°. Using OEC and the Triangle Sum<br />

Conjecture, n 90°.<br />

31. 32 .623.6.3.6 ANSWERS TO EXERCISES 97<br />

<strong>Answers</strong> to Exercises


<strong>Answers</strong> to Exercises<br />

LESSON 8.3<br />

1a. 121,952 ft 2<br />

1b. 244 gal of base paint and 488 gal of finishing<br />

paint<br />

2. He should buy at least four rolls of wallpaper.<br />

(The area of each roll is 125 ft 2 . The total surface<br />

area to be papered is 480 ft 2 .) If paper cut off at the<br />

corners is wasted, he’ll need 5 rolls.<br />

3. 1552 ft 2 ; 776 ft 2 more surface area<br />

4. 21<br />

5. 336 ft 2 ; $1780<br />

98 ANSWERS TO EXERCISES<br />

6. $760<br />

7. 220 terra cotta tiles, 1107 blue tiles; $1598.15<br />

8. 72 cm2 9. AB 16.5 cm, BD 15.3 cm<br />

10. 60 cm2 by either method<br />

11. Because AOB is isosceles, mA 20° and<br />

mAOB 140°. mAB 140° and mCD 82°.<br />

mAC mBD because parallel lines intercept<br />

congruent arcs on a circle. 360°1 40° 82°<br />

<br />

2 69°.<br />

12. E


USING YOUR ALGEBRA SKILLS 8<br />

1. x 2 6x 5 2. 2x 2 7x<br />

x 1 x<br />

3. 6x 2 19x 10<br />

2x 5<br />

4. (3)(2x 1) 5. (x 5)(x 3)<br />

2x 1<br />

6. (2x 3)(x 4)<br />

x 4<br />

7. x 2 26x 165 8. 12x 2 13x 35<br />

9. x 2 8x 16 10. 4x 2 25<br />

x<br />

4<br />

x<br />

11<br />

1<br />

x<br />

4<br />

x<br />

x<br />

x<br />

5<br />

x<br />

x<br />

1<br />

x 2<br />

x<br />

x 2<br />

3<br />

x<br />

4<br />

4x<br />

4x 16<br />

x 5<br />

x<br />

x 2<br />

x<br />

15<br />

15x<br />

11x 165<br />

5<br />

3x 2<br />

x x 2<br />

2x 3<br />

x 3<br />

x 3 x<br />

4x<br />

5<br />

2x<br />

5<br />

3<br />

x<br />

2x 7 x<br />

7<br />

3x<br />

12x 2<br />

2x<br />

4x 2<br />

x<br />

x 5<br />

x<br />

x 2<br />

7<br />

28x<br />

15x 35<br />

5<br />

10x<br />

10x 25<br />

5<br />

11. a 2 2ab b 2 12. a 2 b 2<br />

a<br />

b<br />

13. (x 15)(x 4) 14. (x 12)(x 2)<br />

x<br />

4<br />

15. (x 5)(x 4)<br />

x<br />

4<br />

16. (x 3) 2 (x 3)(x 3)<br />

x<br />

3<br />

17. (x 6)(x 6)<br />

x<br />

6<br />

18. (2x 7)(2x 7)<br />

2x<br />

7<br />

a<br />

a 2<br />

x<br />

x 2<br />

x<br />

x 2<br />

x<br />

x 2<br />

x<br />

x 2<br />

2x<br />

4x 2<br />

b<br />

ab<br />

ab b 2<br />

15<br />

15x<br />

4x 60<br />

3<br />

3x<br />

3x 9<br />

19. x 4 or x 1<br />

20. x 10 or x 3<br />

21. x 3 or x 8<br />

22. x 1<br />

2<br />

or x 4<br />

23a and b.<br />

h<br />

5<br />

5x<br />

4x 20<br />

6<br />

6x<br />

6x 36<br />

7<br />

14x<br />

14x 49<br />

h<br />

x<br />

2<br />

h 4<br />

12<br />

23c. 1<br />

2 [h (h 4)]h 48<br />

23d. h 8 or h 6. The height cannot be<br />

negative, so the only valid solution is h 6.<br />

The height is 6 feet, one base is 6 feet, and the<br />

otherbaseis10feet.<br />

a<br />

b<br />

x<br />

x 2<br />

a<br />

a 2<br />

b<br />

ab<br />

ab b 2<br />

12x<br />

2x 24<br />

ANSWERS TO EXERCISES 99<br />

<strong>Answers</strong> to Exercises


<strong>Answers</strong> to Exercises<br />

LESSON 8.4<br />

1. 2092 cm2 2. 74 cm<br />

3. 256 cm 4. 33 cm2 5. 63 cm 6. 490 cm2 7. 57.6 m 8. 25 ft<br />

9. 42 cm2 10. 58 cm2 11. a 1<br />

2 s; A 1<br />

2 asn 1<br />

2 1<br />

2 s s 4 s2 12. It is impossible to increase its area, because a<br />

regular pentagon maximizes the area. Any<br />

dragging of the vertices decreases the area.<br />

(Subsequent dragging to space them out more<br />

evenly can increase the area again, but never<br />

beyond that of the regular pentagon.)<br />

13. 996 cm2 14. 497 cm2 15. total surface area 13,680 in2 95 ft2 ;<br />

cost $8075<br />

16. Area is 20 square units.<br />

y<br />

y <br />

1_<br />

2<br />

x 5<br />

(2, 6)<br />

y 2x 10<br />

100 ANSWERS TO EXERCISES<br />

x<br />

17. Area is 36 square units.<br />

y<br />

y –<br />

4_ x 12<br />

3<br />

(6, 4)<br />

18. Conjecture: The three medians of a triangle<br />

divide the triangle into six triangles of equal area.<br />

Argument: Triangles 1 and 2 have equal area<br />

because they have equal bases and the same<br />

height. Because the centroid divides each median<br />

into thirds, you can show that the height of<br />

triangles 1 and 2 is 1<br />

<br />

3 the height of the whole<br />

triangle. Each has an area 1<br />

6<br />

the area of the whole<br />

triangle. By the same argument, the other small<br />

triangles also have areas 1<br />

<br />

6 the area of the whole<br />

triangle.<br />

1<br />

h<br />

y –<br />

1_ x 6<br />

3<br />

19. nw ny 2x<br />

20. 504 cm 2<br />

21. 840 cm 2<br />

2<br />

x


LESSON 8.5<br />

1. 9 in 2<br />

2. 49 cm 2<br />

3. 0.8 m 2<br />

4. 3 cm<br />

5. 3 in.<br />

6. 0.5 m<br />

7. 36 in 2<br />

8. 7846 m 2<br />

9. 25 48, or about 30.5 square units<br />

10. 100 128, or about 186 square units<br />

11.<br />

r 18 cm<br />

12. 804 m 2<br />

13. 11,310 km 2<br />

14. 154 m 2<br />

15. 4 times<br />

16. A r 2 because the 100-gon almost<br />

completely fills the circle.<br />

17. 456 cm 2<br />

18. 36 ft 2<br />

19. The triangles have equal area when the point<br />

is at the intersection of the two diagonals. There is<br />

no other location at which all four triangles have<br />

equal area.<br />

20. x mDE 2 24° 48°<br />

21. 90° 38° 28° 28° 180°<br />

22.<br />

6 cm<br />

18 cm<br />

12 cm<br />

24 cm<br />

ANSWERS TO EXERCISES 101<br />

<strong>Answers</strong> to Exercises


<strong>Answers</strong> to Exercises<br />

1. 6 cm2 2. 64<br />

<br />

3<br />

cm2 3. 192 cm2 4. ( 2) cm2 5. (48 32) cm2 6. 33 cm2 7. 21 cm2 8. 105<br />

<br />

2<br />

cm2 9. 6 cm<br />

10. 7 cm<br />

11. 75<br />

12. 100<br />

13. 42<br />

14. $448<br />

15a.<br />

15b.<br />

15c.<br />

LESSON 8.6<br />

102 ANSWERS TO EXERCISES<br />

15d.<br />

16. sample answer:<br />

17a. (144 36) cm2 ; 78.54%<br />

17b. (144 36) cm2 ; 78.54%<br />

17c. (144 36) cm2 ; 78.54%<br />

17d. (144 36) cm2 ; 78.54%<br />

18. 480 m2 19. AB 17.0 cm, AG 6.6 cm<br />

90<br />

20. True. If 24 360<br />

2r, then r 48 cm.<br />

21. True. If 360<br />

<br />

n<br />

24, then n 15.<br />

22. False. It could be a rhombus.<br />

23. true; Triangle Inequality Conjecture


LESSON 8.7<br />

1. 150 cm 2 2. 4070 cm 2<br />

3. 216 cm 2 4. 340 cm 2<br />

5. 103.7 cm 2 6. 1187.5 cm 2<br />

7. 1604.4 cm 2 8. 1040 cm 2<br />

9. 414.7 cm 2 10. 329.1 cm 2<br />

11. area of square 4 area of trapezoid 4 <br />

area of triangle<br />

12. $1570<br />

13. sample answer:<br />

14. sample tiling<br />

3 3 .4 2 /3 2 .4.3.4/4 4<br />

15. a 75°, b 75°, c 30°, d 60°,<br />

e 150°, f 30°<br />

16. About 23 days. Each sector is about 1.767 km 2 .<br />

17. a 50°, b 50°, c 80°, d 100°, e 80°,<br />

f 100°, g 80°, h 80°, k 80°, m 20°,<br />

n 80°. Explanations will vary. Sample<br />

explanation: The angle with measure d corresponds<br />

to the angle forming a linear pair with g.Because<br />

d 100°, by the Parallel Lines Conjecture, the<br />

angle adjacent to g measures 100°, and by the<br />

Linear Pair Conjecture, g 80°. The angle with<br />

measure f corresponds to the angle measuring<br />

100°, so f 100°. The angles measuring g and k<br />

are the base angles of an isosceles triangle, so by<br />

the Isosceles Triangle Conjecture, k 80°.<br />

18. 398 square units<br />

ANSWERS TO EXERCISES 103<br />

<strong>Answers</strong> to Exercises


<strong>Answers</strong> to Exercises<br />

CHAPTER 8 REVIEW<br />

1. B (parallelogram) 2. A (triangle)<br />

3. C (trapezoid) 4. E (kite)<br />

5. F (regular polygon) 6. D (circle)<br />

7. J (sector) 8. I (annulus)<br />

9. G (cylinder) 10. H (cone)<br />

11. 12.<br />

13.<br />

14. Sample answer: Construct an altitude from<br />

the vertex of an obtuse angle to the base. Cut off<br />

the right triangle and move it to the opposite side,<br />

forming a rectangle. Because the parallelogram’s<br />

area hasn’t changed, its area equals the area of the<br />

rectangle. Because the area of the rectangle is<br />

given by the formula A bh, the area of the<br />

parallelogram is also given by A bh.<br />

h<br />

15. Sample answer: Make a copy of the trapezoid<br />

and put the two copies together to form a<br />

parallelogram with base b 1 b 2 and height h.<br />

Thus the area of one trapezoid is given by the<br />

formula A 1<br />

b 1<br />

b 2<br />

Apothem<br />

b<br />

2 b 1 b 2h.<br />

16. Sample answer:Cut a circular region into a large<br />

number of wedges and arrange them into a shape<br />

that resembles a rectangle.The base length of this<br />

“rectangle”is r and the height is r, so its area isr 2 .<br />

Thustheareaof acircleisgivenbytheformulaAr 2 .<br />

r<br />

b 2<br />

h h<br />

b 1<br />

17. 800 cm 2 18. 5990.4 cm 2<br />

19. 60 cm 2 or about 188.5 cm 2<br />

b<br />

r<br />

104 ANSWERS TO EXERCISES<br />

h<br />

20. 32 cm 21. 32 cm 22. 15 cm<br />

23. 81 cm 2 24. 48 cm 25. 40°<br />

26. 153.9 cm 2 27. 72 cm 2 28. 30.9 cm 2<br />

29. 300 cm 2 30. 940 cm 2 31. 1356 cm 2<br />

32. Area is 112 square units.<br />

y<br />

33. Area is 81 square units.<br />

F (0, 0)<br />

A (0, 0)<br />

y<br />

D (6, 8) C (20, 8)<br />

R (4, 15)<br />

O (4, –3)<br />

B (14, 0)<br />

U (9, 5)<br />

34. 6 cm 35. 172.5 cm 2<br />

36. sample answers:<br />

x<br />

x<br />

37. 1250 m 2<br />

38. Circle. For the square, 100 4s, s 25,<br />

A 25 2 625 ft 2 . For the circle, 100 2r,<br />

r 15.9, A (15.9) 2 794 ft 2 .<br />

39. A round peg in a square hole is a better fit.<br />

The round peg fills about 78.5% of area of the<br />

square hole, whereas the square peg fills only about<br />

63.7% of the area of the round hole.<br />

40. giant 41. about 14 oz<br />

42. One-eighth of a 12-inch diameter pie;<br />

one-fourth of a 6-inch pie and one-eighth of a<br />

12-inch pie both have the same length of crust,<br />

which is longer than one-sixth of an 8-inch pie.<br />

43a. 96 ft; 40 ft 43b. 3290 ft 2<br />

44. $3000 45. $4160<br />

46. It’s a bad deal. 2r 1 44 cm. 2r 2 22 cm,<br />

which implies 4r 2 44 cm. Therefore r 1 2r 2 .<br />

The area of the large bundle is 4r 2 2 cm 2 .The<br />

combined area of two small bundles is 2r 2 2 cm 2 .<br />

Thus he is getting half as much for the same price.<br />

47. $2002 48. $384 (16 gal)


<strong>Answers</strong> to Exercises<br />

CHAPTER 9 • CHAPTER CHAPTER 9 • CHAPTER<br />

LESSON 9.1<br />

1. c 19.2 cm 2. a 12 cm<br />

3. b 5.3 cm 4. d 10 cm<br />

5. s 26 cm 6. c 8.5 cm<br />

7. b 24 cm 8. x 3.6 cm<br />

9. x 40 cm 10. s 3.5 cm<br />

11. r 13 cm<br />

12. 127 ft<br />

13. 512 m2 14. 11.3 cm<br />

15. 3, 4, 5<br />

16. 28 m<br />

17. The area of the large square is 4 area of<br />

triangle area of small square.<br />

c 2 4 1<br />

2<br />

ab (b a)2<br />

c 2 2ab b 2 2ab a 2<br />

c 2 a 2 b 2<br />

b a<br />

b<br />

a<br />

c<br />

9<br />

18. Sample answer: Yes, ABC XYZ by SSS.<br />

Both triangles are right triangles, so you can<br />

use the Pythagorean Theorem to find that<br />

CB ZY 3 cm.<br />

19. 54 cm 2<br />

20. 3 6 3 2 .4.3.4<br />

21. Mark the unnamed angles as shown in the<br />

figure below. By the Linear Pair Conjecture,<br />

p 120° 180°, so p 60°. By AIA, m q. By<br />

the Triangle Sum Conjecture, q p n 180°.<br />

Substitute m q and p 60° to get<br />

m 60° n 180°. m n 120°.<br />

m<br />

120°<br />

p<br />

q<br />

n<br />

Or use the Exterior Angle Conjecture to get<br />

q n 120°. By AIA, q m. Substituting,<br />

m n 120°.<br />

22. a 122°, b 74°, c 106°, d 16°, e 90°,<br />

f 74°, g 74°, h 74°, n 74°, r 32°,<br />

s 74°, t 74°, u 32°, v 74°. Possible<br />

explanation: By the Tangent Conjecture, the<br />

quadrilateral containing g has two right angles<br />

formed by radii intersecting tangent lines. Using<br />

c 106° and the Quadrilateral Sum Conjecture,<br />

g 90° 90° 106° 360°, so g 74°.<br />

The angle measures e and f and the measure of the<br />

inscribed angle that intercepts the arc with measure<br />

u sum to 180°. Using e 90° and f 74°, the<br />

inscribed angle measures 16°. Using the Inscribed<br />

Angle Conjecture, u 32°.<br />

23 .<br />

T1 T 2<br />

ANSWERS TO EXERCISES 105<br />

<strong>Answers</strong> to Exercises


<strong>Answers</strong> to Exercises<br />

LESSON 9.2<br />

1. yes<br />

2. yes<br />

3. no<br />

4. no<br />

5. no<br />

6. no<br />

7. no<br />

8. No, the given lengths are not a Pythagorean<br />

triple.<br />

9. y 25 cm<br />

10. y 24 units<br />

11. y 17.3 m<br />

12. 6, 8, 10<br />

13. 60 cm 2<br />

14. 14.1 ft<br />

15. 17.9 cm 2<br />

16. 102 m<br />

17a. 1442 cm 2<br />

17b. 74.8 cm<br />

18. Sample answer: The numbers given satisfy the<br />

Pythagorean Theorem, so the triangle is a right<br />

triangle; but the right angle should be inscribed<br />

in an arc of 180°. Thus the triangle is not a right<br />

triangle.<br />

106 ANSWERS TO EXERCISES<br />

19. Sample answer: (BD) 2 62 32 27;<br />

(BC) 2 (BD) 2 92 108; then (AB) 2 (BC) 2 <br />

(AC) 2 (36 108 144), so ABC is a right<br />

triangle by the Converse of the Pythagorean<br />

Theorem.<br />

20. centroid<br />

3<br />

21. 10<br />

22. Because mDCF 90°, mDCE (90 x)°.<br />

Because DCE is isosceles, mDEC (90 x)°.<br />

mD 180° 2 (90 x)° 2x.Because D is a<br />

central angle, 2x a.Therefore,x 1<br />

2 a.<br />

23. The path from C to M to T lies on a straight<br />

line and therefore must be shorter than the path<br />

from C to A to T.<br />

P<br />

C<br />

24. 790 square units<br />

25.<br />

26. 19<br />

A<br />

U<br />

M<br />

T<br />

B


USING YOUR ALGEBRA SKILLS 9<br />

1. 6 2. 5<br />

3. 182 4. 147<br />

5. 8 6. 23<br />

7. 32 8. 210<br />

9. 53 10. 85<br />

11. 46 12. 24<br />

13. 125 14. 28<br />

15. 623<br />

16. <strong>Answers</strong> will vary. Possible answer: The length<br />

of the hypotenuse of an isosceles right triangle<br />

with legs of length 3 units is 18 units. This is the<br />

same as 2 2 2,or 32.<br />

2<br />

1<br />

17. The hypotenuse represents 5 units. In the<br />

second right triangle, the legs have lengths 4 and 2,<br />

so the hypotenuse has length 20.The length of<br />

the hypotenuse is twice the length of the hypotenuse<br />

of the smaller triangle, so 20 25.<br />

5<br />

2<br />

1<br />

1<br />

18<br />

3<br />

20<br />

4<br />

3<br />

2<br />

18. Possible answer: A right triangle with legs of<br />

lengths 1 and 3 units has a hypotenuse of length<br />

10 units.<br />

A right triangle with legs of lengths 6 and 2 units<br />

has a hypotenuse of length 40 10 <br />

10 210.<br />

19. Possible answer: A right triangle with legs of<br />

lengths 2 and 3 units has a hypotenuse of length<br />

13 units.<br />

13<br />

10<br />

3<br />

2<br />

3<br />

20. x 3, y 12. Because y is twice as<br />

long as x, y 2x, so 12 23.<br />

21. 2 2 3 2 7 2<br />

7<br />

2<br />

6<br />

1<br />

40<br />

3<br />

2<br />

ANSWERS TO EXERCISES 107<br />

<strong>Answers</strong> to Exercises


<strong>Answers</strong> to Exercises<br />

1. 722 cm<br />

2. 13 cm<br />

3. 10 cm, 53 cm<br />

4. 103 cm, 10 cm<br />

5. 34 cm, 17 cm<br />

6. 72 cm<br />

7. 123 cm<br />

8. 50 cm, 100 cm<br />

9. 16 cm2 1 1<br />

10. , <br />

2 2 <br />

LESSON 9.3<br />

11. A 30°-60°-90° triangle must have sides whose<br />

lengths are multiples of 1, 2, and 3. The triangle<br />

shown does not reflect this rule.<br />

12. Possible answer:<br />

Use 1 2 3 2 2 2 and 4 2 48 2 8 2 .<br />

4<br />

3<br />

13. possible answer:<br />

14. Possible answer:<br />

Use 22 12 5 2 and 62 32 45 2 .<br />

5<br />

4<br />

2<br />

3<br />

2<br />

4<br />

1<br />

1<br />

2<br />

3<br />

1<br />

4<br />

2<br />

3<br />

4<br />

45 3<br />

15a. CDA, AEC, AEB, BFA, BFC<br />

15b. MDB, MEB, MEC, MFC, MFA<br />

108 ANSWERS TO EXERCISES<br />

8<br />

1<br />

2<br />

3<br />

6<br />

2<br />

16. c 2 x 2 x 2 Start with the Pythagorean Theorem.<br />

c 2 2x 2 Combine like terms.<br />

c x2 Take the square root of both sides.<br />

17. 1693 m 2 292.7 m 2<br />

18. 390 m 2<br />

19.<br />

20. Construct an isosceles right triangle with legs<br />

of length a, construct a 30°-60°-90° triangle with<br />

legs of lengths a and a3,and construct a right<br />

triangle with legs of lengths a2 and a3.<br />

a<br />

a<br />

x 3<br />

a<br />

2<br />

21. Areas: 4.5 cm2 ,8 cm2 , 12.5 cm2 .4.5 8 12.5; that is, the sum of the areas of the<br />

semicircles on the two legs is equal to the area of<br />

the semicircle on the hypotenuse.<br />

22. 73<br />

<br />

6<br />

12.2 chih<br />

23. Extend the rays that form the right angle.<br />

m4 m5 180° by the Linear Pair<br />

Conjecture, and it’s given that m5 90°.<br />

m4 90°. m2 m3 m4 m2 <br />

m3 90° 180°. m2 m3 90°.<br />

m3 m1 by AIA. m1 m2 90°.<br />

24. 80°<br />

x<br />

30°<br />

4<br />

3<br />

45°<br />

a<br />

1<br />

5<br />

a<br />

2<br />

x<br />

2a<br />

3<br />

2<br />

x<br />

2x<br />

a<br />

3<br />

1<br />

15°<br />

3<br />

45°<br />

a<br />

a<br />

2<br />

5


LESSON 9.4<br />

1. No. The space diagonal of the box is about<br />

33.5 in.<br />

2. 10 m 3. 50 km/h<br />

4. 8 ft 5. area: 60 m2 ; cost: $7200<br />

6. surface area of prism 273 180 cm2 <br />

226.8 cm2 ; surface area of cylinder 78 cm2 <br />

245.0 cm2 7.<br />

36<br />

cm 20.8 cm<br />

3<br />

8. 48.2 ft; 16.6 lb<br />

9a. 160 ft-lb<br />

10. about 4.6 ft<br />

9b. 40 lb 9c. 20 lb<br />

11.<br />

2<br />

2<br />

2<br />

2<br />

2 2 2 2<br />

2<br />

4<br />

2<br />

2<br />

4<br />

2<br />

2<br />

2<br />

2<br />

4<br />

2<br />

4<br />

2<br />

12.<br />

13. 12 units<br />

14. 182 cm<br />

15. Draw radii CD and CB. ABC ADC <br />

90°. For quadrilateral ABCD, 54° 90° mC <br />

90° 360°, so mC 126°. BD 126° but<br />

126° 226° 360°.<br />

16. 4, 43<br />

17. SAA<br />

18. orthocenter<br />

19. 115°<br />

20. PO<br />

ANSWERS TO EXERCISES 109<br />

<strong>Answers</strong> to Exercises


<strong>Answers</strong> to Exercises<br />

LESSON 9.5<br />

1. 5 units 2. 45 units 3. 34 units<br />

4. 354 m 5. 52.4 units 6. isosceles<br />

7. rectangle<br />

10<br />

8<br />

6<br />

4<br />

2<br />

y<br />

8. parallelogram<br />

9. kite<br />

K<br />

8<br />

J<br />

10. square<br />

M<br />

11a.<br />

2<br />

2<br />

2<br />

H<br />

D<br />

8<br />

10<br />

8<br />

4<br />

L<br />

I 2<br />

2<br />

y<br />

P<br />

y<br />

(1, –2)<br />

Circle A<br />

(x, y)<br />

110 ANSWERS TO EXERCISES<br />

4<br />

2<br />

O<br />

N<br />

4 2 2 4<br />

y<br />

A<br />

E 2 4 6<br />

G<br />

C<br />

y<br />

B<br />

4 6 8 10<br />

F<br />

x<br />

x<br />

x<br />

x<br />

x<br />

(0, 2)<br />

y<br />

Circle B<br />

(x, y)<br />

x<br />

11b. Circle A: (x 1) 2 ( y 2) 2 64;<br />

Circle B: x2 (y 2) 2 36<br />

11c. (x h) 2 (y k) 2 (r) 2<br />

12. (x 2) 2 y2 25<br />

13. Center is (0, 1), r 9.<br />

14. (x 3) 2 (y 1) 2 18<br />

15a. 14 units<br />

15b. 176 411 units<br />

15c. (x x 2 1 ) 2 y<br />

(y2 1 ) 2 z<br />

(z2 1 ) 2 <br />

16. 86.5 units<br />

17. 14 units<br />

18. n (n 2) 3 (n 3) (n 1)<br />

19. 3<br />

, <br />

2<br />

1<br />

2 <br />

20. k 2, m 6<br />

21. x , y <br />

22. 96 cm<br />

23. The angle of rotation is approximately 77°.<br />

Connect two pairs of corresponding points.<br />

Construct the perpendicular bisector of each<br />

segment. The point where the perpendicular<br />

bisectors meet is the center of rotation.<br />

24. Any long diagonal of a regular hexagon<br />

divides it into two congruent quadrilaterals. Each<br />

(6 2)<br />

angle of a regular hexagon is <br />

6<br />

180°<br />

6 12<br />

<br />

3 3<br />

120°, and<br />

the diagonal divides two of the 120° angles into 60°<br />

angles. Look at the diagonal as a transversal.<br />

The alternate interior angles are congruent, thus<br />

the opposite sides of a regular hexagon are parallel.<br />

120°<br />

120°<br />

60°<br />

60°<br />

60°<br />

60°<br />

120°<br />

120°


LESSON 9.6<br />

1. 18 cm2 56.5 cm2 2. (8 16) m2 9.1 m2 3. 456 cm2 1433 cm2 4. 120 cm2 377.0 cm2 5. 32 323 m2 45.1 m2 6. (25 48) cm2 30.5 cm2 7. 64 <br />

<br />

3<br />

163 cm2 39.3 cm2 8. (240 18) m2 183.5 m2 9. 102 cm<br />

10. Possible proof:<br />

Given: Circle C with tangents AM and AN <br />

Show: AM AN <br />

C<br />

N<br />

M<br />

Add MC ,NC , and AC to the diagram as shown.<br />

By the Tangent Conjecture, M and N are right<br />

angles, so AMC and ANC are right triangles.<br />

Using the Pythagorean Theorem, (AM) 2 (MC) 2<br />

(AC) 2 and (AN) 2 (NC) 2 (AC) 2 .Solvingfor<br />

AM and AN, AM (AC) 2 (MC) 2 and<br />

AN (AC) 2 (NC) 2 .BecauseMC and NC are<br />

radii of the same circle, they have the same lengths.<br />

So, substitute MC for NC in the second equation:<br />

AN (AC) 2 (MC) 2 AM.Therefore,<br />

AM AN .<br />

11. 18 m<br />

12. 324 cm2 1018 cm2 13. 3931 cm2 A<br />

14. 12 63 cm 22.4 cm<br />

15. 77 cm 8.8 cm<br />

16. 76 cm<br />

17. Inscribed circle: 3 cm 2 . Circumscribed circle:<br />

12 cm 2 . The area of the circumscribed circle is four<br />

times as great as the area of the inscribed circle.<br />

18. 135°<br />

19. (x 3) 2 (y 3) 2 36<br />

20. The diameter is the transversal, and the<br />

chords are parallel by the Converse of the Parallel<br />

Lines Conjecture. The chords are congruent<br />

because they can be shown to be the same distance<br />

from the center. (Draw a perpendicular from each<br />

chord to the center and use AAS and CPCTC.)<br />

Sample construction:<br />

21a. Because a carpenter’s square has a right angle<br />

and both radii are perpendicular to the tangents, a<br />

square is formed. The radius is 10 in., therefore the<br />

diameter is 20 in.<br />

10 in.<br />

d 20 in.<br />

10 in.<br />

10 in.<br />

10<br />

in.<br />

21b. Possible answer: Measure the circumference<br />

with string and divide by .<br />

22. 5<br />

6 <br />

ANSWERS TO EXERCISES 111<br />

<strong>Answers</strong> to Exercises


<strong>Answers</strong> to Exercises<br />

CHAPTER 9 REVIEW<br />

1. 20 cm<br />

2. 10 cm<br />

3. obtuse<br />

4. 26 cm<br />

5. 3<br />

, <br />

2<br />

1<br />

2 <br />

6. 1 1<br />

, 2<br />

2<br />

7. 2003 cm2 346.4 cm2 8. d 122 cm 17.0 cm2 9. 246 cm2 10. 72 in2 226.2 in2 11. 24 cm2 75.4 cm2 12. (2 4) cm2 2.28 cm2 13. 222.8 cm2 14. isosceles right<br />

15. No. The closest she can come to camp is 10 km.<br />

16. No. The 15 cm diagonal is the longer diagonal.<br />

17. 1.4 km; 8 1<br />

<br />

2 min<br />

18. yes<br />

19. 29 ft<br />

20. 45 ft<br />

21. 50 mi<br />

22. 707 m2 23. 63 and 18<br />

24. 12 m<br />

25. 42<br />

26. No. If you reflect one of the right triangles into<br />

the center piece, you’ll see that the area of the kite is<br />

almost half again as large as the area of each of the<br />

other triangles.<br />

Extra<br />

30°<br />

30°<br />

30°<br />

Or students might compare areas by assuming the<br />

short leg of the 30°-60°-90° triangle is 1. The area<br />

3<br />

of each triangle is then 0.87 and the area of<br />

2<br />

the kite is 3 3 1.27.<br />

112 ANSWERS TO EXERCISES<br />

27. 7<br />

9 <br />

28. The quarter-circle gives the maximum area.<br />

Triangle:<br />

s<br />

____<br />

2<br />

A 1<br />

45°<br />

Square:<br />

2<br />

<br />

4<br />

2 s<br />

s<br />

1_<br />

2<br />

s<br />

A 1<br />

2 s 1 2<br />

s<br />

2<br />

s <br />

4<br />

Quarter-circle:<br />

___ 2s<br />

<br />

s 1<br />

4 2r<br />

r 2s<br />

<br />

<br />

A 1<br />

4 2 s<br />

<br />

s 2<br />

<br />

<br />

s2<br />

<br />

4<br />

29. 1.6 m<br />

30.<br />

s<br />

s<br />

s<br />

____<br />

2<br />

2<br />

2<br />

45°<br />

1_<br />

2<br />

s<br />

s<br />

<br />

2<br />

s 2<br />

<br />

<br />

31. 4; 0; 10. The rule is n<br />

<br />

2 if n is even, but 0 if n is<br />

odd.


32. 4 in./s 12.6 in./s<br />

33. true<br />

34. true<br />

35. False. The hypotenuse is of length x2.<br />

36. true<br />

37. false; AB x (x2 1 ) 2 (y<br />

2 y1 ) 2 <br />

38. False. A glide reflection is a combination of a<br />

translation and a reflection.<br />

39. False. Equilateral triangles, squares, and regular<br />

hexagons can be used to create monohedral<br />

tessellations.<br />

40. true<br />

41. D<br />

42. B<br />

43. A<br />

44. C<br />

45. C<br />

46. D<br />

47. See flowchart below.<br />

47. (Chapter 9 Review)<br />

1<br />

2<br />

ABCD is<br />

a rectangle<br />

Given<br />

ABCD is<br />

a parallelogram<br />

Definition of<br />

rectangle<br />

4 D B<br />

3<br />

Definition of<br />

rectangle<br />

<br />

DA CB<br />

Definition of<br />

parallelogram<br />

6 <br />

AC AC<br />

Same segment<br />

48.<br />

W B<br />

8-ball<br />

49. 34 cm2 ; 22 42 27.7 cm<br />

50. 40<br />

<br />

3<br />

cm2 41.9 cm2 51. about 55.9 m<br />

52. about 61.5 cm2 53. 48 cm<br />

54. 322 ft2 5<br />

DAC BCA<br />

AIA Conjecture<br />

N<br />

S<br />

7<br />

ABC CDA<br />

SAA Congruence<br />

Conjecture<br />

A<br />

Cue ball<br />

ANSWERS TO EXERCISES 113<br />

E<br />

<strong>Answers</strong> to Exercises


<strong>Answers</strong> to Exercises<br />

<strong>Answers</strong> to Exercises<br />

10<br />

CHAPTER 10 • CHAPTER CHAPTER 10 • CHAPTER<br />

LESSON 10.1<br />

1. polyhedron; polygonal; triangles<br />

2. PQR, TUS<br />

3. PQUT, QRSU, RPTS<br />

4. QU ,PT,RS<br />

5. 6 cm<br />

6. GYPTAN<br />

7. point E<br />

8. GE,YE,PE,TE,AE,NE<br />

9. 13 cm<br />

10. D<br />

11. L<br />

12. C<br />

13. G<br />

14. B<br />

15. H<br />

16. E<br />

17. A<br />

18. J<br />

19. J<br />

20. M<br />

21. H<br />

22. I<br />

23.<br />

114 ANSWERS TO EXERCISES<br />

24.<br />

25.<br />

26.<br />

27. true<br />

28. False. This statement is true only for a right<br />

prism.<br />

29. true<br />

30. true<br />

31. False. It is a sector of a circle.<br />

32. true<br />

33. false; counterexample:<br />

34. true<br />

35. true<br />

x<br />

x<br />

2x<br />

2x


36. See table below.<br />

Possible answers include that the number of lateral<br />

faces of an antiprism is always twice the number<br />

for the related prism; that the number of vertices is<br />

the same for each related prism and antiprism; and<br />

that the number of edges for a prism is three times<br />

the number of faces, while for an antiprism, the<br />

number of edges is twice the number of faces.<br />

37. Answer should include the idea that the<br />

painting “disappears” into the view out the window.<br />

Students might also note the effect created by the<br />

cone-shaped tower appearing similar to the road<br />

disappearing into the distance.<br />

38. 8<br />

36. (Lesson 10.1)<br />

39. 60<br />

40. 30<br />

41a. yes<br />

41b. yes<br />

41c. no<br />

41d. yes<br />

Triangular Rectangular Pentagonal Hexagonal n-gonal<br />

prism prism prism prism prism<br />

Lateral 3 4 5 6 ... n<br />

faces<br />

Total 5 6 7 8 ... n 2<br />

faces<br />

Edges 9 12 15 18 ... 3n<br />

Vertices 6 8 10 12 ... 2n<br />

Triangular Rectangular Pentagonal Hexagonal n-gonal<br />

antiprism antiprism antiprism antiprism antiprism<br />

Lateral 6 8 10 12 ... 2n<br />

faces<br />

Total 8 10 12 14 ... 2n 2<br />

faces<br />

Edges 12 16 20 24 ... 4n<br />

Vertices 6 8 10 12 ... 2n<br />

<br />

<br />

<br />

<br />

ANSWERS TO EXERCISES 115<br />

<strong>Answers</strong> to Exercises


<strong>Answers</strong> to Exercises<br />

LESSON 10.2<br />

1. 72 cm 3 2. 24 cm 3 3. 108 cm 3<br />

4. 160 cm 3 502.65 cm 3<br />

5. 36 cm 3 113.10 cm 3<br />

6. 324 cm 3 1017.88 cm 3<br />

7. See table below.<br />

8. 960 in 3 9. QT cubic units<br />

10. sample answer:<br />

2<br />

12<br />

12 in.<br />

4 in.<br />

8 in.<br />

12<br />

24 in.<br />

3<br />

8<br />

T<br />

11. 2x 3 12. 3r 3 13. 13x 3<br />

14. Margaretta has room for 0.5625 cord. She<br />

should order a half cord.<br />

15. 170 yd 3 16. 5100 lb 17. 11,140<br />

18. The volume of the quilt in 1996 was 4000 ft 3 .<br />

The quilt panels were stacked 2 ft 8 in. high.<br />

19.<br />

7. (Lesson 10.2)<br />

12<br />

116 ANSWERS TO EXERCISES<br />

Q<br />

20.<br />

21. true<br />

22. false<br />

23.<br />

r<br />

r<br />

24. possible solution: prism<br />

Salt crystal<br />

r<br />

25. approximately 1.89 m<br />

26. 12 62<br />

Information about Height Right triangular Right rectangular Right trapezoidal Right<br />

base of solid of solid prism prism prism cylinder<br />

H<br />

h<br />

b<br />

b 6, b 2 7, H 20 a. V d. V g. V j. V <br />

h 8, r 3 480 cm 3 960 cm 3 1040 cm 3 180 cm 3<br />

b 9, b 2 12, H 20 b. V e. V h. V k. V <br />

h 12, r 6 1080 cm 3 2160 cm 3 2520 cm 3 720 cm 3<br />

b 8, b 2 19, H 23 c. V f. V i. V l. V <br />

h 18, r 8 1656 cm 3 3312 cm 3 5589 cm 3 1472 cm 3<br />

H<br />

h<br />

b<br />

H<br />

h<br />

b 2<br />

b<br />

H<br />

r


LESSON 10.3<br />

1. 192 cm3 2. 84 cm3 263.9 cm3 3. 150 cm3 4. 60 cm3 5. 84 cm3 263.9 cm3 6. 384 cm3 1206 cm3 7. m3<br />

<br />

3<br />

cm3 8. 2<br />

3 b3 cm3 9. 324x3 cm3 ;29.6%<br />

10. See table below.<br />

11. V 1<br />

3 M 2H ft3 12. sample answer:<br />

16<br />

M<br />

27<br />

48<br />

3<br />

M<br />

H<br />

13. Mount Etna is larger. The volume for Mount<br />

Etna is approximately 2193 km 3 ,and the volume<br />

for Mount Fuji is approximately 169 km 3 .<br />

10. (Lesson 10.3)<br />

Information about Height Triangular Rectangular Trapezoidal<br />

base of solid of solid pyramid pyramid pyramid Cone<br />

b<br />

H<br />

h<br />

14. 78,375 grams<br />

15. 48 in3 16. 4 units3 17. 144x3 cm3 18. 40,200 gal; 44 h 40 min<br />

19. 71 ft3 20. 403 barrels<br />

21a. 163 cm2 21b. 96 cm2 21c. 803 cm2 21d. 2413 120 cm2 22. A<br />

b 6, b2 7, H 20 a. V d. V g. V j. V <br />

h 6, r 3<br />

120 cm3 240 cm3 260 cm3 60 cm3 b 9, b2 22, H 20 b. V e. V h. V k. V <br />

h 8, r 6<br />

240 cm3 480 cm3 240 cm3 2480<br />

<br />

3<br />

cm3 b 13, b2 29, H 24 c. V f. V i. V l. V <br />

h 17, r 8<br />

884 cm3 1768 cm3 2856 cm3 512 cm3 D'<br />

H<br />

b<br />

h<br />

D<br />

3 1<br />

C<br />

2<br />

4<br />

B<br />

D''<br />

Possible answer: From the properties of reflection,<br />

1 3 and 2 4. m1 m2 90°, so<br />

m3 m4 90°, and m1 m2 m3 <br />

m4 180°. Therefore D, C,and D are collinear.<br />

23a. Y (a c, d)<br />

23b. Y (a c, b d)<br />

23c. Y (a c e, b d f )<br />

b<br />

b 2<br />

H<br />

h<br />

H<br />

ANSWERS TO EXERCISES 117<br />

r<br />

<strong>Answers</strong> to Exercises


<strong>Answers</strong> to Exercises<br />

LESSON 10.4<br />

1. 58.5 in3 2. 323 cm3 55.43 cm3 3. 15 cm<br />

4. 11 cm<br />

5. 5.0 cm<br />

6. 8. 5 in.<br />

2<br />

2 11 in. 63.24 in3 ;<br />

11 in. 2<br />

2 8.5 in. 81.85 in3 The short, fat cylinder has greater volume.<br />

7. 257 ft 3<br />

8. 4 cm<br />

9. He must refute the statement.<br />

10. 1502 lb<br />

11. 192.4 gal<br />

12. 13 min<br />

13. <strong>Answers</strong> will vary, but r 2 H should equal<br />

about 14.4 in 3 .<br />

14. approximately 38 in 3<br />

15. 100,000 m 3 , or about 314,159 m 3 ;<br />

16,528 loads<br />

16. AB EC because the opposite sides of a<br />

parallelogram are congruent. EC BD because<br />

the diagonals of a rectangle are congruent. So,<br />

AB BD because both are congruent to EC.<br />

Therefore, ABD is isosceles.<br />

118 ANSWERS TO EXERCISES<br />

17. 8.2 cm<br />

18. x 96°<br />

19.<br />

20.<br />

21a. A<br />

21b. S<br />

21c. N<br />

21d. S<br />

21e. A<br />

D<br />

x<br />

45°<br />

45°<br />

x x<br />

45° 45°<br />

A x x<br />

3x<br />

x B<br />

C


LESSON 10.5<br />

1. 675 cm3 2. 36 cm3 113.1 cm3 3. 47 in3 4. 1798.4 g<br />

5. The gold has mass 2728.5 g, and the platinum<br />

has mass 7529.8 g. The solid cone of platinum has<br />

more mass.<br />

6. 1.5 cm<br />

7. 10.5 g/cm3 ;silver<br />

8. 8000 cm3 9. The volume of the medallion is 160 cm3 .Yes,it<br />

is gold, and the Colonel is who he says he is.<br />

10. 679 cm3 11. approximately 193 lb; 22 fish<br />

9<br />

12. 3 2<br />

13. flowchart proof:<br />

<br />

QR SP<br />

Given<br />

R S<br />

AIA<br />

SM MR <br />

Given<br />

RMQ SMP<br />

ASA<br />

<br />

MQ MP<br />

CPCTC<br />

QMR PMS<br />

Vertical Angles<br />

M is the midpoint of PQ <br />

Definition of midpoint<br />

14. 15 sides<br />

15a. (1, 3)<br />

15b. (x 1) 2 (y 3) 2 25<br />

16. 58; 3n 2<br />

ANSWERS TO EXERCISES 119<br />

<strong>Answers</strong> to Exercises


<strong>Answers</strong> to Exercises<br />

1. 36 cm 3 113.1 cm 3<br />

LESSON 10.6<br />

2. <br />

6 cm3 0.533 cm3 3. 9<br />

<br />

32<br />

cm3 0.884 cm3 4. 720 cm3 2262 cm3 5. 30 cm3 94.3 cm3 6. 3456 cm3 10,857 cm3 7. 18 m3 56.6 cm3 8. No. The volume of the ice cream is 85.3 cm3 ,<br />

and the volume of the cone is 64 cm3 .<br />

9. only 20 scoops<br />

10. 148<br />

<br />

3<br />

m3 , or about 155 m3 11. They have the same volume.<br />

12. 9 in.<br />

13. 3 cm<br />

14. 8192<br />

<br />

3<br />

cm3 8579 cm3 15. 18 in3 , or about 57 in3 16. No. The unused volume is 16 cm3 ,and the<br />

volume of the golf ball is 10.6 cm3 .<br />

120 ANSWERS TO EXERCISES<br />

17. approximately 15,704 gallons; 53 days<br />

18. lithium<br />

19. 31 ft<br />

20. ABCD is a parallelogram because the slopes of<br />

CD and AB are both 0 and the slopes of BC and<br />

AD are both 5<br />

B (–3, –5)<br />

y<br />

3 .<br />

C (3, 5) D (9, 5)<br />

A (3, –5)<br />

x<br />

21. They trace two similar shapes, except that the<br />

one traced by C is smaller by a scale factor of 1:2.<br />

22. The line traces an infinite hourglass shape. Or,<br />

it traces the region between the two branches of a<br />

hyperbola.<br />

23. w 110°, x 115°, y 80°


LESSON 10.7<br />

1. V 972 cm3 3054 cm3 S 324 cm2 1018 cm2 2. V 0.972 cm3 3.054 cm3 S 3.24 cm2 10.18 cm2 3. V 1152 cm3 3619 cm3 S 432 cm2 1357 cm2 4. S 160 cm2 502.7 cm2 5. V 256<br />

cm3 268.1 cm3 3<br />

6. S 144 cm 2 452.4 cm 2<br />

7. Area of great circle r 2 . Total surface area<br />

of hemisphere 3r 2 . Total surface area of<br />

hemisphere is three times that of area of great<br />

circle.<br />

8. 2 gal<br />

9. V 1<br />

2 4<br />

3 (1.8)3 1<br />

2 (1.8)2 (4.0) <br />

10.368 m 3 32.57 m 3 ; S 1<br />

2 4(1.8) 2 <br />

1<br />

2 2(1.8)(4.0) 13.68 m 2 42.98 m 2<br />

10a. approximately 3082 ft 2<br />

10b. 13 gal<br />

10c. approximately 9568 bushels<br />

11. 153,200,000 km 2<br />

12. The total cost is $131.95. He will stay under<br />

budget.<br />

13. 1.13%<br />

14. 150 cm 3 471.2 cm 3<br />

15. 1<br />

4 <br />

19a. (Lesson 10.7)<br />

16. 1<br />

2 <br />

17. 3<br />

4 <br />

18. The ratio gets closer to 1.<br />

19a and 19b. See tables below.<br />

20. AB CB and AD CD by the definition of<br />

rhombus, and BD BD because it is the same<br />

segment; therefore ABD CBD by SSS. By<br />

CPCTC, 2 3 and 1 4, which shows<br />

that BD bisects both ABC and ADC.Because<br />

all four sides of the rhombus are congruent, a<br />

similar proof can be used to show that ABC <br />

ADC and thus that A and C are both<br />

bisected by diagonal AC.<br />

21a. AB CB by the definition of rhombus and<br />

BE BE because it is the same segment. 1 2<br />

by the Rhombus Angles Conjecture. Therefore,<br />

AEB CEB by SAS.<br />

21b. AE CE by CPCTC, so BD bisects AC.<br />

21c. 3 4 by CPCTC. Also, 3 and 4 form<br />

a linear pair, so they are supplementary. Because<br />

two angles that are congruent and supplementary<br />

are right angles, 3 and 4 are right angles.<br />

21d. Because 3 and 4 are right angles, the<br />

diagonals are perpendicular.You still need to show<br />

that AC bisects BD. Use a proof similar to that<br />

given in 21a to show that AEB AED.Then,<br />

by CPCTC, BE DE, which shows that AC<br />

bisects BD.<br />

n 1 2 3 4 5 6 . . . n ... 200<br />

f(n) 2 1 4 7 10 13 ... 3n 5 ... 595<br />

19b. (Lesson 10.7)<br />

n 1 2 3 4 5 6 . . . n ... 200<br />

f(n) 0 1<br />

1<br />

3<br />

2 5<br />

3 2 5 3 7<br />

... n<br />

n<br />

1 199<br />

<br />

1<br />

... 201<br />

ANSWERS TO EXERCISES 121<br />

<strong>Answers</strong> to Exercises


<strong>Answers</strong> to Exercises<br />

USING YOUR ALGEBRA SKILLS 10<br />

1. h A<br />

b <br />

P 2h<br />

2. b 2<br />

or b P<br />

<br />

2<br />

h<br />

3. r 3V<br />

H<br />

<br />

4. b c 2 a2 <br />

2 • SA<br />

5. a l<br />

P<br />

6. y2 mx2 x1 y1 or y2 mx2 mx1 y1 7. v d<br />

t <br />

The original formula gives distance in terms of<br />

velocity and time.<br />

9<br />

8. F C 5<br />

32<br />

The original formula converts degrees Fahrenheit to<br />

degrees Celsius.<br />

2<br />

9. L g T<br />

2<br />

The original formula gives the period of a<br />

pendulum (time of one complete swing) in terms<br />

of length and acceleration due to gravity.<br />

10b. (Using Your Algebra Skills 10)<br />

122 ANSWERS TO EXERCISES<br />

10a. V F E 2<br />

10b. See table below.<br />

11a. The corresponding radii are approximately<br />

3.63 cm, 3.30 cm, 3.02 cm, and 2.79 cm.<br />

11b. The corresponding heights are approximately<br />

9.32 cm,10.49 cm,11.61 cm,and 12.70 cm.<br />

11c. The corresponding volumes are approximately<br />

129 cm 3 , 120 cm 3 , 111 cm 3 , and 104 cm 3 .<br />

11d. <strong>Answers</strong> will vary. Sample answer: The cone<br />

with slant height 10 cm has the widest radius, so a<br />

scoop of ice cream is least likely to fall off, and that<br />

cone also has the greatest volume.<br />

11e. <strong>Answers</strong> will vary. Sample answer: The cone<br />

with slant height 13 cm has the greatest height, so<br />

the cone appears bigger even though it has the same<br />

surface area as the other cones; that cone also has<br />

the smallest radius and smallest volume, so it could<br />

hold less ice cream and still appear to be a bigger<br />

cone.<br />

m1 m2 m3 f f<br />

12a. A <br />

5<br />

12b. average of 60: 31; average of 70: 56; average<br />

of 80: 81; average of 90: 106 (impossible)<br />

Pentahedron Hexahedron Octahedron Decahedron Dodecahedron<br />

Number of faces<br />

5 6 8 10 12<br />

Number of edges 8 10 12 16 20<br />

Number of vertices 5 6 6 8 10


CHAPTER 10 REVIEW<br />

1. They have the same formula for volume: V BH.<br />

2. They have the same formula for volume: V 1<br />

3 BH.<br />

3. 6240 cm3 4. 1029 cm3 3233 cm3 5. 1200 cm3 6. 32 cm3 7. 100 cm3 314.2 cm3 8. 2250 cm3 7069 cm3 9. H 12.8 cm<br />

10. h 7 cm<br />

11. r 12 cm<br />

12. r 8 cm<br />

13. 960 cm3 14. 9 m<br />

15. 851 cm3 16. four times as great<br />

17a. Vextra large 54 in3 Vjumbo 201.1 in3 Vcolossal 785.4 in3 17b. 14.5 times as great<br />

18. Cylinder B weighs 8<br />

<br />

3 times as much as cylinder A.<br />

19. 2129 kg; 9 loads<br />

20. H2r. Vsphere<br />

0.524. Thus, 52.4% of<br />

Vbox<br />

the box is filled by the ball.<br />

21. approximately 358 yd3 22. No. The unused volume is 98 in3 ,and the<br />

volume of the meatballs is 32 in3 <br />

.<br />

23. platinum<br />

24. No. The ball weighs 253 lb.<br />

25. 256 lb<br />

26. approximately 3 in.<br />

4 r 3<br />

3<br />

<br />

(2r) 3<br />

27. <br />

8 m3 0.23 m3 33<br />

<br />

32<br />

28. 160 cubic units<br />

ANSWERS TO EXERCISES 123<br />

<strong>Answers</strong> to Exercises


<strong>Answers</strong> to Exercises<br />

<strong>Answers</strong> to Exercises<br />

11<br />

CHAPTER 11 • CHAPTER CHAPTER 11 • CHAPTER<br />

USING YOUR ALGEBRA SKILLS 11<br />

1. 3<br />

; 3<br />

8 5 <br />

AC<br />

3 D 5 D<br />

2. CD<br />

, C <br />

5 BD<br />

, B<br />

8 BC<br />

8<br />

<br />

13<br />

3a. 3<br />

1 <br />

9<br />

3b. 1<br />

4. a 6<br />

5. b 16<br />

6. c 39<br />

7. x 5.6<br />

8. y 8<br />

9. x 12<br />

10. z 6<br />

11. d 1<br />

12. y 5<br />

13. 318 mi<br />

14. 2.01<br />

15. 12 ft by 15 ft<br />

124 ANSWERS TO EXERCISES<br />

16. almost 80 years old<br />

17a. true<br />

17b. false; 3<br />

<br />

6<br />

1 ,but<br />

2 3 1 6 2<br />

17c. true<br />

17d. true<br />

17e. false; 3<br />

<br />

6<br />

1 ,but 3 <br />

2 2<br />

1<br />

6 <br />

17f. true<br />

18a. arithmetic: 10, 25, 40, 65; geometric: 10, 20,<br />

40, 80 or 10, 20, 40, 80<br />

18b. arithmetic: 2, 26, 50, 74; geometric: 2, 10, 50,<br />

250 or 2, 10, 50, 250<br />

18c. arithmetic: 4, 20, 36, 52; geometric: 4, 12, 36,<br />

108 or 4, 12, 36, 108<br />

19a. Add the numbers and divide by 2.<br />

19b. Possible answer: Multiply the numbers and<br />

take the positive square root of the result.<br />

19c. c ab; This formula holds for all positive<br />

values of a and b.<br />

19d. c 2 • 50 100 10;<br />

c 4 • 36 144 12


1. A<br />

2. B<br />

3. possible answer:<br />

3. possible answer:<br />

5. possible answer:<br />

LESSON 11.1<br />

6. Figure A is similar to Figure C. Possible answer:<br />

If and , then .<br />

7. AL 6; RA 10; RG 4; KN 6<br />

8. No; the corresponding angles are congruent,<br />

but the corresponding sides are not proportional.<br />

9. NY 21; YC 42; CM 27; MB 30<br />

10. Yes; the corresponding angles are congruent,<br />

and the corresponding sides are proportional.<br />

11. x 6 cm, y 3.5 cm<br />

12. z 10 2<br />

<br />

<br />

A B<br />

<br />

B C<br />

A C<br />

<br />

3<br />

cm<br />

13. Yes, the corresponding angles are congruent.<br />

Yes, the corresponding sides are proportional.Yes,<br />

AED ABC.<br />

9 9<br />

14. m 2<br />

cm 4.5 cm, n 4 cm 2.25 cm<br />

15. 3<br />

1 ; 9<br />

<br />

1<br />

16. Yes, they are similar.<br />

6<br />

4<br />

2<br />

y<br />

y<br />

Y (2, 5)<br />

R (2, 2)<br />

(0, 6)<br />

(1, 1)<br />

5<br />

17. Possible answer: Assuming an arm is about<br />

three times as long as a face, each arm would be<br />

about 260 ft.<br />

18. Possible answer: Not all isosceles triangles are<br />

similar because two isosceles triangles can have<br />

different angle measures. A counterexample is<br />

shown at right. Not all right triangles are similar<br />

because they can have different side ratios, as in a<br />

triangle with side lengths 3, 4, and 5 and a triangle<br />

with side lengths 5, 12, and 13. All isosceles right<br />

triangles are similar because they have angle<br />

measures 45°, 45°, and 90°, and the side lengths have<br />

the ratio 1:1:2.<br />

19. 36 20. bc 21. d<br />

c <br />

22. Possible answers: Jade might get 1825<br />

<br />

4475<br />

of the<br />

profits, or $2,773.18, and Omar might get 2650<br />

<br />

4475<br />

of<br />

the profits, or $4,026.82. Or they take out their<br />

investments and they divide the remaining $2,325:<br />

Jade, $1825 $1,162.50 $2,987.50; and Omar,<br />

$3,812.50.<br />

23a. 23b.<br />

60°<br />

Y' (6, 15)<br />

R' (6, 6)<br />

(5, 6)<br />

O (6, 2)<br />

(7, 3)<br />

x<br />

O' (18, 6)<br />

24. approximately 92 gallons<br />

x<br />

60°<br />

ANSWERS TO EXERCISES 125<br />

<strong>Answers</strong> to Exercises


<strong>Answers</strong> to Exercises<br />

LESSON 11.2<br />

1. 6 cm<br />

2. 40 cm; 40 cm<br />

3. 28 cm<br />

4. 54 cm; 42 cm<br />

5. No, 37<br />

<br />

30<br />

35<br />

.<br />

28<br />

6. Yes, MOY NOT by SAS.<br />

7. Yes, PHY YHT because YH 12 and<br />

20<br />

<br />

15<br />

16<br />

<br />

12<br />

12<br />

<br />

9 (SSS).Yes, PTY is a right triangle<br />

because 202 152 25 2 .<br />

8. TMR THM MHR by AA.<br />

x 15.1 cm, y 52.9 cm, h 28.2 cm<br />

9. Yes, QTA TUR and QAT ARU.<br />

QTA QUR by AA; 6 2<br />

<br />

3 cm<br />

10. 24 cm; 40 cm<br />

11. Yes, THU GDU and HTU DGU;<br />

52 cm; 42 cm<br />

12. SUN TAN by AA; 13 cm; 20 cm<br />

13. Yes, RGO FRG and GOF RFO.<br />

GOS RFS by AA; 28 cm; 120 cm<br />

14. 20 cm; 21 cm<br />

15. x 50, y 9<br />

126 ANSWERS TO EXERCISES<br />

16. r R R2 1 2<br />

17. She should order approximately 919 lb every<br />

three months. Explanations will vary.<br />

18. 448<br />

19. The corresponding angles are congruent; the<br />

ratio of the lengths of corresponding sides is 3<br />

1 ;the<br />

dilated image is similar to the original.<br />

20. Yes, ABCD ABCD.The ratio ofthe<br />

. The ratio of the areas is 4<br />

perimeters is 2<br />

1<br />

y<br />

D' C'<br />

A' B'<br />

x<br />

2 1<br />

<br />

2 1<br />

21. 118 square units<br />

22. The statue was about 40 ft, or 12 m, tall. To<br />

estimate, you need to approximate the height of a<br />

person (or some part of a person) in the picture,<br />

measure a part of the statue in the picture, calculate<br />

the approximate height of that statue piece, and<br />

assume that the statue has the same proportions<br />

as the average person.<br />

1 .


LESSON 11.3<br />

1. 16 m<br />

2. 4 ft 3 in.<br />

3. 30 ft<br />

4. 10.92 m<br />

5. 5.46 m<br />

6. Thales used similar right triangles. The height of<br />

the pyramid and 240 m are the lengths of the legs of<br />

one triangle; 6.2 m and 10 m are the lengths of the<br />

corresponding legs of the other triangle; 148.8 m.<br />

7. 90 m; R and O are both right angles and<br />

P is the same angle in both triangles, so<br />

PRE POC by AA.<br />

8. 300 cm<br />

9.<br />

3 cm<br />

t<br />

2 cm<br />

20 cm<br />

y<br />

h<br />

d<br />

x<br />

45 cm<br />

30 cm<br />

Possible answer: Walk to the point where the guy<br />

wire touches your head. Measure your height, h;<br />

the distance from you to the end of the guy wire, x;<br />

and the distance from the point on the ground<br />

directly below the top of the tower to the end of the<br />

guy wire, y. Solve a proportion to find the height of<br />

the tower, t: h<br />

<br />

t x .Finally,use y the Pythagorean<br />

Theorem to find the length of the guy<br />

wire: t2 y2<br />

.<br />

10. The triangles are similar by AA (because the<br />

ruler is parallel to the wall), so Kristin can use the<br />

length of string to the ruler, the length of string to<br />

the wall, and the length of the ruler to calculate the<br />

height of the wall; 144 in., or 12 ft.<br />

11. MUN MSA by AA; x 31 2<br />

3 .<br />

12. BDC AEC by AA; y 63.<br />

6<br />

13. GHF FHK GFK by AA; h 18, 13<br />

9 4<br />

x 7, 13<br />

y 44. 13<br />

14. sample answer:<br />

A 1<br />

(8.2)(1.7)<br />

2<br />

6.97 cm2<br />

A 1<br />

(3)(4.6)<br />

2<br />

6.9 cm2<br />

3 cm<br />

15. 5 2<br />

3 <br />

16.<br />

D<br />

Given: Parallelogram ABCD<br />

Show: AB CD and AD BC<br />

AD BC <br />

CPCTC<br />

1.7 cm<br />

4<br />

BD BD <br />

Same segment<br />

A<br />

3<br />

Given<br />

8.2 cm<br />

4.6 cm<br />

ABCD is a parallelogram<br />

AD BC <br />

Definition of<br />

parallelogram<br />

2 4<br />

AIA Conjecture<br />

ABD CDB<br />

ASA<br />

17a. 4.6.12<br />

17b. 3.12.12 or 3.12 2<br />

1<br />

C<br />

2<br />

B<br />

<br />

AB CD<br />

Definition of<br />

parallelogram<br />

1 3<br />

AIA Conjecture<br />

AB CD <br />

CPCTC<br />

ANSWERS TO EXERCISES 127<br />

<strong>Answers</strong> to Exercises


<strong>Answers</strong> to Exercises<br />

18. The golden ratio is 1+ 5 , or approximately<br />

2<br />

1.618. Here is one possible construction:<br />

A<br />

mB 90°<br />

Construct BC 1<br />

2 AB<br />

AC <br />

2 5 AB<br />

AX AD <br />

2 5 AB 1<br />

2 AB<br />

AX 5 1<br />

<br />

2<br />

AB<br />

AB<br />

2<br />

AX<br />

5 1<br />

1+<br />

2 5<br />

<br />

Therefore, X is the golden cut.<br />

128 ANSWERS TO EXERCISES<br />

D<br />

X<br />

C<br />

B<br />

19a. <strong>Answers</strong> will vary.<br />

19b. Possible answer: The shape is an irregular<br />

curve.<br />

19c. <strong>Answers</strong> will vary. Possible answers: As the<br />

circular track becomes smaller, the curve becomes<br />

more circular; as the track becomes larger, the<br />

curve becomes more pointed near the fixed point.<br />

As the rod becomes shorter, the curve becomes<br />

more pointed near the fixed point; as the rod<br />

becomes longer, the curve becomes more like an<br />

oval. As the fixed point moves closer to the traced<br />

endpoint, the curve becomes more pointed near<br />

the fixed point; as the fixed point moves closer to<br />

the circular track, the curve begins to look like a<br />

crescent moon.


1. 18 cm<br />

2. 12 cm<br />

3. 21 cm<br />

4. 15 cm<br />

5. 2.0 cm<br />

6. 126 cm2 ; 504 cm2 7. 16 cm<br />

8. 60 cm<br />

9. 4 4<br />

<br />

9<br />

cm<br />

p b<br />

10. <br />

q<br />

; q <br />

LESSON 11.4<br />

11. 63 cm<br />

12. 6 cm<br />

13. x 3 1<br />

3 cm, y 5 13<br />

3<br />

cm, z 8 2<br />

<br />

3<br />

cm<br />

14. B (3, 5), R 13 4 ,7 ; k 7<br />

<br />

h<br />

<br />

4 <br />

15. 2<br />

1<br />

<br />

16.<br />

H<br />

L<br />

E<br />

O V M<br />

A<br />

Consider similar triangles LVE and MTH with<br />

corresponding angle bisectors EO and HA . To<br />

show that the corresponding angle bisectors are<br />

proportional to corresponding sides, for example<br />

EO EL<br />

HA<br />

, HM<br />

show by AA that LOE MAH,<br />

EO EL<br />

and then you can show that HA<br />

. HM<br />

You<br />

know that L M. Use algebra to show that<br />

LEO MHA.<br />

T<br />

17. a<br />

b 1 c<br />

<br />

d<br />

1<br />

a<br />

<br />

b<br />

b<br />

b c d<br />

<br />

d<br />

<br />

d <br />

a b c d<br />

b<br />

d<br />

18. The ratio will be the same as the ratio for the<br />

original rectangle. The ratio is 1<br />

<br />

2 if it can be divided<br />

like this:<br />

It might be any ratio if divided like this:<br />

19. Yes, by the SSS or the SAS Similarity<br />

Conjecture.<br />

20a. 2a b<br />

20b. 2a b<br />

20c. all values of a and b<br />

20d. no values of a and b<br />

21. AB 3 cm, BC 7.5 cm<br />

ANSWERS TO EXERCISES 129<br />

<strong>Answers</strong> to Exercises


<strong>Answers</strong> to Exercises<br />

LESSON 11.5<br />

1. 18 cm2 2. 18 3. 5; 10<br />

4. 27 cm2 7. <br />

1<br />

5. 49<br />

6. 1:3<br />

m<br />

n2 2<br />

or m 2<br />

n<br />

8. Possible answer: Assuming the ad is sold by<br />

area, Annie should charge $6000.<br />

9. 5000 tiles<br />

11.<br />

9<br />

10. 1<br />

12a. a(x) 2x<br />

Area<br />

x in cm 2<br />

1 2<br />

2 4<br />

3 6<br />

4 8<br />

5 10<br />

6 12<br />

12b. A(x) 2x 2<br />

Area<br />

x in cm 2<br />

1 2<br />

2 8<br />

3 18<br />

4 32<br />

5 50<br />

6 72<br />

12c. The equation for a(x) is linear, so the graph is<br />

a line. The equation for A(x) is quadratic, so the<br />

graph is a parabola.<br />

13. Possible proof: The area of the first rectangle<br />

is bh. The area of the dilated rectangle is rh rb,or<br />

r2bh. The ratio of the area of the dilated rectangle<br />

to the area of the original rectangle is r2bh<br />

, bh<br />

or r2 .<br />

14. 16<br />

, <br />

3<br />

20<br />

<br />

3<br />

15. 8<br />

16a. (i) 40°; 50°; 40° (ii) 60°; 30°; 60°<br />

(iii) 22°; 68°; 22°<br />

Conjecture: similar; right triangle<br />

s h<br />

16b. (i) h<br />

(ii) <br />

x (iii) h b<br />

<br />

m<br />

or a <br />

p h<br />

Conjecture: <br />

h q<br />

<br />

16c. h pq <br />

Area in cm 2<br />

Area in cm 2<br />

10<br />

5<br />

70<br />

60<br />

50<br />

40<br />

30<br />

20<br />

10<br />

130 ANSWERS TO EXERCISES<br />

y<br />

y<br />

5<br />

5<br />

x<br />

x<br />

9<br />

16d. x 9, y 16, 12<br />

12<br />

,<br />

16<br />

h (9)(16 ) <br />

144 12<br />

17. 7.1 m 14.2 m 14.2 m 6.5 m<br />

18.<br />

42 m<br />

L<br />

10 m<br />

E<br />

O V M<br />

A<br />

Consider similar triangles LVE and MTH with<br />

corresponding altitudes EO and HA . Show<br />

EO EL<br />

that LOE MAH, then HA<br />

. HM<br />

You know<br />

that L M. Because EO and HA are altitudes,<br />

LOE and MAH are both right angles, and<br />

LOE MAH.<br />

EO EL<br />

So, LOE MAH by AA. Thus HA<br />

, HM<br />

which shows that the corresponding altitudes<br />

are proportional to corresponding sides.<br />

19. x 92°; Explanations will vary but should<br />

reference properties of linear pairs, isosceles<br />

triangles, and alternate interior angles formed<br />

by parallel lines.<br />

20. 105.5 cm2 21. 60 ft2 22. true<br />

23. Two pairs of angles are congruent, so the<br />

triangles are similar by the AA Similarity<br />

Conjecture. However, the two sets of corresponding<br />

sides are not proportional 6<br />

0<br />

<br />

80<br />

105<br />

,so 135<br />

the<br />

triangles are not similar.<br />

24.<br />

Top<br />

Front<br />

42 m<br />

Right side<br />

Top: 6 square units; front: 4 square units; right side:<br />

4 square units. The sum of the areas is half the<br />

surface area. The volume of the original solid is<br />

8 cubic units. The volume of the enlarged solid is<br />

512 cubic units. The ratio of volumes is 64<br />

.<br />

H<br />

T<br />

1


1. 1715 cm 3<br />

LESSON 11.6<br />

2. 16 cm, 4 cm; 768 cm3 2412.7 cm3 ;<br />

12 cm3 37.7 cm3 ; 64<br />

<br />

1<br />

3. 3 25<br />

; 1 ;<br />

5 27<br />

1500 cm3 4. 1944 ft3 6107.3 ft3 ; H 24<br />

3<br />

64<br />

;<br />

27<br />

32 ft<br />

5. 125<br />

<br />

27<br />

6. 2:5 7. 2<br />

<br />

3<br />

8. $1,953.13<br />

9. 2432 lb<br />

10. Possible answer: No, a 4-foot chicken, similar<br />

to a 14-inch 7-pound chicken, would weigh<br />

approximately 282 pounds 1 43<br />

7<br />

483<br />

. x It is unlikely<br />

that the legs of the giant chicken would be able to<br />

support its weight.<br />

11. Possible answer: Assuming the body types of<br />

the African goliath frog and the Brazilian gold frog<br />

are similar, the gold frog would weigh about<br />

0.0001 kg, or 0.1 g.<br />

12. surface area ratio 16<br />

,volume 1 ratio 64<br />

.The 1<br />

dolphin has the greater surface area to volume<br />

ratio.<br />

1<br />

13. The ratio of the volumes is .The 27<br />

ratio ofthe<br />

surface areas is 1<br />

9 .<br />

14.<br />

800<br />

600<br />

400<br />

200<br />

x 1 2 3 4 5<br />

Surface area in cm 2 22 88 198 352 550<br />

Volume in cm 3 6 48 162 384 750<br />

y<br />

Volume<br />

5<br />

Surface area<br />

x<br />

S(x) 22x 2 and V(x) 6x3 .<br />

Possible answer: The surface area equation is<br />

quadratic, so the graph is a parabola, and the<br />

volume equation is cubic, so the graph is a cubic<br />

graph.<br />

15. Possible proof: The volume of the first<br />

rectangular prism is lwh. The volume of the second<br />

rectangular prism is rl rw rh,or r3lwh.The ratio<br />

of the volumes is r3lwh<br />

, lwh<br />

or r 3 .<br />

16. 9,120 m3 or approximately 28,651 m3 s s s2 s<br />

17. 22<br />

4<br />

or <br />

4<br />

<br />

18. yes, because 182 242 302 (Converse of the<br />

Pythagorean Theorem)<br />

19a. Possible answer: Fold a pair of corresponding<br />

vertices (any vertex in the original figure and the<br />

corresponding vertex in the image) together and<br />

crease; repeat for another pair of corresponding<br />

vertices; the intersection of the two creases is the<br />

center of rotation.<br />

19b. Possible answer: Draw a segment (a chord)<br />

between a pair of corresponding vertices and<br />

construct the perpendicular bisector; repeat for<br />

another pair of corresponding vertices; the<br />

intersection of the two perpendicular bisectors<br />

is the center of rotation.<br />

20e. Label the third vertex C. Construct segment<br />

CD, which bisects C. AD<br />

<br />

DB<br />

2x<br />

<br />

3x<br />

2<br />

3 ,or 2:3<br />

ANSWERS TO EXERCISES 131<br />

<strong>Answers</strong> to Exercises


<strong>Answers</strong> to Exercises<br />

LESSON 11.7<br />

1. 5 cm 2. 33 1<br />

<br />

3<br />

cm<br />

3. 45 cm 4. 21 cm<br />

5. 28 cm 6. no<br />

7. José’s method is correct. Possible explanation:<br />

Alex’s first ratio compares only part of a side of the<br />

larger triangle to the entire corresponding side of the<br />

smaller triangle, while the second ratio compares<br />

entire corresponding sides of the triangles.<br />

8. yes<br />

9. 6 cm; 4.5 cm<br />

10. 13.3 cm; 21.6 cm<br />

11. yes; no; no<br />

12. yes; yes; yes<br />

13. 32 cm; 62 cm<br />

14.<br />

15.<br />

E<br />

16. Extended Parallel/Proportionality Conjecture<br />

17. You should connect the two 75-marks. By the<br />

Extended Parallel/Proportionality Conjecture,<br />

drawing a segment between the 75-marks will<br />

75<br />

form a similar segment that has length <br />

100<br />

of the original.<br />

18. 2064 cm3 6484 cm3 19. possible proof:<br />

a b<br />

<br />

c d<br />

I J<br />

ad cb<br />

ad ab cb ab<br />

a(d b) b(c a)<br />

a(d b)<br />

<br />

ab<br />

b(c a)<br />

<br />

ab<br />

d b c a<br />

b<br />

a<br />

132 ANSWERS TO EXERCISES<br />

F<br />

, or 75%,<br />

So two pairs of corresponding sides of XYZ and<br />

XAB are proportional. X X, so<br />

XYZ XAB by the SAS Similarity Conjecture.<br />

Because XYZ XAB, XAB XYZ.<br />

Hence, AB YZ by the Converse of the Parallel<br />

Lines Conjecture.<br />

20. Set the screw so that the shorter lengths of the<br />

styluses are three-fourths as long as the longer<br />

lengths.<br />

21. x 4.6 cm, y 3.4 cm<br />

22. x 45 ft, y 40 ft, z 35 ft<br />

23. 343<br />

<br />

729<br />

24. She is incorrect. She can make only nine 8 cm<br />

diameter spheres.<br />

25. 6x2 ;24x2 ;54x2 26. 1<br />

3 r<br />

27a. Possible construction method: Use the<br />

triangle-and-circle construction from Lesson 11.3,<br />

Exercise 18, to locate the golden cut, X, of AB.<br />

Then use perpendicular lines and circles to create a<br />

rectangle with length AB and width AX.<br />

27b. Possible construction method: Construct<br />

golden rectangle ABCD following the method from<br />

27a. For square AEFD, locate EF by constructing<br />

circle A and circle D each with radius AD. Repeat<br />

the process of cutting off squares as often as<br />

desired. For the golden spiral from point D to point<br />

E, construct circle F with radius EF; select point D,<br />

point E, and circle F and choose Arc On Circle from<br />

the Construct menu.<br />

28. possible answer:<br />

A<br />

S<br />

P<br />

D<br />

B<br />

R<br />

Q<br />

C<br />

Given: Circumscribed quadrilateral ABCD, with<br />

points of tangency P, Q , R, and S<br />

Show: AB DC AD BC


Use segment addition to show that each sum of<br />

lengths of opposite sides is composed of four lengths:<br />

AB DC (AP BP) (DR CR) and AD <br />

BC (AS DS) (BQ CQ). Using the Tangent<br />

Segments Conjecture, AP AS, BP BQ,<br />

CR CQ, and DR DS, and the four lengths in<br />

each sum are equivalent. Here are the algebraic<br />

steps to show that the whole sums are equivalent.<br />

AB DC (AP BP) (DR CR) Segment<br />

addition.<br />

(AS BQ) (DS CQ) Substitute AS<br />

for AP, BQ<br />

for BP, DS for<br />

DR, and CQ<br />

for CR.<br />

(AS DS) (BQ CQ) Regroup the<br />

measurements<br />

by common<br />

points of<br />

tangency.<br />

AB DC AD BC Use segment<br />

addition to<br />

rewrite the<br />

right side as<br />

the other sum<br />

of opposite<br />

sides.<br />

29.<br />

ANSWERS TO EXERCISES 133<br />

<strong>Answers</strong> to Exercises


<strong>Answers</strong> to Exercises<br />

CHAPTER 11 REVIEW<br />

1. x 24 2. x 66<br />

3. x 6 4. x 17<br />

5. 6 cm; 4.5 cm; 7.5 cm; 3 cm<br />

6. 4 1<br />

<br />

6<br />

cm; 71 <br />

2<br />

cm 7. 13 ft 2 in.<br />

8. It would still be a 20° angle.<br />

9.<br />

K P<br />

L<br />

10. Yes. If two triangles are congruent, then<br />

corresponding angles are congruent and<br />

corresponding sides are proportional with ratio<br />

1 , 1 so the triangles are similar.<br />

11. 15 m 12. 4 gal; 8 times<br />

13. Possible answer: You would measure the<br />

height and weight of the real clothespin and the<br />

height of the sculpture.<br />

Wsculpture<br />

W clothespin<br />

Hsculpture<br />

Hclothespin<br />

3<br />

If you don’t know the height of the sculpture, you<br />

could estimate it from this photo by setting up a<br />

ratio, for example,<br />

Hperso<br />

n Hsculpture<br />

Hp<br />

Hsc<br />

erson’<br />

s photo<br />

ulpture’s<br />

photo<br />

134 ANSWERS TO EXERCISES<br />

14. 9:49<br />

15. 5 25<br />

; 1 <br />

4 64<br />

16. $266.67<br />

17. 640 cm3 18. 32; 24; 40; 126<br />

19. 841 coconuts<br />

20a. 1 to <br />

4 to 1<br />

2 ,or 4 to to 2<br />

20b. 3 to 2 to 1<br />

20c. <strong>Answers</strong> will vary.<br />

21. Possible answer: If food is proportional to<br />

1<br />

body volume, then 8000<br />

of the usual amount of food<br />

is required. If clothing is proportional to surface<br />

1<br />

area, then 400<br />

of the usual amount of clothing is<br />

required. It would take 20 times longer to walk a<br />

given distance.<br />

22. The ice cubes would melt faster because they<br />

have greater surface area.


<strong>Answers</strong> to Exercises<br />

CHAPTER 12 • CHAPTER CHAPTER 12 • CHAPTER<br />

LESSON 12.1<br />

1. 0.6018<br />

2. 0.8746<br />

3. 0.1405<br />

4. 11.57<br />

5. 30.86<br />

6. 62.08<br />

7. sin A s<br />

;cos<br />

t<br />

A r<br />

t ;tan A s<br />

<br />

r<br />

8. sin 4<br />

;cos <br />

5<br />

3 ;tan <br />

5<br />

4<br />

3 <br />

7<br />

9. sin A ;cos 25<br />

A 24<br />

7<br />

;tan<br />

25<br />

A ; 24<br />

sin B 24<br />

7<br />

;cos<br />

25<br />

B ;tan 25<br />

B 24<br />

<br />

7<br />

10. 30°<br />

11. 53°<br />

12<br />

12. 30°<br />

13. 24°<br />

14. a 35 cm<br />

15. b 15 cm<br />

16. c 105 yd<br />

17. d 40°<br />

18. e 50 cm<br />

19. f 33°<br />

20. g 18 in.<br />

21. approximately 237 m<br />

22. x 121 ft<br />

23. 6.375<br />

24. 2.2<br />

25. 16-inch pizza<br />

26. box of ice cream<br />

27. 83 cm 13.9 cm<br />

28. V 288 ft 3 905 ft 3 , S 144 ft 2 452 ft 3<br />

ANSWERS TO EXERCISES 135<br />

<strong>Answers</strong> to Exercises


<strong>Answers</strong> to Exercises<br />

LESSON 12.2<br />

1. 9 cm<br />

2. 64°<br />

3. 7 cm<br />

4. 24 m<br />

5. 22 in.<br />

6. 49°<br />

7. 127°<br />

8. 30°<br />

9. 64 cm<br />

10. approximately 655 m<br />

11. approximately 101 m<br />

12. approximately 65 m<br />

13. approximately 188 m<br />

14. approximately 1621 m<br />

15. approximately 1570 m<br />

16a. approximately 974 m<br />

16b. approximately 1007 m<br />

16c. Yes, the height of the balloon would be 1 m<br />

less because you don’t have to account for the<br />

tripod. The distance to a point under the balloon<br />

would not change.<br />

136 ANSWERS TO EXERCISES<br />

17. 45°<br />

18. 60°<br />

19. 60°<br />

20a. 15.04<br />

20b. 2.05<br />

20c. 5.2304<br />

20d. 2.644<br />

21. x 3.5, y 9 1<br />

<br />

7<br />

9.14<br />

22. The block has a volume of 90 cm3 but it<br />

displaces only 62.8 cm3 of water. So not all of the<br />

block is under water, which means it floats.<br />

23. CZ AX BY<br />

24a. decreases, approaching 0<br />

24b. decreases, approaching 0<br />

24c. remains 90°<br />

24d. increases, approaching (AO) 2<br />

24e. decreases, approaching 1<br />

24f. decreases, approaching 1<br />

25. R(8, 7) and E(3, 9), or R(4, 3) and<br />

E(1, 1)


LESSON 12.3<br />

1. 329 cm 2<br />

2. 4 cm 2<br />

3. 11,839 cm 2<br />

4. 407 cm 2<br />

5. 35 cm<br />

6. 17 cm<br />

7. 30 cm<br />

8. 56°<br />

9. 45°<br />

10. 66°<br />

11a. approximately 2200 m<br />

11b. approximately 1600 m<br />

11c. approximately 1400 m<br />

12. The other two walls were 300 ft and<br />

approximately 413 ft. The area was approximately<br />

45,000 ft 2 .<br />

13. approximately 48 m<br />

14. 2366 cm; approximately 41°<br />

15. approximately 5°<br />

16. 93<br />

<br />

8<br />

cm3 , or approximately 6 cm3 17. Because AB CD , A D by the AIA<br />

Conjecture. Because D and B intercept the<br />

same arc, D B. Therefore A B by the<br />

transitive property. So ABE is isosceles by the<br />

Converse of the Isosceles Triangle Conjecture.<br />

18. Draw line through the two points. Then fold<br />

the paper so that the two points coincide. Draw<br />

another line along the fold. These two lines contain<br />

the diagonals of the square. Now fold the paper so<br />

that the two lines coincide. Mark the vertices on<br />

the other line.<br />

19. AC 60 cm, AE 93.75 cm, AF 117 cm<br />

20. Box 1; about 1.2 in. longer<br />

ANSWERS TO EXERCISES 137<br />

<strong>Answers</strong> to Exercises


<strong>Answers</strong> to Exercises<br />

LESSON 12.4<br />

1. 32 cm<br />

2. 47 cm<br />

3. 341 cm<br />

4. 74°<br />

5. 64°<br />

6. 85°<br />

7. approximately 43 cm<br />

8. approximately 30°<br />

9. approximately 116° and 64°<br />

10. approximately 6 min<br />

11. approximately 87.8 ft<br />

12. approximately 139 m<br />

13. approximately 143 m<br />

14. The ladder at approximately a 76° angle is not<br />

safe. The ladder should be at least 6.5 ft and no<br />

more than 14.3 ft from the base of the wall.<br />

15.<br />

c<br />

A b<br />

a<br />

sin A a<br />

c <br />

cos A b<br />

c <br />

tan A a<br />

b <br />

sinA<br />

cos<br />

A<br />

a<br />

c <br />

c a<br />

b<br />

b<br />

tan A<br />

a<br />

c <br />

<br />

b<br />

c <br />

16a. PR x3<br />

16b. Since x 2 x3 2 (2x) 2 ,TPR is a right<br />

triangle by the Converse of the Pythagorean<br />

Theorem. Therefore mTPR 90°.<br />

17a. increases<br />

17b. decreases<br />

17c. increases then decreases<br />

18a. The base perimeters are equal.<br />

18b. The cone has the greater volume.<br />

18c. The cone has the greater surface area.<br />

19a. a translation right 10 units;<br />

(x, y) → (x 10, y)<br />

19b. a rotation 180° about the origin;<br />

(x, y) → (x, y)<br />

138 ANSWERS TO EXERCISES<br />

20. possible answer:<br />

21. possible answer:<br />

x<br />

x<br />

x<br />

x<br />

120° 120°<br />

60° 60°<br />

2x<br />

x<br />

22. <strong>Answers</strong> will vary from simple (circles and<br />

segments) to quite complex. See below for a more<br />

detailed answer.<br />

Students might begin with the simplest case where<br />

all factors are equal. With circles of the same radius<br />

and with endpoints of the segment traveling in the<br />

same direction, at the same speed, and starting<br />

from the same relative position, the midpoint will<br />

trace a circle equal in size to those constructed.<br />

Changing the size of both circles will change the<br />

size of the circle traced, as will changing the<br />

starting positions of the endpoints. The maximum<br />

radius of the traced circle is limited by the radius of<br />

the constructed circles; the smallest observable<br />

trace is a single point.<br />

With all other factors equal and the endpoints of<br />

the segment traveling in opposite directions<br />

around the circles, the midpoint will trace a<br />

segment. Changing the relative speeds of the<br />

endpoints, or the relative radii of the circles, will<br />

produce more complex polar curves, which<br />

students might describe as flowers, radii, or<br />

“spirograph-like.” Changing the distance between<br />

the centers has no effect on the shape of the trace,<br />

only its position.<br />

y<br />

y<br />

2y<br />

y<br />

120° 120°<br />

60° 60°<br />

2y<br />

y


LESSON 12.5<br />

1. approximately 12.7 m<br />

2. approximately 142 mi/h<br />

3. approximately 51 km<br />

4. approximately 5.9 m<br />

5a. approximately 9.1 km<br />

5b. approximately 255°<br />

6. approximately 240 m<br />

7. approximately 42 ft<br />

8. They must dig at an angle of approximately<br />

71.6° and dig for approximately 25.3 m.<br />

9. approximately 168 km/h<br />

10.<br />

6 cm<br />

10a. apothem 9.23 cm; area 277 cm2 10b. leg 9.7 cm; area of triangle 27.7 cm2 ;<br />

area of decagon 277 cm2 10c. They are the same.<br />

11. approximately 108 cm3 12. Sample answer: Any regular polygon can be<br />

divided into n congruent isosceles triangles, each<br />

with a vertex angle of 360°<br />

.<br />

n<br />

The altitude from the vertex angle of an isosceles<br />

triangle bisects the angle and the opposite side.<br />

This altitude is also an apothem of the regular<br />

polygon.<br />

s_<br />

2<br />

360°<br />

___<br />

n<br />

360°<br />

___<br />

2n<br />

a<br />

360°<br />

___<br />

2n<br />

s_<br />

2<br />

For simplicity, let 360<br />

. 2n<br />

Now use the tangent<br />

ratio to find the length of the apothem.<br />

tan <br />

s<br />

a 2tan<br />

Now use the area formula for a regular polygon.<br />

A 1<br />

2 aP<br />

A 1<br />

2 s<br />

2tan<br />

ns<br />

ns2<br />

A 4tan<br />

where 360<br />

<br />

2n<br />

13. approximately 12.8 m at an angle of<br />

approximately 48°<br />

14. 15363 cm3 2660 cm3 s<br />

2<br />

<br />

a<br />

15. The area increases by a factor of 9.<br />

–6<br />

(–3, 0)<br />

y<br />

(0, 2)<br />

A x<br />

(1, 0) 6<br />

1 1_<br />

2 2 1<br />

A 3 1_<br />

2 6 9<br />

–6 (0, –6)<br />

16. 5 15<br />

<br />

4<br />

5 <br />

12<br />

17a. decreases then increases<br />

17b. does not change<br />

18a. a reflection across the x-axis;<br />

(x, y) → (x, y)<br />

18b. a reflection across the y-axis; (x, y) → (x, y)<br />

19. <strong>Answers</strong> will vary.<br />

ANSWERS TO EXERCISES 139<br />

<strong>Answers</strong> to Exercises


<strong>Answers</strong> to Exercises<br />

1.<br />

2.<br />

–2<br />

10<br />

8<br />

6<br />

4<br />

2<br />

USING YOUR ALGEBRA SKILLS 12<br />

All three graphs are V-shaped, consisting of two<br />

parts that have slopes of 1 and 1. Graph g is a<br />

translation of graph f right 5 units. Graph h is a<br />

translation of graph f down 5 units.<br />

3.<br />

y<br />

–4<br />

y<br />

2<br />

x<br />

f(x) x2 –2<br />

–4 –2<br />

–4<br />

2 4<br />

–6<br />

2<br />

2<br />

y<br />

–2 2 4<br />

–2<br />

–4<br />

–6<br />

y<br />

6<br />

4<br />

2<br />

–4 –2 2 4<br />

x<br />

–2<br />

–4<br />

h(x) x3 y<br />

–6 –4 –2<br />

–2<br />

–4<br />

–6<br />

g(x) x<br />

x<br />

2<br />

2<br />

4<br />

y<br />

f(x) x 2<br />

g(x) x 5<br />

–5 5 10<br />

x<br />

–2<br />

–4<br />

A vertical stretch by a factor of 1 reflects the<br />

graph across the x-axis.<br />

140 ANSWERS TO EXERCISES<br />

2<br />

x<br />

g(x) x<br />

f(x) x<br />

4<br />

6<br />

x<br />

6<br />

4<br />

2<br />

–2<br />

–2<br />

–4<br />

–6<br />

y<br />

h(x) x 5<br />

h(x) x 3<br />

2<br />

x<br />

4.<br />

The graph of y 2x 1 is the same as the<br />

graph of p(x). The slope of a line is equal to<br />

the vertical stretch of the linear function. The<br />

y-intercept is equal to ah k.<br />

5. a vertical stretch by a factor of 2 and a vertical<br />

translation down 5 units<br />

–3<br />

6. a horizontal translation right 5 units and a<br />

vertical translation up 2 units<br />

6<br />

4<br />

2<br />

–2<br />

y<br />

–5<br />

2<br />

–2<br />

–4<br />

–6<br />

–8<br />

y<br />

2<br />

7. a vertical stretch by a factor of 1<br />

<br />

2 (a vertical<br />

shrink), a horizontal translation right 2 units,<br />

and a vertical translation down 5 units<br />

1<br />

–6<br />

y<br />

–4 –2<br />

–2<br />

3<br />

g(x) (x 5) 3 2<br />

2<br />

2<br />

–2<br />

–4<br />

–6<br />

y<br />

p(x) 2(x 3) 5<br />

x<br />

f(x) 2x 3 5<br />

6 8<br />

4 6 8<br />

h(x) <br />

1<br />

2 x 2 5<br />

x<br />

x<br />

x<br />

8. f(x) |x 3| 1; a horizontal translation<br />

right3unitsandaverticaltranslationdown1unit<br />

9. f(x) 3x 2 ; a vertical stretch by a factor of 3<br />

10. f(x) (x 3) 2 ; a vertical stretch by a factor<br />

of 1 (a reflection across the x-axis) and a horizontal<br />

translation right 3 units<br />

11. f(x) 2|x| 3; A vertical stretch by a factor<br />

of 2 and a vertical translation up 3 units<br />

12. f(x)3(x1) 2 3; A vertical stretch by a factor<br />

of 3, a horizontal translation left 1 unit, and a<br />

vertical translation down 3 units


13. f(x) 1<br />

2 (x3)2 2; A vertical stretch by a factor<br />

of 1<br />

<br />

2 (a vertical shrink), a horizontal translation<br />

right 3 units, and a vertical translation up 2 units<br />

14. Both graphs have the same “hills and valleys”<br />

shape, but one is a horizontal translation of the<br />

other.<br />

p(x) sin(x)<br />

–360<br />

15. a horizontal translation; a 1, h 90°, k 0<br />

16. The dashed function in each graph is the parent<br />

function f(x) sin(x).<br />

–360<br />

–360<br />

2<br />

2<br />

–2<br />

2<br />

–2<br />

q(x) cos(x)<br />

y<br />

y<br />

y<br />

p(x) 2sin(x)<br />

360<br />

360<br />

r(x) = –sin(x)<br />

360<br />

x<br />

x<br />

x<br />

–360<br />

Var ying a in the equation of the sine function<br />

causes the same types of vertical stretches as with<br />

other functions.<br />

2<br />

–2<br />

y<br />

360<br />

q(x) <br />

1_<br />

2<br />

sin(x)<br />

x<br />

17. Possible answer: f(x) 2sin(x 90°) 1;<br />

a vertical stretch by a factor of 2, a horizontal<br />

translation right 90°, and a vertical translation<br />

down 1 unit<br />

ANSWERS TO EXERCISES 141<br />

<strong>Answers</strong> to Exercises


<strong>Answers</strong> to Exercises<br />

CHAPTER 12 REVIEW<br />

1. 0.8387<br />

2. 0.9877<br />

3. 28.6363<br />

4. a<br />

b ; c a<br />

; <br />

b c <br />

8<br />

5. ; <br />

17<br />

15<br />

8<br />

; 17<br />

15<br />

6. s; t; s<br />

t <br />

7. 33°<br />

8. 86°<br />

9. 71°<br />

10. 1823 cm2 11. 15,116 cm3 12. Yes, the plan meets the act’s requirements. The<br />

angle of ascent is approximately 4.3°.<br />

13. approximately 52 km<br />

14. approximately 7.3°<br />

15. approximately 22 ft<br />

16. approximately 6568 m<br />

17. approximately 2973 km/h<br />

18. 393 cm2 19. 30 cm<br />

20. 78°<br />

21. 105 cm<br />

22. 51°<br />

23. 759 cm2 24. approximately 25 cm<br />

25. 72 cm2 26. approximately 15.7 cm<br />

27. approximately 33.5 cm2 28. approximately 10.1 km/h at an approximate<br />

bearing of 24.5°<br />

29. False; an octahedron is a polyhedron with<br />

eight faces.<br />

30. false<br />

31. true<br />

32. true<br />

142 ANSWERS TO EXERCISES<br />

33. False; the ratio of their areas is m<br />

34. true<br />

35. false; tangent of T <br />

36. true<br />

37. true<br />

38. true<br />

39. true<br />

40. False; the slope of line 2 is <br />

m<br />

41. false<br />

42. B 43. C 44. A 45. D<br />

46. B 47. B 48. A 49. B<br />

50. C<br />

54. <br />

51. D 52. C 53. A<br />

100<br />

cm3 3<br />

55. 28 cm3 56. 30.5 cm3 57. 33<br />

58. (x, y) → (x 1, y 3)<br />

59. w 48 cm, x 24 cm, y 28.5 cm<br />

60. approximately 18 cm<br />

61. (x 5) 2 (y 1) 2 9<br />

62. Sample answer: Each interior angle in a regular<br />

pentagon is 108°. Three angles would have a sum of<br />

324°, or 36° short of 360, which would leave a gap.<br />

Four angles would have a sum exceeding 360° and<br />

hence create an overlap.<br />

63. approximately 99.5 m<br />

64. 30 ft<br />

65. 4 cm<br />

66.<br />

B<br />

A<br />

D<br />

C<br />

66a. mABC 2 mABD<br />

66b. Possible answers: BD is the perpendicular<br />

bisector of AC. It is the angle bisector of ABC,it<br />

is a median, and it divides ABC into two<br />

congruent right triangles.<br />

1 .<br />

.<br />

n2 2<br />

length of leg opposite T<br />

<br />

length of leg adjacent to T


<strong>Answers</strong> to Exercises<br />

CHAPTER 13 • CHAPTER CHAPTER 13 • CHAPTER<br />

LESSON 13.1<br />

1. A postulate is a statement accepted as true<br />

without proof. A theorem is deduced from other<br />

theorems or postulates.<br />

2. Subtraction: Equals minus equals are equal.<br />

Multiplication: Equals times equals are equal.<br />

Division: Equals divided by nonzero equals are equal.<br />

3. Reflexive: Any figure is congruent to itself.<br />

A<br />

C<br />

ABC ABC<br />

Transitive: If Figure A is congruent to Figure B and<br />

Figure B is congruent to Figure C, then Figure A is<br />

congruent to Figure C.<br />

If , then .<br />

P Q R S X Y P Q X Y<br />

If PQ RS and RS XY, then PQ XY.<br />

Symmetric: If Figure A is congruent to Figure B,<br />

then Figure B is congruent to Figure A.<br />

If<br />

then<br />

X<br />

Y Z<br />

L<br />

B<br />

M N<br />

<br />

<br />

L<br />

M N<br />

X<br />

Y Z<br />

,<br />

.<br />

13<br />

If XYZ LMN, then LMN XYZ.<br />

4. reflexive property of equality; reflexive<br />

property of congruence<br />

5. transitive property of congruence<br />

19. (Lesson 13.1)<br />

Given<br />

20. (Lesson 13.1)<br />

6. subtraction property of equality<br />

7. division property of equality<br />

8. Distributive; Subtraction; Addition; Division<br />

9. Given; Addition property of equality;<br />

Multiplication property of equality; Commutative<br />

property of addition.<br />

10. true, definition of midpoint<br />

11. true, Midpoint Postulate<br />

12. true, definition of angle bisector<br />

13. true, Angle Bisector Postulate<br />

14. false, Line Intersection Postulate<br />

15. false, Line Postulate<br />

16. true, Angle Addition Postulate<br />

17. true, Segment Addition Postulate<br />

18.<br />

• That all men are created equal.<br />

• That they are endowed by their creator with<br />

certain inalienable rights, that among these are life,<br />

liberty, and the pursuit of happiness.<br />

• That to secure these rights, governments are<br />

instituted among men, deriving their just powers<br />

from the consent of the governed.<br />

• That whenever any form of government becomes<br />

destructive to these ends, it is the right of the<br />

people to alter or to abolish it, and to institute new<br />

government, laying its foundation on such<br />

principles and organizing its powers in such<br />

form as to them shall seem most likely to effect<br />

their safety and happiness.<br />

19. See flowchart below.<br />

20. See flowchart below.<br />

?<br />

<br />

AO and BO are radii<br />

AOB is isosceles<br />

1 ?<br />

2<br />

AO BO<br />

3<br />

Definition of circle<br />

Definition of ?<br />

<br />

isosceles<br />

?<br />

<br />

<br />

<br />

<br />

1<br />

1 ? 2<br />

2<br />

m n<br />

3<br />

3 ? 4<br />

? Given<br />

Corresponding<br />

? Corresponding<br />

Angles Postulate<br />

Angles Postulate<br />

<br />

ANSWERS TO EXERCISES 143<br />

<strong>Answers</strong> to Exercises


<strong>Answers</strong> to Exercises<br />

21. See flowchart below.<br />

22. See flowchart below.<br />

23. (2n 1) (2m 1) 2n 2m 2 <br />

2(n m 1)<br />

24. (2n 1)(2m 1) 4nm 2n 2m 1 <br />

4nm 2n 2m 2 1 <br />

2(2nm n m 1) 1<br />

25. Let n be any integer. Then the next two<br />

consecutive integers are n 1 and n 2. The sum<br />

of these three integers is (n) (n 1) (n 2) <br />

n n 1 n 2. Combining like terms: 3n 3 <br />

3(n 1), which is divisible by 3.<br />

26. 299 m<br />

27. sphere, cylinder, cone; cylinder, cone, sphere;<br />

cone, cylinder, sphere<br />

21. (Lesson 13.1)<br />

1 AC BD<br />

1 Construct angle<br />

bisector BD<br />

Angle Bisector<br />

Postulate<br />

144 ANSWERS TO EXERCISES<br />

2 ? AD BC<br />

4<br />

ABC <br />

3<br />

?<br />

<br />

<br />

?<br />

<br />

Given<br />

Given<br />

AB BA<br />

Reflexive property<br />

of congruence<br />

22. (Lesson 13.1)<br />

2<br />

28. 30 ft<br />

29. FG 26 and DG 23 because ABGF<br />

and BCDG are parallelograms. Triangle FGD is<br />

right (mFGD 90°) by the Converse of the<br />

Pythagorean Theorem because 26 2 23 2<br />

62 .But mFGB 128° and mDGB 140° by<br />

conjectures regarding angles in a parallelogram.<br />

So, mFGD 92° because the sum of the angles<br />

around G is 360°. So, mFGD is both 90° and 92°.<br />

30. x 54°, y 126°, a 7.3 m<br />

31a. <br />

4 <br />

31b. 1 <br />

4 <br />

31c. <br />

<br />

2<br />

1<br />

?<br />

?<br />

? ?<br />

Given<br />

CBD<br />

BCD A C<br />

3 5 6<br />

ABD BAD <br />

Definition<br />

SAS Congruence CPCTC<br />

of angle bisector Postulate<br />

4 BD BD ?<br />

?<br />

?<br />

<br />

AB CB<br />

BAD<br />

Reflexive property<br />

of congruence<br />

?<br />

<br />

SSS Congruence<br />

Postulate<br />

? ?<br />

5<br />

D C<br />

?<br />

<br />

CPCTC


LESSON 13.2<br />

1. Linear Pair Postulate<br />

2. Parallel Postulate, Angle Addition Postulate,<br />

Linear Pair Postulate, Corresponding Angles<br />

Postulate<br />

3. Parallel Postulate<br />

4. Perpendicular Postulate<br />

5.<br />

2 1<br />

Use the definition of supplementary angles and the<br />

substitution property to get m1 m1 180°.<br />

Then use the division property and the definition<br />

of a right angle.<br />

6.<br />

1 2 4<br />

3<br />

Use the definition of supplementary angles and the<br />

transitive property to get m1 m2 m3 <br />

m4. Then use the substitution property and the<br />

subtraction property to get m1 m4.<br />

7.<br />

1 2<br />

Use the definition of a right angle and the<br />

transitive property to get m1 m2. Then use<br />

the definition of congruence to get 1 2.<br />

8.<br />

3 1<br />

3<br />

Use the VA Theorem and the transitive property to<br />

get 1 3. Therefore the lines are parallel by the<br />

CA Postulate.<br />

9.<br />

3 2<br />

3<br />

2<br />

1<br />

1<br />

1<br />

Use the VA Theorem and the CA Postulate to get<br />

1 3 and 2 3. Then use the transitive<br />

property to get 1 2.<br />

2<br />

2<br />

10.<br />

2<br />

3<br />

Use the VA Theorem and the transitive property to<br />

get 3 4. Therefore the lines are parallel by the<br />

Converse of the AIA Theorem.<br />

11.<br />

3 Use the Linear Pair Postulate and the definition of<br />

supplementary angles to get m1 m3 180°.<br />

Then use the CA Postulate and the substitution<br />

property to get m1 m2 180°.<br />

12.<br />

3 3 1<br />

Use the Linear Pair Postulate and the definition of<br />

supplementary angles to get m1 m3 <br />

m1 m2. Then use the subtraction property<br />

and the Converse of the AIA Theorem to get<br />

1 2 .<br />

13.<br />

A<br />

1<br />

B 2<br />

3<br />

4<br />

2<br />

2<br />

1<br />

3<br />

1<br />

3<br />

Construct a transversal across lines 1 and 2 ;it<br />

will intersect line 3 by the Parallel Postulate. Use<br />

the Interior Supplements Theorem and the<br />

definition of supplementary angles to get m1 <br />

m2 180°. Use the CA Postulate and the<br />

substitution property to get m1 m3 180°.<br />

Therefore 1 3 by the Converse of the Interior<br />

Supplements Theorem.<br />

1<br />

1<br />

1<br />

1<br />

2<br />

3<br />

2<br />

2<br />

2<br />

ANSWERS TO EXERCISES 145<br />

<strong>Answers</strong> to Exercises


<strong>Answers</strong> to Exercises<br />

14.<br />

Use the definition of perpendicular lines and the<br />

transitive property to get m1 m2. Therefore<br />

lines 1 and 2 are parallel by the Converse of the<br />

AIA Theorem.<br />

15.<br />

2<br />

1<br />

By the Triangle Sum Theorem, m1 m2 <br />

m3 180°. By the definition of a right triangle,<br />

1 is a right angle. By the definition of right<br />

angle, m1 90°. Using the subtraction property,<br />

m2 m3 90°, so by the definition of<br />

complementary angles, 2 and 3 are<br />

complementary.<br />

16. Linear Pair<br />

Post.<br />

VA Thm.<br />

1<br />

Converse of<br />

AIA Thm.<br />

Converse of<br />

AEA Thm.<br />

CA Post.<br />

146 ANSWERS TO EXERCISES<br />

3 17. 1066 cm 3<br />

2<br />

3<br />

1<br />

2<br />

18. Area (ft 2 )<br />

Bottom 6<br />

2 sides 9<br />

Back and front 6<br />

2 rooftops 8 1<br />

2 <br />

2 gable ends 2<br />

Total 31 1<br />

2 <br />

Plywood (4 by 8) 32<br />

The area is less than that of one sheet of plywood.<br />

However, it is impossible to cut the correct size<br />

pieces from one piece. This answer assumes that<br />

the bottom of the dog house is included. If the<br />

bottom is not included, and the gables can be cut<br />

separately from the rectangular part of the front<br />

and back, then the pieces can be cut from a single<br />

sheet of plywood.<br />

19. A(6, 10), B(2, 6), C(0, 0); mapping<br />

rule: (x, y) → (2x 8, 2y 2)


LESSON 13.3<br />

1. Case 1: P is collinear with A and B. Use the Line<br />

Intersection Postulate to show that P and E are the<br />

same point and use the definitions of bisector and<br />

midpoint to get AP BP.<br />

Case 2: P is not collinear with A<br />

and B. Use the SAS Congruence Postulate to get<br />

AEP BEP. Then use CPCTC to get AP BP.<br />

A<br />

2. Case 1: P is collinear with A and B. Use the<br />

definitions of congruence and midpoint to show<br />

that P is the midpoint of AB.Then use the<br />

definition of perpendicular bisector.<br />

A<br />

C<br />

P<br />

E<br />

D<br />

P<br />

E<br />

Case 2: P is not collinear with A and B.Draw<br />

midpoint E and PE. Use the SSS Congruence<br />

Postulate to get AEP BEP. Then use CPCTC<br />

and the Congruent and Supplementary Theorem<br />

to prove that AEP and BEP are both right<br />

angles. Therefore PE is the perpendicular bisector<br />

of AB by the definitions of midpoint and<br />

perpendicular.<br />

3. Use the reflexive property and<br />

C<br />

the SSS Congruence Postulate to get<br />

ABC BAC. Therefore,<br />

A B by CPCTC.<br />

4. Use the reflexive property and<br />

the ASA Congruence Postulate to<br />

get ABC BAC. Then use<br />

CPCTC and the definition of<br />

isosceles triangle.<br />

5.<br />

B<br />

C A<br />

P<br />

B<br />

B<br />

A B<br />

A B<br />

Draw line BA.<br />

Use the Isosceles Triangle Theorem to get<br />

PAB PBA. Use the Angle Addition<br />

Postulate and the subtraction property to get<br />

BAC ABC. Then use the Converse of the<br />

C<br />

Isosceles Triangle Theorem (CB CA), the reflexive<br />

property, and the SSS Congruence Postulate to get<br />

ACP BCP. Therefore,ACP BCP by<br />

CPCTC.<br />

6.<br />

C m<br />

A<br />

Use the Line Intersection Postulate and the<br />

Perpendicular Bisector Theorem to get AP BP<br />

and BP CP. Then use the transitive property and<br />

the Converse of the Perpendicular Bisector<br />

Theorem to prove that point P is on line n.<br />

7.<br />

C<br />

Use the Line Intersection Postulate and the Angle<br />

Bisector Theorem to prove that Q is equally distant<br />

from AB and AC and from AB and BC. Then use the<br />

transitive property and the Converse of the Angle<br />

Bisector Theorem to prove that point Q is on line n.<br />

8.<br />

B<br />

A<br />

Use the Linear Pair Postulate and the definition of<br />

supplementary angles to get m3 m4 180°.<br />

Then use the Triangle Sum Theorem and the<br />

transitive property to get m1 m2 <br />

m3 m3 m4. Therefore, m1 <br />

m2 m4 by the subtraction property.<br />

9. D<br />

A<br />

n<br />

m<br />

A B<br />

1<br />

3 4<br />

2<br />

1<br />

2<br />

B<br />

Q<br />

n<br />

P<br />

<br />

3 4<br />

C<br />

C<br />

<br />

Use the Triangle Sum Theorem and the addition<br />

property to get mA m1 m3 mC <br />

m4 m2 360°. Then use the Angle Addition<br />

Postulate and the substitution property to get<br />

mA mABC mC mCDA 360°.<br />

10. Use the definitions of median<br />

and midpoint to get BM 1 BC 2 and<br />

AN 1<br />

C<br />

N M<br />

AC. 2 Then use the multiplication<br />

property and the substitution<br />

A B<br />

property to get AN BM . By the<br />

reflexive property, the Isosceles Triangle Theorem,<br />

and the SAS Congruence Postulate, ABN <br />

BAM. Therefore, BN AM by CPCTC.<br />

B<br />

ANSWERS TO EXERCISES 147<br />

<strong>Answers</strong> to Exercises


<strong>Answers</strong> to Exercises<br />

11.<br />

A B<br />

Use the Angle Addition Postulate and the<br />

definition of angle bisector to get<br />

mPAB 1 mCAB 2 and<br />

mQBA 1 mCBA. 2 Then use the Isosceles<br />

Triangle Theorem, the multiplication property, and<br />

the substitution property to get PAB QBA.<br />

By the reflexive property and the ASA Congruence<br />

Postulate, ABP BAQ. Therefore, AP BQ<br />

by CPCTC.<br />

12.<br />

C<br />

A B<br />

Use the Isosceles Triangle Theorem, the Right<br />

Angles Are Congruent Theorem, and the SAA<br />

Theorem to get ABT BAS. Therefore,<br />

AS BT by CPCTC.<br />

13.<br />

C<br />

A D B<br />

median → angle bisector<br />

Use the definitions of median, midpoint, and<br />

isosceles triangle, the reflexive property, and the<br />

SSS Congruence Postulate to prove that<br />

ADC BDC. Then use CPCTC and<br />

the definition of angle bisector.<br />

C<br />

Q P<br />

T S<br />

A D B<br />

C median by CPCTC and the definitions of midpoint<br />

and median.<br />

altitude → median<br />

Use the Right Angles Are Congruent Theorem, the<br />

Isosceles Triangle Theorem, and the SAA Theorem<br />

to get ADC BDC. Therefore, CD is the<br />

148 ANSWERS TO EXERCISES<br />

angle bisector → altitude<br />

Use the definitions of angle bisector and isosceles<br />

triangle, the reflexive property, and the SAS<br />

Congruence Postulate to get ADC BDC.<br />

Then use CPCTC, the Linear Pair Postulate, and<br />

the Congruent and Supplementary Theorem to<br />

prove that ADC and BDC are both right<br />

angles. Therefore CD is the altitude by the<br />

definitions of perpendicular and altitude.<br />

14. x 6, y 3<br />

15. 21.9 m<br />

16. first image: (0, 3); second image: (6, 1)<br />

17. BC FC makes ABCF a rhombus, so its<br />

diagonals are perpendicular. mFGC 90°, so<br />

mCFG mFCG 90°. FD GE, so by<br />

subtraction and transitivity m2 mFCG.<br />

1 FCG by AIA, so 1 2.<br />

18a. 18b. 18c.<br />

19. One possible sequence:<br />

1. Fold A onto B and crease. Label as 1 . Label the<br />

midpoint of the arc M.<br />

2. Fold line 1 onto itself so that M is on the crease.<br />

Label as 2 .<br />

M is the midpoint of AB 3<br />

4<br />

, and 2 is the desired<br />

tangent.<br />

<br />

<br />

3<br />

<br />

<br />

6 <br />

3<br />

4<br />

A<br />

C<br />

A D B<br />

1 M<br />

2<br />

B<br />

20. mBAC 13°<br />

21a. B<br />

21b. B<br />

22a. x 1<br />

(a<br />

2<br />

c), y 1 (a<br />

2<br />

b), z 1 (b<br />

2<br />

c)<br />

22b. w 1<br />

(a<br />

2<br />

b), x 1 (b<br />

2<br />

e), y 1 (e<br />

2<br />

d),<br />

z 1<br />

(d<br />

2<br />

a)


1.<br />

A<br />

LESSON 13.4<br />

Use x to represent the measures of one pair of<br />

congruent angles and y for the other pair. Use the<br />

Quadrilateral Sum Theorem and the division<br />

property to get x y 180°. Therefore, the<br />

opposite sides are parallel by the Converse of the<br />

Interior Supplements Theorem.<br />

2. D C<br />

A<br />

Use the AIA Theorem, the reflexive property, and<br />

the SAS Congruence Postulate to get ADC <br />

CBA. Then use CPCTC and the Converse of the<br />

AIA Theorem to get AB DC .<br />

3. D C<br />

A<br />

Use the definition of rhombus, the reflexive<br />

property,and the SSS Congruence Postulate to get<br />

ABC ADC. Then use CPCTC and the<br />

definition of angle bisector to prove that AC bisects<br />

DAB and BCD. Repeat the steps above using<br />

diagonal DB.<br />

4. D C<br />

A<br />

Use the definition of parallelogram to get AD BC<br />

and AB DC . Then use the Interior Supplements<br />

Theorem.<br />

5. D C<br />

A<br />

D C<br />

1<br />

7<br />

8<br />

1<br />

2<br />

4<br />

3<br />

2<br />

6 5<br />

4<br />

3<br />

B<br />

4<br />

1<br />

B<br />

B<br />

3 2<br />

B<br />

B<br />

Use the reflexive property and the SSS Congruence<br />

Postulate to get ABD CDB. Then use<br />

CPCTC and the Converse of the AIA Theorem to<br />

get AB CD and AD CB. Therefore,ABCD is a<br />

rhombus by the definitions of parallelogram and<br />

rhombus.<br />

6.<br />

D<br />

A<br />

Use the Converse of the Opposite Angles Theorem<br />

to prove that ABCD is a parallelogram. Then use<br />

the definition of rectangle.<br />

7. D<br />

C<br />

A<br />

Use the definition of rectangle to prove that ABCD<br />

is a parallelogram and DAB CBA. Then use<br />

the Parallelogram Opposite Sides Theorem, the<br />

reflexive property, and the SAS Congruence Postulate<br />

to get DAB CBA. Finish with CPCTC.<br />

8. D<br />

C<br />

A<br />

Use the Parallelogram Opposite Sides Theorem,<br />

the reflexive property, and the SSS Congruence<br />

Postulate to get DAB CBA. Repeat the above<br />

steps to get ADC CBA and DAB BCD.<br />

Then use CPCTC and the transitive property to get<br />

DAB ABC BCD ADC. Finish with<br />

the Four Congruent Angles Rectangle Theorem.<br />

9. D<br />

C<br />

A E<br />

Use the Parallel Postulate to construct DE CB.<br />

Then use the Parallelogram Opposite Sides<br />

Theorem and the transitive property to prove that<br />

AED is isosceles. Therefore, A B by the<br />

Isosceles Triangle Theorem, the CA Postulate, and<br />

substitution.<br />

10. D<br />

C<br />

A<br />

C<br />

B<br />

B<br />

B<br />

B<br />

B<br />

Use the Isosceles Trapezoid Theorem, the reflexive<br />

property, and the SAS Congruence Postulate to get<br />

DAB CBA. Then AC BD by CPCTC.<br />

ANSWERS TO EXERCISES 149<br />

<strong>Answers</strong> to Exercises


<strong>Answers</strong> to Exercises<br />

11.<br />

A<br />

2 1<br />

D<br />

4<br />

3<br />

Use the Parallelogram Opposite Angles Theorem,<br />

the multiplication property, and the definition of<br />

angle bisector to get 1 3. Then use the<br />

Converse of the Isosceles Triangle Theorem,<br />

the definition of isosceles triangle, and the<br />

Parallelogram Opposite Sides Theorem to get<br />

AB BC DC AD .<br />

12.<br />

W Z<br />

X P Y<br />

Use the Converse of the Angle Bisector Theorem<br />

to prove that WY is the angle bisector of Y. In<br />

like manner, WY is the angle bisector of W.<br />

Therefore, WXYZ is a rhombus by the Converse<br />

of the Rhombus Angles Theorem.<br />

13. Linear Pair Postulate<br />

CA Postulate<br />

Interior Supplements Theorem<br />

Parallelogram Consec. Angle Theorem<br />

150 ANSWERS TO EXERCISES<br />

B<br />

C<br />

Q<br />

17. (Lesson 13.4)<br />

parallelogram<br />

2 diagonals<br />

2 bisectors<br />

of sides<br />

isosceles<br />

trapezoid<br />

14.<br />

ASA Congruence<br />

Postulate<br />

Parallelogram<br />

Diagonal Lemma<br />

Opposite Sides<br />

Theorem<br />

Converse of the<br />

IT Theorem<br />

Opposite Angles<br />

Theorem<br />

Converse of the Rhombus<br />

Angles Theorem<br />

Line Postulate Angle Addition<br />

Postulate<br />

Double-Edged<br />

Straightedge Theorem<br />

SSS Congruence<br />

Postulate<br />

IT Theorem<br />

Converse of the Angle<br />

Bisector Theorem<br />

15a. A<br />

15b. S<br />

15c. S<br />

15d. N<br />

16. 2386 ft 2<br />

17. See table below.<br />

18. V 1 V 2 has length 12.8 and bearing 72.6°.<br />

19a. B<br />

19b. A<br />

20a. 19°<br />

20b. 52°<br />

20c. 52°<br />

20d. 232°<br />

20e. 19°<br />

Name Lines of symmetry Rotational symmetry<br />

none 2-fold<br />

trapezoid none none<br />

kite 1 diagonal none<br />

square 4-fold<br />

rectangle 2 bisectors of sides 2-fold<br />

rhombus 2 diagonals 2-fold<br />

1 bisector of sides none


LESSON 13.5<br />

1. D: Paris is in France; Tucson is in the U.S.;<br />

London is in England. Bamako must be the capital<br />

of Mali.<br />

2. C: The “Sir” in part A shows that Halley was<br />

English; Julius Caesar was an emperor, not a<br />

scientist; Madonna is a singer. Galileo Galilei must<br />

be the answer.<br />

3. No, the proof is claiming only that if two<br />

particular angles are not congruent, then the two<br />

particular sides opposite them are not congruent. It<br />

still might be the case that a different pair of angles<br />

are congruent and that therefore a different pair of<br />

sides are congruent.<br />

4. Yes, this statement is the contrapositive of the<br />

conjecture proved in Example B, so they are<br />

logically equivalent.<br />

5. 1. Assume the opposite of the conclusion;<br />

2. Triangle Sum Theorem; 3. Substitution property<br />

of equality; 4. 0°; Subtraction property of equality<br />

6. Assume ZOID is equiangular. Use the definition<br />

of equiangular and the Four Congruent Angles<br />

Rectangle Theorem to prove that ZOID is a<br />

rectangle. Therefore ZOID is a parallelogram, which<br />

creates a contradiction.<br />

7. Assume CD is the altitude to AB. Use the<br />

definitions of altitude, median, and midpoint, the<br />

Right Angles Are Congruent Theorem, and the<br />

SAS Congruence Postulate to get ADC BDC.<br />

Therefore AC BC, which creates a contradiction.<br />

8. Assume ZO ID. Use the Opposite Sides<br />

Parallel and Congruent Theorem to prove that<br />

ZOID is a parallelogram, which creates a<br />

contradiction.<br />

9. Given: Circle O with chord AB and perpendicular<br />

bisector CD <br />

Show: CD passes through O<br />

Assume CD does not pass through O. Use the Line<br />

Postulate to construct OB and OA and the<br />

Perpendicular Postulate to construct OE. Then use<br />

the Isosceles Triangle Theorem, the Right Angles<br />

Are Congruent Theorem, and the SAA Theorem to<br />

get OEA OEB. From CPCTC and the<br />

definition of midpoint, prove that E is the midpoint<br />

of AB, which creates a contradiction.<br />

C<br />

O<br />

10. a 75°, b 47°, c 58°<br />

11. 42 ft3 132 ft 3<br />

12a.<br />

12b. 1 3 <br />

13a. A<br />

13b. N<br />

13c. S<br />

13d. S<br />

13e. A<br />

<br />

3 <br />

3<br />

<br />

4 12<br />

B<br />

D<br />

E<br />

A<br />

ANSWERS TO EXERCISES 151<br />

<strong>Answers</strong> to Exercises


<strong>Answers</strong> to Exercises<br />

1.<br />

LESSON 13.6<br />

Case 1 The same: Use the Inscribed Angle Theorem<br />

and the transitive property to get A B.<br />

B<br />

A<br />

B<br />

Case 2 Congruent: Use the multiplication<br />

property to get 1<br />

2 mYZ 1<br />

2 mWX . Then follow<br />

the steps in Case 1.<br />

2. D<br />

Use the Inscribed Angle Theorem, the addition<br />

property, and the distributive property to get<br />

mA mC 1<br />

2 mBCD<br />

mDAB<br />

.Then use<br />

the definition of degrees in a circle, the substitution<br />

property, and the definition of supplementary<br />

angles to get A and C are supplementary.<br />

Repeat the steps using mB and mD to get B<br />

and D are supplementary.<br />

3.<br />

Use the Line Postulate to construct AD . Then use<br />

the AIA Theorem, the Inscribed Angle Theorem,<br />

and substitution to get AC BD .<br />

4.<br />

T<br />

C<br />

R<br />

A<br />

A<br />

A<br />

C<br />

W<br />

152 ANSWERS TO EXERCISES<br />

Z<br />

W<br />

X<br />

Y<br />

B<br />

C<br />

E<br />

X<br />

B<br />

D<br />

Use the Cyclic Quadrilateral Theorem, the<br />

Opposite Angles Theorem, and the Congruent and<br />

Supplementary Theorem to get R, C, E, and<br />

T are right angles.<br />

5.<br />

P<br />

Use the Line Postulate to construct OS ,OT, and<br />

OP. Then use the Tangent Theorem, the Converse<br />

of the Angle Bisector Theorem, and the SAA<br />

Theorem to get OSP OTP. PS PT by<br />

CPCTC.<br />

6.<br />

D<br />

Use the Line Postulate to construct AD . Then use<br />

the Inscribed Angle Theorem, the addition<br />

property, and the distributive property to get<br />

m2 m3 1<br />

2 mAC mBD . Therefore,<br />

m1 1<br />

2 mAC mBD by the Triangle Exterior<br />

Angle Theorem and the transitive property.<br />

7.<br />

P<br />

B<br />

B<br />

S<br />

2<br />

O<br />

D<br />

O<br />

a<br />

3<br />

1<br />

E<br />

b<br />

A<br />

A<br />

1<br />

T<br />

C<br />

C<br />

Intersecting Secants Theorem: The measure of an<br />

angle formed by two secants intersecting outside a<br />

circle is half the difference of the measure of the<br />

larger intercepted arc and the measure of the<br />

smaller intercepted arc. Use the Triangle Exterior<br />

Angle Theorem and the subtraction property to get<br />

x b a. Then use the Inscribed Angle Theorem,<br />

the substitution property, and the distributive<br />

property to get x 1<br />

2 mBD mAC .<br />

2<br />

x


8.<br />

Use the Inscribed Angle Theorem to get<br />

mBDC 1<br />

2 mBEC .By the definition of<br />

semicircle, mBEC<br />

mBDC<br />

180°, so by the<br />

division property, mBDC 90°. By the<br />

definition of right angle, BDC is a right angle.<br />

Because only definitions, properties, and the<br />

Inscribed Angle Theorem are needed to prove this<br />

conjecture, it is a corollary of the Inscribed Angle<br />

Theorem.<br />

9.<br />

10.<br />

B<br />

Line<br />

Postulate<br />

SSS<br />

Congruence<br />

Postulate<br />

IT<br />

Theorem<br />

E<br />

Linear Pair<br />

Postulate<br />

A<br />

Converse of the<br />

IT Theorem<br />

D<br />

ASA Congruence<br />

Postulate<br />

Converse of the Angle<br />

Bisector Theorem<br />

Exterior Angle<br />

Sum Theorem<br />

Inscribed Angle<br />

Theorem<br />

Cyclic Quadrilateral<br />

Theorem<br />

CA<br />

Postulate<br />

C<br />

SAA<br />

Theorem<br />

VA<br />

Theorem<br />

Angle Addition<br />

Postulate<br />

Tangent Segments<br />

Theorem<br />

Parallel<br />

Postulate<br />

AIA<br />

Theorem<br />

Triangle Sum<br />

Theorem<br />

SSS Congruence<br />

Postulate<br />

Right Angles Are<br />

Congruent Theorem<br />

Tangent<br />

Theorem<br />

Parallelogram<br />

Diagonal Lemma<br />

Opposite Angles<br />

Theorem<br />

IT<br />

Theorem<br />

Parallelogram Inscribed<br />

in a Circle Theorem<br />

Angle Addition<br />

Postulate<br />

ASA Congruence<br />

Postulate<br />

SAS Congruence<br />

Postulate<br />

Midpoint<br />

Postulate<br />

Perpendicular<br />

Postulate<br />

Right Angles<br />

Are Congruent<br />

Theorem<br />

Arc Addition<br />

Postulate<br />

Congruent and<br />

Supplementary Theorem<br />

11. A(4.0, 2.9), P(1.1, 2.8)<br />

12. 3; 1; 9<br />

13. BC,AB,AC,CD ,AD <br />

14. 32.5<br />

15. As long as point P is inside the triangle,<br />

a b c h.<br />

Proof: Let x be the length of a side. The areas of the<br />

three small triangles are 1<br />

<br />

2 xa, 1 <br />

2 xb, and 1 <br />

2 xc. The area<br />

of the large triangle is 1<br />

<br />

2 xh. So, 1 <br />

2 xa 1 <br />

2 xb 1 <br />

2 xc <br />

xh. Divide both sides by 1 x. So, a b c h.<br />

1<br />

2<br />

h<br />

a<br />

P<br />

b<br />

c<br />

16a. 27°<br />

16b. 47°<br />

16c. 67°<br />

16d. 133°<br />

16e. cannot be determined<br />

16f. cannot be determined<br />

17.<br />

x<br />

A<br />

O M P<br />

B<br />

Steps:<br />

1. Construct OP.<br />

2. Bisect OP. Label midpoint M.<br />

3. Construct circle with center M and radius PM.<br />

4. Label the intersection of the two circles A and B.<br />

5. Construct PA and PB. PA and PB are the<br />

required tangents.<br />

OAP and OBP are right angles because they are<br />

inscribed in semicircles.<br />

2<br />

ANSWERS TO EXERCISES 153<br />

<strong>Answers</strong> to Exercises


<strong>Answers</strong> to Exercises<br />

1.<br />

T<br />

B H<br />

A<br />

LESSON 13.7<br />

Use the Right Angles Are Congruent Theorem,<br />

CASTC, and the AA Similarity Postulate to get<br />

BT<br />

TH<br />

BHT GEV. Therefore, GV<br />

VE<br />

by CSSTP. See<br />

the Solutions Manual for the family tree.<br />

2. A<br />

G<br />

S L M<br />

Use the definitions of median and midpoint, the<br />

Segment Addition Postulate, the substitution<br />

property, and the multiplication property to get<br />

BY 1 BI 2 and SL 1 SM. 2 Then use CASTC,<br />

CSSTP, and the SAS Similarity Theorem to get<br />

BYG SLA. Therefore, BG<br />

GY<br />

<br />

SA AL<br />

by CSSTP. See<br />

the Solutions Manual for the family tree.<br />

3. C<br />

F<br />

1 3<br />

A B<br />

P<br />

Use the definition of angle bisector, the Angle<br />

Addition Postulate, and the substitution property to<br />

get mACB 2m1 and mDFE 2m2. Then<br />

use CASTC and the AA Similarity Postulate to get<br />

APC DQF. Therefore, AC<br />

CP<br />

<br />

DF<br />

FQ<br />

by CSSTP.<br />

4. C<br />

D E<br />

1<br />

2<br />

A B<br />

Use the CA Postulate and the AA Similarity<br />

Postulate to get CDE CAB. Then use CSSTP<br />

and the Segment Addition Postulate to get CD DA<br />

<br />

CD<br />

<br />

CE EB<br />

DA<br />

EB<br />

. CE<br />

Therefore, CD<br />

CE<br />

by algebra.<br />

5. C<br />

D E<br />

1<br />

2<br />

A B<br />

2 4<br />

D E<br />

Q<br />

DA<br />

EB<br />

Use the addition property to get CD<br />

1 <br />

CE<br />

1.<br />

Then use algebra and the Segment Addition<br />

CA<br />

CB<br />

Postulate to get CD<br />

. CE<br />

Therefore ABC <br />

DEC by the SAS Similarity Theorem, 1 2<br />

by CASTC, and DE AB by the CA Postulate.<br />

154 ANSWERS TO EXERCISES<br />

G<br />

V<br />

E<br />

B Y I<br />

L<br />

6.<br />

Use the Right Angles Are Congruent Theorem, the<br />

reflexive property, and the AA Similarity Postulate<br />

to get ADC ACB and ACB CDB.<br />

Therefore, ADC ACB CDB by the<br />

transitive property of similarity.<br />

7.<br />

C<br />

Use the Three Similar Right Triangles Theorem to<br />

get ADC CDB. Then use CSSTP to get<br />

x h<br />

h <br />

y .<br />

8.<br />

A<br />

A<br />

Draw the altitude to the hypotenuse, then use the<br />

ratios given by the Three Similar Right Triangles<br />

a c<br />

yields a2 c2 cd.<br />

Theorem. In particular, c d <br />

a<br />

b c<br />

Now look at the other small triangle and use d b <br />

to get b2 cd.<br />

9.<br />

C<br />

Begin by constructing a second triangle, right<br />

triangle DEF (with E a right angle), with legs of<br />

lengths a and b and hypotenuse of length x. The<br />

plan is to show that x c, so that the triangles<br />

are congruent. Then show that C and E are<br />

congruent. Once you show that C is a right angle,<br />

then ABC is a right triangle.<br />

10. H<br />

L<br />

Y<br />

a<br />

d<br />

C<br />

1 2<br />

D<br />

h<br />

x D y<br />

b a<br />

c<br />

B A<br />

c<br />

b<br />

c – d<br />

P E<br />

B<br />

B<br />

D F<br />

x<br />

Use the Pythagorean Theorem to write expressions<br />

for the lengths of the unknown legs. Show that the<br />

expressions are equivalent. The triangles are<br />

congruent by SSS or SAS.<br />

a<br />

E<br />

G<br />

b


11.<br />

CA<br />

Postulate<br />

12.<br />

13.<br />

Perpendicular<br />

Postulate<br />

Segment Duplication<br />

Postulate<br />

Parallel<br />

Postulate<br />

CA<br />

Postulate<br />

AA Similarity<br />

Postulate<br />

SAS<br />

Congruence<br />

Postulate<br />

SAS Similarity<br />

Theorem<br />

SSS Similarity<br />

Theorem<br />

Three Similar<br />

Right Triangles<br />

Theorem<br />

Pythagorean<br />

Theorem<br />

Segment Addition<br />

Postulate<br />

Parallel/Proportionality<br />

Theorem<br />

AA Similarity<br />

Postulate<br />

Right Angles<br />

Are Congruent<br />

Theorem<br />

SSS<br />

Congruence<br />

Postulate<br />

AA Similarity<br />

Postulate<br />

14. approximately 1.5 cm or 6.5 cm<br />

15a. C<br />

15b. A<br />

15c. A<br />

15d. A<br />

15e. C<br />

16a. The vectors are diagonals of your quadrilateral.<br />

16b. A 180° rotation about the midpoint of the<br />

common side; the entire tessellation maps onto<br />

itself.<br />

17a. a 180° rotation about the midpoint of any<br />

side<br />

17b. possible answer: a vector running from each<br />

vertex of the quadrilateral to the opposite vertex<br />

(or any multiple of that vector)<br />

18a. 33°<br />

18b. 66°<br />

18c. 57°<br />

18d. 62°<br />

18e. PSQ and PTQ are 90° by the Tangent<br />

Theorem and so are supplementary. So, SPT<br />

and SQT must also be supplementary by the<br />

Quadrilateral Sum Theorem. Therefore, opposite<br />

angles are supplementary, so it’s cyclic.<br />

18f. Because PS is a tangent, mPSQ 90°<br />

(Tangent Theorem). Because mPSQ 90°, SQ<br />

must be a tangent (Converse of the Tangent<br />

Theorem).<br />

<br />

19a. 12<br />

1<br />

19b. <br />

3<br />

2<br />

3<br />

<br />

6<br />

4 1<br />

20a. CDG CFG by SAA; GEA GEB<br />

by SAS; DGA FGB by the HL Theorem<br />

20b. CD CF and DA FB by CPCTC; CD <br />

DA CF FB (addition property of equality).<br />

Therefore, CA CB, and ABC is isosceles.<br />

20c. The figure is inaccurate.<br />

20d. The angle bisector does not intersect the<br />

perpendicular bisector inside the triangle as<br />

shown, except in the special case of an isosceles<br />

triangle, when they coincide.<br />

21. 173 cm, 346 cm, 20 stones. Draw the trapezoid<br />

and extend the legs until they meet to form a<br />

triangle. Use parallel proportionality to find the<br />

rise. The span is twice the rise. Use inverse tangent<br />

to find the central angle measure. Divide into 180°<br />

to find the number of voussoirs.<br />

ANSWERS TO EXERCISES 155<br />

<strong>Answers</strong> to Exercises


<strong>Answers</strong> to Exercises<br />

USING YOUR ALGEBRA SKILLS 13<br />

1. B(a, 0)<br />

2. C(a b, c)<br />

3. Ca b, a2 b2<br />

, Db, a2 b<br />

4. possible answer:<br />

T (0, b)<br />

156 ANSWERS TO EXERCISES<br />

2 <br />

0 0 0<br />

slope of RE a<br />

0<br />

a<br />

0<br />

b 0<br />

slope of CE a<br />

a<br />

b<br />

0 <br />

(undefined)<br />

slope of TC b<br />

b 0<br />

<br />

a 0<br />

a<br />

0<br />

slope of TR b<br />

0<br />

<br />

0 0<br />

b<br />

0 <br />

(undefined)<br />

Opposite sides have the same slope and are<br />

therefore parallel by the parallel slope property.<br />

Two sides are horizontal and two sides are vertical,<br />

so the angles are all congruent right angles. RECT<br />

is an equiangular parallelogram and is a rectangle<br />

by definition.<br />

5. possible answer:<br />

T (0, 0)<br />

y<br />

y<br />

R (0, 0)<br />

Z<br />

b ( ,<br />

c<br />

2 2)<br />

X<br />

a ( , 0<br />

2 )<br />

I (b, c)<br />

C (a, b)<br />

x<br />

E (a, 0)<br />

Y<br />

a + b ( ,<br />

c )<br />

2<br />

R (a, 0)<br />

2<br />

x<br />

Let X be the midpoint of TR.<br />

0 0 0<br />

X a , 2 2 a<br />

2 ,0<br />

Let Y be the midpoint of RI .<br />

b 0 c b c<br />

Y a , 2 2 a , 2 2<br />

Let Z be the midpoint of TI .<br />

0 c 0<br />

Z b , 2 2 b<br />

2 , c 2<br />

X, Y, and Z are the midpoints of TR,RI , and TI ,<br />

respectively, by the coordinate midpoint property.<br />

So XY,YZ, and ZX are midsegments by definition.<br />

6. possible answer:<br />

y<br />

P (c, d)<br />

T (0, 0)<br />

A (a – c, d)<br />

R (a, 0)<br />

0 0 0<br />

slope of TR a<br />

0<br />

a<br />

0<br />

d 0 d<br />

slope of RA a c a<br />

c<br />

d d 0<br />

slope of AP a<br />

2c<br />

a 2c<br />

0<br />

slope of PT d<br />

0<br />

<br />

c 0<br />

d<br />

c <br />

TR and AP have the same slope and are parallel by<br />

the parallel slope property. So TRAP has only one<br />

pair of parallel sides and is a trapezoid by definition.<br />

PT (c ) 0 2 (d 0) 2 c2 d2<br />

RA (a c a) 2 (d 0) 2 (c) 2 d2 <br />

c2 d2<br />

The nonparallel sides of the trapezoid have the<br />

same length. So trapezoid TRAP is isosceles by<br />

definition.<br />

7.<br />

y<br />

U (0, a 3)<br />

E Q<br />

(–a, 0) (a, 0)<br />

Show: EQU is equilateral<br />

EQ 2a<br />

EU (a 0) 2 a3)<br />

(0 2 <br />

a2 a 3 2 <br />

4a2 <br />

2a<br />

UQ (a ) 0 2 0 a<br />

a2 a 3 2 <br />

4a2 <br />

2a<br />

EQ EU UQ<br />

EQU is equilateral<br />

x<br />

x<br />

<br />

3 2


8. Task 1: Given: A rectangle with both diagonals<br />

Show: The diagonals are congruent<br />

Task 2: y<br />

T (0, b)<br />

Task 3: Given: Rectangle RECT with diagonals RC<br />

and TE<br />

Show: RC TE<br />

Task 4: To show that two segments are congruent,<br />

you use the distance formula to show that they<br />

have the same length.<br />

Task 5: RC (a ) 0 2 (b 0) 2 a2 b2<br />

<br />

TE (a ) 0 2 (0 b) 2 a2 b2<br />

So RC TE because both segments have the same<br />

length. Therefore the diagonals of a rectangle are<br />

congruent.<br />

9. Task 1: Given: A triangle with one midsegment<br />

Show: The midsegment is parallel to and half the<br />

length of the third side<br />

Task 2: y<br />

T (0, 0)<br />

R (0, 0)<br />

Z<br />

c ( , )<br />

2 d 2<br />

I (c, d)<br />

C (a, b)<br />

x<br />

E (a, 0)<br />

Y<br />

a + c ( , )<br />

R (a, 0)<br />

Task 3: Given: TRI and midsegment YZ<br />

Show: YZ TR and YZ 1<br />

2 TR<br />

Task 4: To show that two segments are parallel, use<br />

the parallel slope property. The segments are<br />

horizontal, so to compare lengths, subtract their<br />

x-coordinates.<br />

Task 5:<br />

0 0 0<br />

Slope of TR a<br />

0<br />

a<br />

0<br />

Slope of YZ 0<br />

The slopes are the same, so the segments are parallel<br />

by the parallel slope property.<br />

TR a 0 a<br />

a c c a<br />

YZ 2<br />

2<br />

2<br />

1 a<br />

2<br />

1<br />

2 TR<br />

0<br />

<br />

a<br />

<br />

2 <br />

d<br />

<br />

2<br />

d<br />

2 <br />

<br />

a c c<br />

2<br />

2<br />

2<br />

d 2<br />

x<br />

So the midsegment is half the length of the third<br />

side. Therefore the midsegment of a triangle is<br />

parallel to the third side and half the length of the<br />

third side.<br />

10. Task 1: Given: A trapezoid<br />

Show: The midsegment is parallel to the bases<br />

Task 2: y<br />

One possible set of the coordinates for TRAP is<br />

shown in the figure. By the coordinate midpoint<br />

property, the coordinates of M are b<br />

2 , c 2 and of N<br />

a d c<br />

are , . 2 2<br />

Task 3: Given: Trapezoid TRAP with midsegment<br />

MN <br />

Show: MN TR<br />

Task 4: To show that the midsegment and bases<br />

are parallel, you need to find their slopes.<br />

Task 5:<br />

slope of MN <br />

c c<br />

2<br />

2 <br />

0<br />

0<br />

a d a d b<br />

2<br />

<br />

2<br />

0 0 0<br />

slope of TR a<br />

0<br />

a<br />

0<br />

b<br />

2 <br />

The slope of PA is d<br />

c c 0<br />

<br />

b <br />

d b<br />

0. The slopes are<br />

equal, therefore the lines are parallel.<br />

11. Task 1: Given: A quadrilateral in which only<br />

one diagonal is the perpendicular bisector of the<br />

other<br />

Show: The quadrilateral is a kite<br />

Task 2:<br />

y<br />

T (–a, 0)<br />

P (b, c) A (d, c)<br />

M N<br />

T (0, 0)<br />

I (0, b)<br />

M(0, 0)<br />

E (0, c)<br />

R (a, 0)<br />

K(a, 0)<br />

Task 3: Given: Quadrilateral KITE with diagonal<br />

IE, which is the perpendicular bisector of diagonal<br />

TK<br />

x<br />

x<br />

ANSWERS TO EXERCISES 157<br />

<strong>Answers</strong> to Exercises


<strong>Answers</strong> to Exercises<br />

Show: KITE is a kite<br />

Task 4: To show that a quadrilateral is a kite, you<br />

use the distance formula to show that only two<br />

pairs of adjacent sides have the same length.<br />

Task 5:<br />

KI (0 ) a 2 b ( 0) 2 a2 b2<br />

IT (0 a)) ( 2 (b 0) 2 a2 b2<br />

TE (0 a)) ( 2 (c 0) 2 a2 c2<br />

EK (a ) 0 2 (0 c) 2 a2 c2<br />

<br />

Adjacent sides KI and IT have the same length and<br />

adjacent sides TE and EK have the same length,<br />

and because ⏐b⏐ ⏐c⏐ the pairs are not equal in<br />

length to each other. Therefore, KITE is a kite by<br />

definition. Therefore, if only one diagonal of a<br />

quadrilateral is the perpendicular bisector of the<br />

other diagonal, then the quadrilateral is a kite.<br />

12. Task 1: Given: A quadrilateral with midpoints<br />

connected to form a second quadrilateral<br />

Show: The second quadrilateral is a parallelogram<br />

Task 2: y<br />

( )<br />

d<br />

,<br />

e<br />

2 2<br />

G<br />

Q (0, 0)<br />

D (d, e)<br />

P<br />

a ( , 0<br />

2 )<br />

b + d<br />

,<br />

c + e<br />

2 2<br />

A (b, c)<br />

L ( )<br />

U (a, 0)<br />

Task 3: Given: Quadrilateral QUAD with<br />

midpoints P, R, L, and G<br />

Show: PRLG is a parallelogram<br />

Task 4: To show that a quadrilateral is a<br />

parallelogram, we need to show that opposite<br />

sides have the same slope.<br />

c<br />

Task 5: slope of PR b<br />

e<br />

slope of RL d a<br />

c<br />

slope of LG b<br />

e e<br />

2<br />

0<br />

2 e<br />

slope of GP d a<br />

d a<br />

2<br />

d<br />

<br />

2<br />

a<br />

2 <br />

c<br />

<br />

2<br />

<br />

<br />

e c e<br />

2<br />

2<br />

<br />

b<br />

<br />

2<br />

d<br />

c<br />

2<br />

<br />

<br />

c e c e<br />

2<br />

2 2<br />

<br />

b d a b d a<br />

2<br />

2 2<br />

d<br />

<br />

2<br />

b <br />

2<br />

b<br />

2 <br />

c<br />

2<br />

0<br />

<br />

a b<br />

2<br />

a<br />

2 <br />

158 ANSWERS TO EXERCISES<br />

a + b<br />

,<br />

c<br />

2 2<br />

R ( )<br />

x<br />

Opposite sides PR and LG have the same slope, and<br />

opposite sides RL and GP have the same slope. So<br />

they are parallel by the parallel slope property, and<br />

PRLG is a parallelogram by definition. Therefore<br />

the figure formed by connecting the midpoints of<br />

the sides of a quadrilateral is a parallelogram.<br />

13. Task 1: Given: An isosceles triangle with the<br />

midpoint of the base connected to the midpoint of<br />

each leg, to form a quadrilateral<br />

Show: The quadrilateral is a rhombus<br />

Task 2: y<br />

Task 3: Given: Isosceles triangle ABC with<br />

midpoint of base, E, and midpoints of legs, D and<br />

F, connected to form quadrilateral ADEF.<br />

Show: ADEF is a rhombus<br />

Task 4: You need to show that all the sides of<br />

ADEF have the same length.<br />

Task 5: By the distance formula,<br />

2<br />

2<br />

AD a a 2<br />

<br />

h h 2 a 2<br />

2<br />

h 2<br />

<br />

2<br />

<br />

h<br />

2<br />

2<br />

h a 2<br />

2<br />

h 2 <br />

AF 3 a<br />

<br />

2<br />

a 2<br />

<br />

<br />

DE a a<br />

2<br />

<br />

2<br />

<br />

0 h<br />

2 2<br />

2<br />

2<br />

<br />

a 2<br />

h 2 2<br />

<br />

2<br />

2<br />

EF 3 a<br />

<br />

2<br />

a 2<br />

h 2<br />

2<br />

0 a 2<br />

h 2<br />

<br />

a<br />

D ( ,<br />

h<br />

2 2)<br />

a2 h2<br />

<br />

2<br />

a2 h2<br />

<br />

2<br />

a2 h2<br />

<br />

2<br />

a2 h2<br />

<br />

2<br />

A (a, h)<br />

B (0, 0) E (a, 0)<br />

3a<br />

F ( ,<br />

h<br />

2 2)<br />

C (2a, 0)<br />

AD AF DE EF by the transitive property<br />

of equality.<br />

Therefore, ADEF is a rhombus by the definition of<br />

a rhombus.<br />

x


CHAPTER 13 REVIEW<br />

1. False. The quadrilateral could be an isosceles<br />

trapezoid.<br />

2. true<br />

3. False. The figure could be an isosceles trapezoid<br />

or a kite.<br />

4. true<br />

5. False. The angles are supplementary but not<br />

necessarily congruent.<br />

6. False. See Lesson 13.5, Example B.<br />

7. true<br />

8. perpendicular<br />

9. congruent<br />

10. the center of the circle<br />

11. four congruent triangles that are similar to the<br />

original triangle<br />

12. an auxiliary theorem proven specifically to<br />

help prove other theorems<br />

13. If a segment joins the midpoints of the<br />

diagonals of a trapezoid, then it is parallel to the<br />

bases.<br />

14. Angle Bisector Postulate<br />

15. Perpendicular Postulate<br />

16. Assume the opposite of what you want to<br />

prove, then use valid reasoning to derive a contradiction.<br />

17a. Smoking is not glamorous.<br />

17b. If smoking were glamorous, then this smoker<br />

would look glamorous. This smoker does not look<br />

glamorous, therefore smoking is not glamorous.<br />

18. False. The parallelogram is a rhombus.<br />

19. False. Possible counterexample:<br />

140<br />

20<br />

40 40<br />

140 20<br />

20 20<br />

20. False; x 360° 2b so x 90° only if<br />

b 135°.<br />

x<br />

a a<br />

b<br />

x 540 (180 2b)<br />

x 360 2b<br />

b<br />

21. True (except in the special case of an isosceles<br />

right triangle, in which the segment is not defined<br />

because the feet coincide).<br />

Given: IsoscelesABC with<br />

altitudes AD and BE;AC BC;ED<br />

Show: ED AB<br />

EAB DBA by the Isosceles<br />

Triangle Theorem. AEB <br />

BDA by the definition of<br />

altitude and by the Right Angles<br />

Are Congruent Theorem.<br />

AB AB by the reflexive property of congruence,<br />

so AEB BDA by SAA. AE BD by<br />

CPCTC. By the definition of congruence, AC BC<br />

and AE BD, so EC DC by the subtraction<br />

property of equality and the Segment Addition<br />

Postulate. By the division property of equality,<br />

EC<br />

AE<br />

D<br />

C<br />

E<br />

D<br />

X<br />

A<br />

B<br />

C ,so BD ED divides the sides of ABC<br />

proportionally. Therefore ED AB by the<br />

Converse of the Parallel/Proportionality Theorem.<br />

22. true<br />

Given: Rhombus ROME with diagonals RM and<br />

EO intersecting at B<br />

E<br />

R<br />

2<br />

1<br />

4 B<br />

3<br />

O<br />

M<br />

Show: RM EO<br />

Because diagonals bisect the angles in a rhombus,<br />

the diagonals are angle bisectors.<br />

Statement Reason<br />

1. 1 2 1. Rhombus Angles<br />

Theorem<br />

2. RO RE 2. Definition of rhombus<br />

3. RO RE 3. Definition of<br />

congruence<br />

4. RB RB 4. Reflexive property of<br />

congruence<br />

5. ROB REB 5. SAS Congruence<br />

Postulate<br />

6. 3 4 6. CPCTC<br />

7. 3 and 4 are 7. Definition of linear<br />

a linear pair pair<br />

8. 3 and 4 are 8. Linear Pair Postulate<br />

supplementary<br />

9. 3 and 4 are right 9. Congruent and<br />

angles Supplementary<br />

Theorem<br />

10. RM EO 10. Definition of<br />

perpendicular<br />

ANSWERS TO EXERCISES 159<br />

<strong>Answers</strong> to Exercises


<strong>Answers</strong> to Exercises<br />

23. true<br />

D E<br />

1 3<br />

A F B<br />

BAD DCB by the Opposite Angles Theorem.<br />

m1 1<br />

<br />

2 mBAD 1 <br />

2 mDCB m2 by<br />

the definition of angle bisector. AB DC by the<br />

definition of parallelogram. 2 and 3 are<br />

supplementary by the Interior Supplements<br />

Theorem.<br />

1 and 3 are supplementary by the<br />

substitution property.<br />

AE FC by the Converse of the Interior<br />

Supplements Theorem.<br />

24. Use the Inscribed Angle Theorem, the addition<br />

property, and the distributive property to get<br />

mP mE mN mT mA <br />

1<br />

2 mTN mAT mPA mEP mNE .Because<br />

there are 360° in a circle, mP mE mN <br />

mT mA 180°.<br />

25. T<br />

R<br />

Assume mH 45° and mT 45°. Use the<br />

Triangle Sum Theorem, the substitution<br />

property, and the subtraction property to get<br />

mH mT 90°, which creates a<br />

contradiction.Therefore mH45° or mT45°.<br />

26. Y<br />

M<br />

T R<br />

Use the definition of midpoint, the Segment<br />

Addition Postulate, the substitution property, and<br />

the division property to get MY<br />

<br />

TY 1<br />

2 and YS<br />

<br />

YR<br />

1<br />

2 .<br />

Then use the reflexive property and the SAS<br />

Similarity Theorem to get MSY TRY.<br />

Therefore, MS 1 TR 2 by CSSTP and the<br />

multiplication property and MS TR by<br />

CASTC and the CA Postulate.<br />

27. D Y<br />

T<br />

1 3<br />

Z<br />

4 2<br />

O P<br />

Use the Line Postulate to extend ZO and DR. Then<br />

use the Line Intersection Postulate to label P as the<br />

intersection of ZO and DR . DYR POR by the<br />

160 ANSWERS TO EXERCISES<br />

2<br />

S<br />

C<br />

H<br />

R<br />

SAA Theorem; thus DY OP by CPCTC. Use<br />

the Triangle Midsegment Theorem and the<br />

substitution property to get TR 1<br />

<br />

2 (ZO DY).<br />

Also, use the Triangle Midsegment Theorem to<br />

get TR ZO.<br />

28a. The quadrilateral formed when the<br />

midpoints of the sides of a rectangle are connected<br />

is a rhombus.<br />

28b.<br />

H<br />

Use the Right Angles Are Congruent Theorem and<br />

the SAS Congruence Postulate to get AEH <br />

BEF DGH CGF. Then use CPCTC to<br />

prove that EFGH is a rhombus.<br />

29a. The quadrilateral formed when the<br />

midpoints of the sides of a rhombus are connected<br />

is a rectangle.<br />

29b.<br />

I<br />

L<br />

D<br />

A<br />

P<br />

O<br />

M<br />

Y X<br />

By the Triangle Midsegment Theorem both PM <br />

and ON are parallel to LJ , and both PO and MN <br />

are parallel to IK. Because LJ and IK are<br />

perpendicular, we can use corresponding angles<br />

on parallel lines to prove that the lines that are<br />

adjacent sides of the quadrilateral are also<br />

perpendicular.<br />

30a. The quadrilateral formed when the midpoints<br />

of the sides of a kite are connected is a rectangle.<br />

30b. By the Triangle Midsegment<br />

I<br />

Theorem both PM and ON are<br />

parallel to LJ , and both PO and<br />

MN are parallel to IK. Because<br />

L<br />

P M<br />

J<br />

LJ and IK are perpendicular, we can<br />

use the CA Postulate to prove that<br />

O N<br />

the lines that are adjacent sides of<br />

the quadrilateral are also perpendicular.<br />

K<br />

31. Use the Line Postulate<br />

to construct chords DB and<br />

C<br />

AC. Then use the Inscribed<br />

Angles Intercepting Arcs<br />

P<br />

B<br />

Theorem and the AA<br />

Similarity Postulate to get<br />

APC DPB. Therefore,<br />

A<br />

O<br />

by CSSTP and the multiplication<br />

property, AP PB DP PC.<br />

K<br />

G<br />

E<br />

N<br />

C<br />

B<br />

J<br />

F<br />

D

Hooray! Your file is uploaded and ready to be published.

Saved successfully!

Ooh no, something went wrong!